Resp - 4th semester- URDEN/lewis

Réussis tes devoirs et examens dès maintenant avec Quizwiz!

1. How do microorganisms reach the lungs and cause pneumonia (select all that apply)? a. Aspiration b. Lymphatic spread c. Inhalation of microbes in the air d. Touch contact with the infectious microbes e. Hematogenous spread from infections elsewhere in the body

1. a, c, e. Microorganisms that cause pneumonia reach the lungs by aspiration from the nasopharynx or oropharynx, inhalation of microbes in the air, and hematogenous spread from infections elsewhere in the body. The other causes of infection do not contribute to pneumonia.

1. When explaining respiratory failure to the patient's family, what should the nurse use as an accurate description? a. The absence of ventilation b. Any episode in which part of the airway is obstructed c. Inadequate gas exchange to meet the metabolic needs of the body d. An episode of acute hypoxemia caused by a pulmonary dysfunction

1. c. Respiratory failure results when the transfer of oxygen or carbon dioxide function of the respiratory system is impaired and, although the definition is determined by PaO2 and PaCO2 levels, the major factor in respiratory failure is inadequate gas exchange to meet tissue oxygen (O2) needs. Absence of ventilation is respiratory arrest and partial airway obstruction may not necessarily cause respiratory failure. Acute hypoxemia may be caused by factors other than pulmonary dysfunction.

10. Which changes of aging contribute to the increased risk for respiratory failure in older adults (select all that apply)? a. Alveolar dilation b. Increased delirium c. Changes in vital signs d. Increased infection risk e. Decreased respiratory muscle strength f. Diminished elastic recoil within the airways

10. a, d, e, f. Changes from aging that increase the older adult's risk for respiratory failure include alveolar dilation, increased risk for infection, decreased respiratory muscle strength, and diminished elastic recoil in the airways. Although delirium can complicate ventilator management, it does not increase the older patient's risk for respiratory failure. The older adult's blood pressure (BP) and heart rate (HR) increase but this does not affect the risk for respiratory failure. The ventilatory capacity is decreased and the larger air spaces decrease the surface area for gas exchange, which increases the risk.

10. Following assessment of a patient with pneumonia, the nurse identifies a nursing diagnosis of impaired gas exchange based on which finding? a. SpO2 of 86% c. Temperature of 101.4oF (38.6oC) b. Crackles in both lower lobes d. Production of greenish purulent sputum

10. a. Oxygen saturation obtained by pulse oximetry should be between 90% and 100%. An SpO2 lower than 90% indicates hypoxemia and impaired gas exchange. Crackles, purulent sputum, and fever are all manifestations of pneumonia, but do not necessarily relate to impaired gas exchange.

10. A 73-year-old patient has an SpO2 of 70%. What other assessment should the nurse consider before making a judgment about the adequacy of the patient's oxygenation? a. What the oxygenation status is with a stress test b. Trend and rate of development of the hyperkalemia c. Comparison of patient's SpO2 values with the normal values d. Comparison of patient's current vital signs with normal vital signs

10. d. The respiratory rate, pulse rate, and blood pressure will all increase with decreased oxygenation when compared to the patient's own normal results. The position of the oximeter should also be assessed. The oxygenation status with a stress test would not assist the nurse in caring for the patient now. Hyperkalemia is not occurring and will not directly affect oxygenation initially. The SpO2 compared with normal values will not be helpful in this older patient or in a patient with respiratory disease, as the patient's expected normal will not be the same as standard normal values.

11. A patient with pneumonia has a nursing diagnosis of ineffective airway clearance related to pain, fatigue, and thick secretions. What is an expected outcome for this patient? a. SpO2 is 90% b. Lungs clear to auscultation c. Patient tolerates walking in hallway d. Patient takes three or four shallow breaths before coughing to minimize pain

11. b. Clear lung sounds indicate that the airways are clear. SpO2 of 90% to 100% indicates appropriate gas exchange. Tolerating walking in the hallway also indicates appropriate gas exchange, not improved airway clearance. Deep breaths are necessary to move mucus from distal airways but this is not an outcome.

11. Which values are indicators of the criteria needed for the use of continuous oxygen therapy? a. SpO2 of 92%; PaO2 of 65 mm Hg c. SpO2 of 90%; PaO2 of 60 mm Hg b. SpO2 of 95%; PaO2 of 70 mm Hg d. SpO2 of 88%; PaO2 of 55 mm Hg

11. d. An SpO2 of 88% and a PaO2 of 55 mm Hg indicate inadequate oxygenation and are the criteria for prescription of continuous oxygen therapy (see Table 26-3). These values may be adequate for patients with chronic hypoxemia if no cardiac problems occur but will affect the patients' activity tolerance.

11. The nurse assesses that a patient in respiratory distress is developing respiratory fatigue and the risk of respiratory arrest when the patient displays which behavior? a. Cannot breathe unless he is sitting upright c. Has an increased inspiratory-expiratory (I/E) ratio b. Uses the abdominal muscles during expiration d. Has a change in respiratory rate from rapid to slow

11. d. The increase in respiratory rate required to blow off accumulated CO2 predisposes to respiratory muscle fatigue. The slowing of a rapid rate in a patient in acute distress indicates tiring and the possibility of respiratory arrest unless ventilatory assistance is provided. A decreased inspiratory-expiratory (I/E) ratio, orthopnea, and accessory muscle use are common findings in respiratory distress but do not necessarily signal respiratory fatigue or arrest.

12. A patient has a PaO2 of 50 mm Hg and a PaCO2 of 42 mm Hg because of an intrapulmonary shunt. Which therapy is the patient most likely to respond best to? a. Positive pressure ventilation b. Oxygen administration at a FIO2 of 100% c. Administration of O2 per nasal cannula at 1 to 3 L/min d. Clearance of airway secretions with coughing and suctioning

12. a. Patients with a shunt are usually more hypoxemic than patients with a V/Q mismatch because the alveoli are filled with fluid, which prevents gas exchange. Hypoxemia resulting from an intrapulmonary shunt is usually not responsive to high O2 concentrations and the patient will usually require positive pressure ventilation. Hypoxemia associated with a V/Q mismatch usually responds favorably to O2 administration at 1 to 3 L/min by nasal cannula. Removal of secretions with coughing and suctioning is generally not effective in reversing an acute hypoxemia resulting from a shunt.

12. During an annual health assessment of a 65-year-old patient at the clinic, the patient tells the nurse he had the pneumonia vaccine when he was age 58. What should the nurse advise him about the best way for him to prevent pneumonia? a. Seek medical care and antibiotic therapy for all upper respiratory infections b. Obtain the pneumococcal vaccine this year with an annual influenza vaccine c. Obtain the pneumococcal vaccine if he is exposed to individuals with pneumonia d. Obtain only the influenza vaccine every year because he has immunity to the pneumococcus

12. b. A second dose of the pneumococcal vaccine should be provided to all persons 65 years of age or older who have not received the vaccine within 5 years and were younger than 65 years of age at the time of vaccination. Influenza vaccine should be taken each year by those older than 65 years of age. Antibiotic therapy is not appropriate for all upper respiratory infections unless secondary bacterial infections develop.

13. A patient with a massive hemothorax and pneumothorax has absent breath sounds in the right lung. To promote improved V/Q matching, how should the nurse position the patient? a. On the left side c. In a reclining chair bed b. On the right side d. Supine with the head of the bed elevated

13. a. When there is impaired function of one lung, the patient should be positioned with the unaffected lung in the dependent position to promote perfusion to the functioning tissue. If the diseased lung is positioned dependently, more V/Q mismatch would occur. The head of the bed may be elevated or a reclining chair may be used, with the patient positioned on the unaffected side, to maximize thoracic expansion if the patient has increased work of breathing.

13. Which respiratory defense mechanism is most impaired by smoking? a. Cough reflex c. Mucociliary clearance b. Filtration of air d. Reflex bronchoconstriction

13. c. Ciliary action impaired by smoking and increased mucus production may be caused by the irritants in tobacco smoke, leading to impairment of the mucociliary clearance system. Smoking does not directly affect filtration of air, the cough reflex, or reflex bronchoconstriction but it does impair the respiratory defense mechanism provided by alveolar macrophages.

13. Tobacco smoke causes defects in multiple areas of the respiratory system. What is a long-term effect of smoking? a. Bronchospasm and hoarseness b. Decreased mucus secretions and cough c. Increased function of alveolar macrophages d. Increased risk of infection and hyperplasia of mucous glands

13. d. Increased risk of infection, hyperplasia of mucous glands, cancer, and chronic bronchitis are the long-term effects of smoking.

14. Which age-related changes in the respiratory system cause decreased secretion clearance (select all that apply)? a. Decreased functional cilia d. Small airway closure earlier in expiration b. Decreased force of cough e. Decreased functional immunoglobulin A (IgA) c. Decreased chest wall compliance

14. a, b. Decreased functional cilia and decreased force of cough from declining muscle strength cause decreased secretion clearance. The other options contribute to other age-related changes. Decreased compliance contributes to barrel chest appearance. Early small airway closure contributes to decreased PaO2. Decreased immunoglobulin A (IgA) decreases the resistance to infection.

14. A patient in hypercapnic respiratory failure has a nursing diagnosis of ineffective airway clearance related to increasing exhaustion. What is an appropriate nursing intervention for this patient? a. Inserting an oral airway c. Teaching the patient huff coughing b. Performing augmented coughing d. Teaching the patient slow pursed lip breathing

14. b. Augmented coughing is done by applying pressure on the abdominal muscles at the beginning of expiration. This type of coughing helps to increase abdominal pressure and expiratory flow to assist the cough to remove secretions in the patient who is exhausted. An oral airway is used only if there is a possibility that the tongue will obstruct the airway. Huff coughing prevents the glottis from closing during the cough and works well for patients with chronic obstructive pulmonary disease (COPD) to clear central airways. Slow pursed lip breathing allows more time for expiration and prevents small bronchioles from collapsing.

15. What causes the pulmonary vasoconstriction leading to the development of cor pulmonale in the patient with COPD? a. Increased viscosity of the blood b. Alveolar hypoxia and hypercapnia c. Long-term low-flow oxygen therapy d. Administration of high concentrations of oxygen

15. b. Constriction of the pulmonary vessels, leading to pulmonary hypertension, is caused by alveolar hypoxia and the acidosis that results from hypercapnia. Polycythemia is a contributing factor in cor pulmonale because it increases the viscosity of blood and the pressure needed to circulate the blood. Long-term low-flow oxygen therapy dilates pulmonary vessels and is used to treat cor pulmonale. High oxygen administration is not related to cor pulmonale.

15. A patient has central venous oxygen saturation/mixed venous oxygen saturation (ScvO2/SvO2) of 52%, CO of 4.8 L/min, SpO2 of 95%, and an unchanged hemoglobin level. What should the nurse assess the patient for? a. Dysrhythmias c. Pulmonary edema b. Pain on movement d. Signs of septic shock

15. b. The normal central venous oxygen saturation/mixed venous oxygen saturation (ScvO2/SvO2) of 60% to 80% becomes decreased with decreased arterial oxygenation, low CO, low hemoglobin, or increased oxygen consumption. With normal CO, arterial oxygenation, and hemoglobin, the factor that is responsible for decreased ScvO2/SvO2 is increased oxygen consumption, which can result from increased metabolic rate, pain, movement, or fever.

15. The patient with a history of heart failure and acute respiratory failure has thick secretions that she is having difficulty coughing up. Which intervention would best help to mobilize her secretions? a. Administer more IV fluid c. Provide O2 by aerosol mask b. Perform postural drainage d. Suction airways nasopharyngeally

15. c. For the patient with a history of heart failure, current acute respiratory failure, and thick secretions, the best intervention is to liquefy the secretions with either aerosol mask or using normal saline administered by a nebulizer. Excess IV fluid may cause cardiovascular distress and the patient probably would not tolerate postural drainage with her history. Suctioning thick secretions without thinning them is difficult and increases the patient's difficulty in maintaining oxygenation. With copious secretions, this could be done after thinning the secretions.

16. Priority Decision: After endotracheal intubation and mechanical ventilation have been started, a patient in respiratory failure becomes very agitated and is breathing asynchronously with the ventilator. What is it most important for the nurse to do first? a. Evaluate the patient's pain level, ABGs, and electrolyte values b. Sedate the patient to unconsciousness to eliminate patient awareness c. Administer the PRN vecuronium (Norcuron) to promote synchronous ventilations d. Slow the rate of ventilations provided by the ventilator to allow for spontaneous breathing by the patient

16. a. It is most important to assess the patient for the cause of the restlessness and agitation (e.g., pain, hypoxemia, electrolyte imbalances) and treat the underlying cause before sedating the patient. Although sedation, analgesia, and neuromuscular blockade are often used to control agitation and pain, these treatments may contribute to prolonged ventilator support and hospital days.

16. The abnormal assessment findings of dullness and hyperresonance are found with which assessment technique? a. Inspection c. Percussion b. Palpation d. Auscultation

16. c. Dullness and hyperresonance are found in the lungs using percussion, not the other assessment techniques.

16. In addition to smoking cessation, what treatment is included for COPD to slow the progression of the disease? a. Use of bronchodilator drugs b. Use of inhaled corticosteroids c. Lung volume-reduction surgery d. Prevention of respiratory tract infections

16. d. Smoking cessation is one of the most important factors in preventing further damage to the lungs in COPD but prevention of infections that further increase lung damage is also important. The patient is very susceptible to infections and these infections make the disease worse, creating a vicious cycle. Bronchodilators, inhaled corticosteroids, and lung volume-reduction surgery help to control symptoms but these are symptomatic measures.

17. Which method of oxygen administration is the safest system to use for a patient with COPD? a. Venturi mask b. Nasal cannula c. Simple face mask d. Non-rebreathing mask

17. a. A Venturi mask is helpful to administer low, constant O2 concentrations to patients with COPD and can be set to administer a varied percentage of O2. The amount of O2 inhaled via the nasal cannula depends on room air and the patient's breathing pattern. The simple face mask must have a tight seal and may generate heat under the mask. The non-rebreathing mask is more useful for short-term therapy with patients needing high O2 concentrations.

17. Palpation is the assessment technique used to find which abnormal assessment findings (select all that apply)? a. Stridor b. Finger clubbing c. Tracheal deviation d. Limited chest expansion e. Increased tactile fremitus f. Use of accessory muscles

17. c, d, e. Palpation identifies tracheal deviation, limited chest expansion, and increased tactile fremitus. Stridor is identified with auscultation. Finger clubbing and accessory muscle use are identified with inspection.

17. What is the primary reason that hemodynamic monitoring is instituted in severe respiratory failure? a. To detect V/Q mismatches b. To continuously measure the arterial BP c. To evaluate oxygenation and ventilation status d. To evaluate cardiac status and blood flow to tissues

17. d. Hemodynamic monitoring with a pulmonary artery catheter is instituted in severe respiratory failure to determine the amount of blood flow to tissues and the response of the lungs and heart to hypoxemia. Continuous BP monitoring may be performed but BP is a reflection of cardiac activity, which can be determined by the pulmonary artery catheter findings. Arterial blood gases (ABGs) are important to evaluate oxygenation and ventilation status and V/Q mismatches.

18. Patients with acute respiratory failure will have drug therapy to meet their individual needs. Which drugs will meet the goal of reducing pulmonary congestion (select all that apply)? a. Morphine b. Furosemide (Lasix) c. Nitroglycerin (Tridil) d. Albuterol (Ventolin) e. Ceftriaxone (Rocephin) f. Methylprednisolone (Solu-Medrol)

18. a, b, c. Morphine and nitroglycerin (e.g., Tridil) will decrease pulmonary congestion caused by heart failure; IV diuretics (e.g., furosemide [Lasix]) are also used. Inhaled albuterol (Ventolin) or metaproterenol (Alupent) will relieve bronchospasms. Ceftriaxone (Rocephin) and azithromycin (Zithromax) are used to treat pulmonary infections. Methylprednisolone (Solu-Medrol), an IV corticosteroid, will reduce airway inflammation. Morphine is also used to decrease anxiety, agitation, and pain.

18. Priority Decision: To reduce the risk for most occupational lung diseases, what is the most important measure the occupational nurse should promote? a. Maintaining smoke-free work environments for all employees. b. Using masks and effective ventilation systems to reduce exposure to irritants. c. Inspection and monitoring of workplaces by national occupational safety agencies. d. Requiring periodic chest x-rays and pulmonary function tests for exposed employees.

18. b. Although all of the precautions identified in this question are appropriate in decreasing the risk of occupational lung diseases, using masks and effective ventilation systems to reduce exposure is the most efficient and affects the greatest number of employees.

18. What is characteristic of a partial rebreathing mask? a. Used for long-term O2 therapy b. Reservoir bag conserves oxygen c. Provides highest oxygen concentrations d. Most comfortable and causes the least restriction on activities

18. b. The partial rebreathing mask has O2 flow into the reservoir bag and mask during inhalation. The O2-conserving cannula is used for long-term therapy at home versus during hospitalization. The Venturi mask is able to deliver the highest concentrations of O2. The nasal cannula is the most comfortable and mobile delivery device.

18. How does the nurse assess the patient's chest expansion? a. Put the palms of the hands against the chest wall. b. Put the index fingers on either side of the trachea. c. Place the thumbs at the midline of the lower chest. d. Place one hand on the lower anterior chest and one hand on the upper abdomen.

18. c. To assess the extent and symmetry of chest movement, the nurse places the hands over the lower anterior chest wall along the costal margin and moves them inward until the thumbs meet at the midline and then asks the patient to breathe deeply and observes the movement of the thumbs away from each other. The palms are placed against the chest wall to assess tactile fremitus. To determine the tracheal position, the nurse places the index fingers on either side of the trachea just above the suprasternal notch and gently presses backward.

19. When does the nurse record the presence of an increased anteroposterior (AP) diameter of the chest? a. There is a prominent protrusion of the sternum. b. The width of the chest is equal to the depth of the chest. c. There is equal but diminished movement of the two sides of the chest. d. The patient cannot fully expand the lungs because of kyphosis of the spine.

19. b. An increased AP diameter is characteristic of a barrel chest, in which the AP diameter is about equal to the side-to-side diameter. Normally the AP diameter should be one third to one half the side-to-side diameter. A prominent protrusion of the sternum is the pectus carinatum and diminished movement of the two sides of the chest indicates decreased chest excursion. Lack of lung expansion caused by kyphosis of the spine results in shallow breathing with decreased chest expansion.

19. A patient is being discharged with plans for home O2 therapy provided by an O2 concentrator with an O2-conserving portable unit. In preparing the patient to use the equipment, what should the nurse teach the patient? a. The portable unit will last about 6 to 8 hours. b. The unit is strictly for portable and emergency use. c. The unit concentrates O2 from the air, providing a continuous O2 supply. d. Weekly delivery of one large cylinder of O2 will be necessary for a 7- to 10-day supply of O2.

19. c. Oxygen concentrators or extractors continuously supply O2 concentrated from the air. O2-conserving units will last for up to 20 hours. Portable liquid O2 units will hold about 6 to 8 hours of O2 but because of the expense they are only used for portable and emergency use. Compressed O2 comes in various tank sizes but generally it requires weekly deliveries of four to five large tanks to meet a 7- to 10-day supply.

29. The nurse suspects that a patient with PEEP is experiencing negative effects of this ventilatory maneuver when which of the following is assessed? a. Increasing PaO2 c. Decreasing heart rate (HR) b. Decreasing blood pressure d. Increasing central venous pressure (CVP)

29. b. PEEP increases intrathoracic and intrapulmonic pressures, compresses the pulmonary capillary bed, and reduces blood return to both the right and left sides of the heart. Increased PaO2 is an expected effect of PEEP. Preload (CVP) and cardiac output (CO) are decreased, often with a dramatic decrease in BP.

19. In caring for a patient in acute respiratory failure, the nurse recognizes that noninvasive positive pressure ventilation (NIPPV) may be indicated for which patient? a. Is comatose and has high oxygen requirements b. Has copious secretions that require frequent suctioning c. Responds to hourly bronchodilator nebulization treatments d. Is alert and cooperative but has increasing respiratory exhaustion

19. d. Noninvasive positive pressure ventilation (NIPPV) involves the application of a face mask and delivery of a volume of air under inspiratory pressure. Because the device is worn externally, the patient must be able to cooperate in its use and frequent access to the airway for suctioning or inhaled medications must not be necessary. It is not indicated when high levels of oxygen are needed or respirations are absent.

2. Which descriptions are characteristic of hypoxemic respiratory failure (select all that apply)? a. Referred to as ventilatory failure b. Primary problem is inadequate O2 transfer c. Risk of inadequate O2 saturation of hemoglobin exists d. Body is unable to compensate for acidemia of increased PaCO2 e. Most often caused by ventilation-perfusion (V/Q) mismatch and shunt f. Exists when PaO2 is 60 mm Hg or less, even when O2 is administered at 60%

2. b, c, e, f. Hypoxemic respiratory failure is often caused by ventilation-perfusion (V/Q) mismatch and shunt. It is called oxygenation failure because the primary problem is inadequate oxygen transfer. There is a risk of inadequate oxygen saturation of hemoglobin and it exists when PaO2 is 60 mm Hg or less, even when oxygen is administered at 60%. Ventilatory failure is hypercapnic respiratory failure. Hypercapnic respiratory failure results from an imbalance between ventilatory supply and ventilatory demand and the body is unable to compensate for the acidemia of increased PaCO2.

2. A 92-year-old female patient is being admitted to the emergency department with severe shortness of breath. Being aware of the patient's condition, what approach should the nurse use to assess the patient's lungs (select all that apply)? a. Apex to base b. Base to apex c. Lateral sequence d. Anterior then posterior e. Posterior then anterior

2. b, e. This patient is older and short of breath. To obtain the most information, auscultate the posterior to avoid breast tissue and start at the base because of her respiratory difficulty and the chance that she will tire easily. Important sounds may be missed if the other strategies are used first.

22. Which dietary modification helps to meet the nutritional needs of patients with COPD? a. Eating a high-carbohydrate, low-fat diet b. Avoiding foods that require a lot of chewing c. Preparing most foods of the diet to be eaten hot d. Drinking fluids with meals to promote digestion

22. b. Eating is an effort for patients with COPD and frequently these patients do not eat because of fatigue, dyspnea, and difficulty holding their breath while swallowing. Foods that require much chewing cause more exhaustion and should be avoided. A low-carbohydrate diet is indicated if the patient has hypercapnia because carbohydrates are metabolized into carbon dioxide. Fluids should be avoided at meals to prevent a full stomach and cold foods seem to give less of a sense of fullness than hot foods.

12. Why does a patient's respiratory rate increase when there is an excess of carbon dioxide in the blood? a. CO2 displaces oxygen on hemoglobin, leading to a decreased PaO2. b. CO2 causes an increase in the amount of hydrogen ions available in the body. c. CO2 combines with water to form carbonic acid, which lowers the pH of cerebrospinal fluid. d. CO2 directly stimulates chemoreceptors in the medulla to increase respiratory rate and volume.

2. c. A combination of excess CO2 and H2O results in carbonic acid, which lowers the pH of cerebrospinal fluid and stimulates an increase in the respiratory rate. Peripheral chemoreceptors in the carotid and aortic bodies also respond to increases in PaCO2 to stimulate the respiratory center. Excess CO2 does not increase the amount of hydrogen ions available in the body but does combine with the hydrogen of water to form an acid.

2. Why is the classification of pneumonia as community-acquired pneumonia (CAP) or medical care-associated pneumonia (MCAP) clinically useful? a. Atypical pneumonia syndrome is more likely to occur in MCAP. b. Diagnostic testing does not have to be used to identify causative agents. c. Causative agents can be predicted and empiric treatment is often effective. d. IV antibiotic therapy is necessary for MCAP but oral therapy is adequate for CAP.

2. c. Pneumonia that has its onset in the community is usually caused by different microorganisms than pneumonia that develops related to hospitalization and treatment can be empiric—based on observations and experience without knowing the exact causative organism. Frequently a causative organism cannot be identified from cultures and treatment is based on experience.

20. The patient progressed from acute lung injury to acute respiratory distress syndrome (ARDS). He is on the ventilator and receiving propofol (Diprivan) for sedation and fentanyl (Sublimaze) to decrease anxiety, agitation, and pain in order to decrease his work of breathing, O2 consumption, carbon dioxide production, and risk of injury. What intervention is recommended in caring for this patient? a. A sedation holiday c. Keeping his legs still to avoid dislodging the airway b. Monitoring for hypermetabolism d. Repositioning him every 4 hours to decrease agitation

20. a. A sedation holiday is needed to assess the patient's condition and readiness to extubate. A hypermetabolic state occurs with critical illness. With malnourished patients, enteral or parenteral nutrition is started within 24 hours; with well-nourished patients it is started within 3 days. With these medications, the patient will be assessed for cardiopulmonary depression. Venous thromboembolism prophylaxis will be used but there is no reason to keep the legs still. Repositioning the patient every 2 hours may help to decrease discomfort and agitation.

20. Which breathing technique should the nurse teach the patient with moderate COPD to promote exhalation? a. Huff coughing b. Thoracic breathing c. Pursed lip breathing d. Diaphragmatic breathing

20. c. Pursed lip breathing prolongs exhalation and prevents bronchiolar collapse and air trapping. Huff coughing is a technique used to increase coughing patterns to remove secretions. Thoracic breathing is not as effective as diaphragmatic breathing and is the method most naturally used by patients with COPD. Diaphragmatic breathing emphasizes the use of the diaphragm to increase maximum inhalation but it may increase the work of breathing and dyspnea.

20. How is the presence of bronchovesicular breath sounds in the peripheral lung fields described? a. Rhonchi c. Adventitious sounds b. Crackles d. Abnormal lung sounds

20. d. Bronchovesicular breath sounds are normal breath sounds when they are heard anteriorly over the mainstem bronchi on either side of the sternum and posteriorly between the scapulae. If they are heard in the peripheral lung fields, they are considered abnormal breath sounds. Adventitious lung sounds are extra abnormal sounds that include crackles, rhonchi, wheezes, and pleural friction rubs. See Table 26-7 for descriptions of these sounds.

21. Although ARDS may result from direct lung injury or indirect lung injury as a result of systemic inflammatory response syndrome (SIRS), the nurse is aware that ARDS is most likely to occur in the patient with a host insult resulting from a. sepsis. c. prolonged hypotension. b. oxygen toxicity. d. cardiopulmonary bypass.

21. a. Although ARDS may occur in the patient who has virtually any severe illness and may be both a cause and a result of systemic inflammatory response syndrome (SIRS), the most common precipitating insults of ARDS are sepsis, gastric aspiration, and severe massive trauma.

21. What does the nurse include when planning for postural drainage for the patient with COPD? a. Schedules the procedure 1 hour before and after meals b. Has the patient cough before positioning to clear the lungs c. Assesses the patient's tolerance for dependent (head-down) positions d. Ensures that percussion and vibration are performed before positioning the patient

21. c. Many postural drainage positions require placement in Trendelenburg position but patients with heart disease, hemoptysis, chest trauma, or severe dyspnea should not be placed in these positions. Postural drainage should be done 1 hour before and 3 hours after meals if possible. Coughing, percussion, and vibration are all performed after the patient has been positioned.

22. What are the primary pathophysiologic changes that occur in the injury or exudative phase of ARDS (select all that apply)? a. Atelectasis d. Hyaline membranes line the alveoli b. Shortness of breath e. Influx of neutrophils, monocytes, and lymphocytes c. Interstitial and alveolar edema

22. a, c, d. The injury or exudative phase is the early phase of ARDS when atelectasis and interstitial and alveoli edema occur and hyaline membranes composed of necrotic cells, protein, and fibrin line the alveoli. Together, these decrease gas exchange capability and lung compliance. Shortness of breath occurs but it is not a physiologic change. The increased inflammation and proliferation of fibroblasts occurs in the reparative or proliferative phase of ARDS, which occurs 1 to 2 weeks after the initial lung injury.

22. A nurse has been exposed to tuberculosis (TB) during care of a patient with TB and has a TB skin test performed. Wan is the nurse considered infected? a. There is no redness or induration at the injection site. b. There is an induration of only 5 mm at the injection site. c. A negative skin test is followed by a negative chest x-ray. d. Testing causes a 10-mm red, indurated area at the injection site.

22. d. A 10-mm red indurated injection site could be a positive result for a nurse as an employee in a high-risk setting. Because antibody production in response to infection with the TB bacillus may not be sufficient to produce a reaction to TB skin testing immediately after infection, two-step testing is recommended for individuals likely to be tested often, such as health care providers. An initial negative skin test should be repeated in 1 to 3 weeks and if the second test is negative, the individual can be considered uninfected. All other answers indicate a negative response to skin testing.

23. Delegation Decision: The nurse is caring for a patient with COPD. Which intervention could be delegated to unlicensed assistive personnel (UAP)? a. Assist the patient to get out of bed. b. Auscultate breath sounds every 4 hours. c. Plan patient activities to minimize exertion. d. Teach the patient pursed lip breathing technique.

23. a. Assistance with positioning and activities of daily living (ADL) is within the training of unlicensed assistive personnel (UAP). Teaching, assessing, and planning are all part of the RN's practice.

23. In preparing a patient in the ICU for oral ET intubation, what should the nurse do? a. Place the patient supine with the head extended and the neck flexed. b. Tell the patient that the tongue must be extruded while the tube is inserted. c. Position the patient supine with the head hanging over the edge of the bed to align the mouth and trachea. d. Inform the patient that while it will not be possible to talk during insertion of the tube, speech will be possible after it is correctly placed.

23. a. The patient is positioned with the mouth, pharynx, and trachea in direct alignment, with the head extended in the "sniffing position," but the head must not hang over the edge of the bed. The patient may be asked to extrude the tongue during nasal intubation. Speaking is not possible during intubation or while the oral ET tube is in place because the tube separates the vocal cords.

23. In patients with ARDS who survive the acute phase of lung injury, what manifestations are seen when they progress to the fibrotic phase? a. Chronic pulmonary edema and atelectasis b. Resolution of edema and healing of lung tissue c. Continued hypoxemia because of diffusion limitation d. Increased lung compliance caused by the breakdown of fibrotic tissue

23. c. In the fibrotic phase of ARDS, diffuse scarring and fibrosis of the lungs occur, resulting in decreased surface area for gas exchange and continued hypoxemia caused by diffusion limitation. Although edema is resolved, lung compliance is decreased because of interstitial fibrosis. Long-term mechanical ventilation is required. The patient has a poor prognosis for survival.

23. What is a primary nursing responsibility after obtaining a blood specimen for ABGs? a. Adding heparin to the blood specimen b. Applying pressure to the puncture site for 2 full minutes c. Taking the specimen immediately to the laboratory in an iced container d. Avoiding any changes in oxygen intervention for 20 minutes following the procedure

23. c. Samples for ABGs must be iced to keep the gases dissolved in the blood (unless the specimen is to be analyzed in <1 minute) and taken directly to the lab. The syringe used to obtain the specimen is rinsed with heparin before the specimen is taken and pressure is applied to the arterial puncture site for 5 minutes after obtaining the specimen. Changes in oxygen therapy or interventions should be avoided for 20 minutes before the specimen is drawn because these changes might alter blood gas values.

24. What should the nurse do when preparing a patient for a pulmonary angiogram scan? a. Assess the patient for iodine allergy. b. Implement NPO orders for 6 to 12 hours before the test. c. Explain the test before the patient signs the informed consent form. d. Inform the patient that radiation isolation for 24 hours after the test is necessary.

24. a. A pulmonary angiogram involves the injection of an iodine-based radiopaque dye and iodine or shellfish allergies should be assessed before injection. A bronchoscopy requires NPO status for 6 to 12 hours before the test and invasive tests (e.g., bronchoscopy, mediastinoscopy, biopsies) require informed consent that the physician should obtain from the patient. Nuclear scans use radioactive materials for diagnosis but the amounts are very small and no radiation precautions are indicated for the patient.

24. In caring for the patient with ARDS, what is the most characteristic sign the nurse would expect the patient to exhibit? a. Refractory hypoxemia c. Progressive hypercapnia b. Bronchial breath sounds d. Increased pulmonary artery wedge pressure (PAWP)

24. a. Refractory hypoxemia, hypoxemia that does not respond to increasing concentrations of oxygenation by any route, is a hallmark of ARDS and is always present. Bronchial breath sounds may be associated with the progression of ARDS. PaCO2 levels may be normal until the patient is no longer able to compensate in response to the hypoxemia. Pulmonary artery wedge pressure (PAWP) that is normally elevated in cardiogenic pulmonary edema is normal in the pulmonary edema of ARDS.

24. Priority Decision: A patient has an oral ET tube inserted to relieve an upper airway obstruction and to facilitate secretion removal. What is the first responsibility of the nurse immediately following placement of the tube? a. Suction the tube to remove secretions. b. Place an end tidal CO2 detector on the ET tube. c. Secure the tube to the face with adhesive tape. d. Assess for bilateral breath sounds and symmetric chest movement.

24. b. The first action of the nurse is to use an end tidal CO2 detector. If no CO2 is detected, the tube is in the esophagus. The second action by the nurse following ET intubation is to auscultate the chest to confirm bilateral breath sounds and observe to confirm bilateral chest expansion. If this evidence is present, the tube is secured and connected to an O2 source. Then the placement is confirmed immediately with x-ray and the tube is marked where it exits the mouth. The patient should be suctioned as needed.

24. Which medication is a dry powder inhaler (DPI) that is used only for COPD? a. Roflumilast (Daliresp) b. Salmeterol (Serevent) c. Ipratropium (Atrovent HFA) d. Indacterol (Arcapta Neohaler)

24. d. Indacterol (Arcapta Neohaler) is a DPI that is used only for COPD. Roflumilast (Daliresp) is an oral medication used for COPD. Salmeterol (Serevent) is a DPI but it is also used in asthma with inhaled corticosteroids. Ipratropium (Atrovent HFA) is used for COPD but it is delivered via metered-dose inhaler or nebulizer.

24. A male patient has chronic obstructive pulmonary disease (COPD) and is a smoker. The nurse notices respiratory distress and no breath sounds over the left chest. Which type of pneumothorax should the nurse suspect is occurring? a. Tension pneumothorax c. Traumatic pneumothorax b. Iatrogenic pneumothorax d. Spontaneous pneumothorax

24. d. Spontaneous pneumothorax is seen from the rupture of small blebs on the apex of the lung in patients with lung disease or smoking, as well as in tall, thin males with a family history of or a previous spontaneous pneumothorax. Tension pneumothorax occurs with mechanical ventilation and with blocked chest tubes. Iatrogenic pneumothorax occurs due to the laceration or puncture of the lung during medical procedures. Traumatic pneumothorax can occur with penetrating or blunt chest trauma.

25. To determine whether a tension pneumothorax is developing in a patient with chest trauma, for what does the nurse assess the patient? a. Dull percussion sounds on the injured side b. Severe respiratory distress and tracheal deviation c. Muffled and distant heart sounds with decreasing blood pressure d. Decreased movement and diminished breath sounds on the affected side

25. b. A tension pneumothorax causes many of the same manifestations as other types of pneumothoraces but severe respiratory distress from collapse of the entire lung with movement of the mediastinal structures and trachea to the unaffected side is present in a tension pneumothorax. Percussion dullness on the injured site indicates the presence of blood or fluid and decreased movement and diminished breath sounds are characteristic of a pneumothorax. Muffled and distant heart sounds indicate a cardiac tamponade.

25. The nurse suspects the early stage of ARDS in any seriously ill patient who manifests what? a. Develops respiratory acidosis c. Exhibits dyspnea and restlessness b. Has diffuse crackles and rhonchi d. Has a decreased PaO2 and an increased PaCO2

25. c. Early signs of ARDS are insidious and difficult to detect but the nurse should be alert for any early signs of hypoxemia, such as dyspnea, restlessness, tachypnea, cough, and decreased mentation, in patients at risk for ARDS. Abnormal findings on physical examination or diagnostic studies, such as adventitious lung sounds, signs of respiratory distress, respiratory alkalosis, or decreasing PaO2, are usually indications that ARDS has progressed beyond the initial stages.

25. The nurse uses the minimal occluding volume technique to inflate the cuff on an ET tube to minimize the incidence of what? a. Infection c. Tracheal necrosis b. Hypoxemia d. Accidental extubation

25. c. The minimal occluding volume (MOV) involves adding air to the ET tube cuff until no leak is heard at peak inspiratory pressure but ensures that minimal pressure is applied to the tracheal wall to prevent pressure necrosis of the trachea. The MOV should be between 20 and 25 cm H2O of pressure to prevent tracheal injury. The cuff does not secure the tube in place but rather prevents escape of ventilating gases through the upper airway.

25. Priority Decision: During an acute exacerbation of mild COPD, the patient is severely short of breath and the nurse identifies a nursing diagnosis of ineffective breathing pattern related to obstruction of airflow and anxiety. What is the best action by the nurse? a. Prepare and administer routine bronchodilator medications. b. Perform chest physiotherapy to promote removal of secretions. c. Administer oxygen at 5 L/min until the shortness of breath is relieved. d. Position the patient upright with the elbows resting on the over-the-bed table.

25. d. The tripod position with an elevated backrest and supported upper extremities to fix the shoulder girdle maximizes respiratory excursion and an effective breathing pattern. Staying with the patient and encouraging pursed lip breathing also helps. Bronchodilators may help but can also increase nervousness and anxiety; rescue inhalers would be used before routine bronchodilators. Postural drainage is not tolerated by a patient in acute respiratory distress and oxygen is titrated to an effective rate based on ABGs because of the possibility of carbon dioxide narcosis.

26. When suctioning an ET tube, the nurse should use a suction pressure of mm Hg.

26. 100 to 120 mm Hg

26. After which test should the nurse observe the patient for symptoms of a pneumothorax? a. Thoracentesis c. Ventilation-perfusion scan b. Pulmonary function test d. Positron emission tomography (PET) scan

26. a. The greatest chance for a pneumothorax occurs with a thoracentesis because of the possibility of lung tissue injury during this procedure. Ventilation-perfusion scans and positron emission tomography (PET) scans involve injections but no manipulation of the respiratory tract is involved. Pulmonary function tests are noninvasive.

26. A patient with ARDS has a nursing diagnosis of risk for infection. To detect the presence of infections commonly associated with ARDS, what should the nurse monitor? a. Gastric aspirate for pH and blood c. Subcutaneous emphysema of the face, neck, and chest b. Quality, quantity, and consistency of sputum d. Mucous membranes of the oral cavity for open lesions

26. b. Ventilator-associated pneumonia (VAP) is one of the most common complications of ARDS. Early detection requires frequent monitoring of sputum smears and cultures and assessment of the quality, quantity, and consistency of sputum. Prevention of VAP is done with strict infection control measures, ventilator bundle protocol, and subglottal secretion drainage. Blood in gastric aspirate may indicate a stress ulcer and subcutaneous emphysema of the face, neck, and chest occurs with barotrauma during mechanical ventilation. Oral infections may result from prophylactic antibiotics and impaired host defenses but are not common.

26. Following a motor vehicle accident, the nurse assesses the driver for which distinctive sign of flail chest? a. Severe hypotension c. Absence of breath sounds b. Chest pain over ribs d. Paradoxical chest movement

26. d. Flail chest may occur when two or more ribs are fractured, causing an unstable segment. The chest wall cannot provide the support for ventilation and the injured segment will move paradoxically to the stable portion of the chest (in on expiration; out on inspiration). Hypotension occurs with a number of conditions that impair cardiac function, and chest pain occurs with a single fractured rib and will be of high priority with flail chest. Absent breath sounds occur following pneumothorax or hemothorax.

26. The husband of a patient with severe COPD tells the nurse that he and his wife have not had any sexual activity since she was diagnosed with COPD because she becomes too short of breath. What is the nurse's best response? a. "You need to discuss your feelings and needs with your wife so she knows what you expect of her." b. "There are other ways to maintain intimacy besides sexual intercourse that will not make her short of breath." c. "You should explore other ways to meet your sexual needs since your wife is no longer capable of sexual activity." d. "Would you like for me to talk to you and your wife about some modifications that can be made to maintain sexual activity?"

26. d. Specific guidelines for sexual activity help to preserve energy and prevent dyspnea and maintenance of sexual activity is important to the healthy psychologic well-being of the patient. Open communication between partners is needed so that the modifications can be made with consideration of both partners.

27. The best patient response to treatment of ARDS occurs when initial management includes what? a. Treatment of the underlying condition c. Treatment with diuretics and mild fluid restriction b. Administration of prophylactic antibiotics d. Endotracheal intubation and mechanical ventilation

27. a. Because ARDS is precipitated by a physiologic insult, a critical factor in its prevention and early management is treatment of the underlying condition. Prophylactic antibiotics, treatment with diuretics and fluid restriction, and mechanical ventilation are also used as ARDS progresses.

27. What should the nurse include when teaching the patient with COPD about the need for physical exercise? a. All patients with COPD should be able to increase walking gradually up to 20 minutes per day. b. A bronchodilator inhaler should be used to relieve exercise-induced dyspnea immediately after exercise. c. Shortness of breath is expected during exercise but should return to baseline within 5 minutes after the exercise. d. Monitoring the heart rate before and after exercise is the best way to determine how much exercise can be tolerated.

27. c. Shortness of breath usually increases during exercise but the activity is not being overdone if breathing returns to baseline within 5 minutes after stopping. Bronchodilators can be administered 10 minutes before exercise but should not be administered for at least 5 minutes after activity to allow recovery. Patients are encouraged to walk 15 to 20 minutes per day with gradual increases but actual patterns will depend on patient tolerance. Dyspnea most frequently limits exercise and is a better indication of exercise tolerance than is heart rate in the patient with COPD.

27. The nurse suctions the patient's ET tube when the patient has what? a. Peripheral wheezes in all lobes c. Coarse rhonchi over central airways b. Has not been suctioned for 2 hours d. A need for stimulation to cough and deep breathe

27. c. Suctioning an ET tube is performed when adventitious sounds over the trachea or bronchi confirm the presence of secretions that can be removed by suctioning. Visible secretions in the ET tube, respiratory distress, suspected aspiration, increase in peak airway pressures, and changes in oxygen status are other indications. Peripheral wheezes or crackles are not an indication for suctioning. Suctioning as a means of inducing a cough is not recommended because of the complications associated with suctioning.

27. The health care provider orders a pulmonary angiogram for a patient admitted with dyspnea and hemoptysis. For which problem is this test most commonly used as a diagnostic measure? a. Tuberculosis (TB) c. Airway obstruction b. Cancer of the lung d. Pulmonary embolism

27. d. A pulmonary angiogram outlines the pulmonary vasculature and is useful to diagnose obstructions or pathologic conditions of the pulmonary vessels, such as a pulmonary embolus. The tissue changes of TB and cancer of the lung may be diagnosed by chest x-ray or computed tomography (CT), magnetic resonance imaging (MRI), or PET scans. Airway obstruction is most often diagnosed with pulmonary function testing.

28. When mechanical ventilation is used for the patient with ARDS, what is the rationale for applying positive end- expiratory pressure (PEEP)? a. Prevent alveolar collapse and open up collapsed alveoli b. Permit smaller tidal volumes with permissive hypercapnia c. Promote complete emptying of the lungs during exhalation d. Permit extracorporeal oxygenation and carbon dioxide removal outside the body

28. a. Positive end-expiratory pressure (PEEP) used with mechanical ventilation applies positive pressure to the airway and lungs at the end of exhalation, keeping the lung partially expanded and preventing collapse of the alveoli and helping to open up collapsed alveoli. Permissive hypercapnia is allowed when the patient with ARDS is ventilated with smaller tidal volumes to prevent barotrauma. Extracorporeal membrane oxygenation and extracorporeal CO2 removal involve passing blood across a gas-exchanging membrane outside the body and then returning oxygenated blood to the body.

28. The patient has had COPD for years and his ABGs usually show hypoxia (PaO2 <60 mm Hg or SaO2 <88%) and hypercapnia (PaCO2 >45 mm Hg). Which ABG results show movement toward respiratory acidosis and further hypoxia indicating respiratory failure? a. pH 7.35, PaO2 62 mm Hg, PaCO2 45 mm Hg b. pH 7.34, PaO2 45 mm Hg, PaCO2 65 mm Hg c. pH 7.42, PaO2 90 mm Hg, PaCO2 43 mm Hg d. pH 7.46, PaO2 92 mm Hg, PaCO2 32 mm Hg

28. b. These results show worsening respiratory function and failure. The results in option a show potential normal results for the patient described. The results in option c show normal ABGs. The results in option d show alkalosis, probably respiratory, but the HCO - results are needed to be sure.

28. What nursing care is included for the patient with an ET tube? a. Check the cuff pressure every hour. c. Hyperoxygenate before and after suctioning. b. Keep a tracheostomy tray at the bedside. d. Reuse the suction catheter at the bedside for 24 hours.

28. c. Nursing care for a patient with an ET tube includes (1) hyperoxygenation before and after suctioning, (2) keeping suctioning equipment and a self-inflating bag-valve-mask (BVM) at the bedside, and (3) using either one-time use sterile suction catheters for open suction technique or a suction catheter that is enclosed in a plastic sleeve connected directly to the patient ventilator circuit, which is changed per facility protocol for the closed suction technique. Used suction catheters are not left at the bedside.

29. Pulmonary rehabilitation (PR) is designed to reduce symptoms and improve the patient's quality of life. Along with improving exercise capacity, what are the anticipated results of PR (select all that apply)? a. Decreased FEV1 b. Decreased anxiety c. Decreased depression d. Increased oxygen need e. Decreased hospitalizations

29. b, c, e. Decreasing anxiety, depression, and hospitalizations along with improving exercise capacity are the anticipated effects of pulmonary rehabilitation. The other options may occur but are not predicted.

29. When should the nurse check for leaks in the chest tube and pleural drainage system? a. There is continuous bubbling in the water-seal chamber. b. There is constant bubbling of water in the suction control chamber. c. Fluid in the water-seal chamber fluctuates with the patient's breathing. d. The water levels in the water-seal and suction control chambers are decreased.

29. b. The water-seal chamber should bubble intermittently as air leaves the lung with exhalation in a spontaneously breathing patient. Continuous bubbling indicates a leak. The water in the suction control chamber will bubble continuously and the fluid in the water-seal chamber fluctuates with the patient's breathing. Water in the suction control chamber, and perhaps in the water- seal chamber, evaporates and may need to be replaced periodically.

29. Priority Decision: While suctioning the ET tube of a spontaneously breathing patient, the nurse notes that the patient develops bradycardia with premature ventricular contractions. What should the nurse do first? a. Stop the suctioning and assess the patient for spontaneous respirations. b. Attempt to resuction the patient with reduced suction pressure and pass time. c. Stop the suctioning and ventilate the patient with slow, small-volume breaths using a bag-valve-mask (BVM) device. d. Stop suctioning and ventilate the patient with a BVM device with 100% oxygen until the HR returns to baseline.

29. d. If new dysrhythmias occur during suctioning, the suctioning should be stopped and the patient should be slowly ventilated via BVM with 100% oxygen until the dysrhythmia subsides. Patients with bradycardia should not be suctioned excessively. Ventilation of the patient with slow, small-volume breaths using the BVM is performed when severe coughing results from suctioning.

33. Which patient's medical diagnosis should the nurse know is most likely to need mechanical ventilation (select all that apply)? a. Sleep apnea d. Type 2 diabetes mellitus b. Cystic fibrosis e. Acute respiratory distress syndrome (ARDS) c. Acute kidney failure

33. b, e. Cystic fibrosis and acute respiratory distress syndrome (ARDS) are the most likely of these diagnoses to need mechanical ventilation related to severe hypoxia or respiratory muscle fatigue. Other indications for mechanical ventilation are apnea or impending inability to breathe and acute respiratory failure.

3. What keeps alveoli from collapsing? a. Carina b. Surfactant c. Empyema d. Thoracic cage

3 . b. Surfactant is a lipoprotein that lowers the surface tension in the alveoli. It reduces the pressure needed to inflate the alveoli and decreases the tendency of the alveoli to collapse. The other options do not maintain inflation of the alveoli. The carina is the point of bifurcation of the trachea into the right and left bronchi. Empyema is a collection of pus in the thoracic cavity. The thoracic cage is formed by the ribs and protects the thoracic organs.

3. ThemicroorganismsPneumocystisjiroveci(PCP)andcytomegalovirus(CMV)areassociatedwithwhichtypeofpneumonia? a. Bronchial pneumonia c. Hospital-associated pneumonia b. Opportunistic pneumonia d. Community-acquired pneumonia

3. b. People at risk for opportunistic pneumonia include those with altered immune responses. Pneumocystis jiroveci rarely causes pneumonia in healthy individuals but is the most common cause of pneumonia in persons with HIV disease. Cytomegalovirus (CMV) occurs in people with an impaired immune response. Medical care-associated pneumonia is frequently caused by Pseudomonas aeruginosa, Escherichia coli, Klebsiella, and Acinetobacter species. Community- acquired pneumonia is most commonly caused by Streptococcus pneumonia.

3. When teaching the patient about what was happening when experiencing an intrapulmonary shunt, which explanation is accurate? a. This occurs when an obstruction impairs the flow of blood to the ventilated areas of the lung. b. This occurs when blood passes through an anatomic channel in the heart and bypasses the lungs. c. This occurs when blood flows through the capillaries in the lungs without participating in gas exchange. d. Gas exchange across the alveolar capillary interface is compromised by thickened or damaged alveolar membranes.

3. c. Intrapulmonary shunt occurs when blood flows through the capillaries in the lungs without participating in gas exchange (e.g., acute respiratory distress syndrome [ARDS], pneumonia). Obstruction impairs the flow of blood to the ventilated areas of the lung in a V/Q mismatch ratio greater than 1 (e.g., pulmonary embolus). Blood passes through an anatomic channel in the heart and bypasses the lungs with anatomic shunt (e.g., ventricular septal defect). Gas exchange across the alveolar capillary interface is compromised by thickened or damaged alveolar membranes in diffusion limitation (e.g., pulmonary fibrosis, ARDS).

30. What precautions should the nurse take during mouth care and repositioning of an oral ET tube to prevent and detect tube dislodgement (select all that apply)? a. Confirm bilateral breath sounds after care. b. Use suction pressures less than 120 mm Hg. c. Use water swabs to prevent mucosal drying. d. Use humidified inspired gas to help thin secretions. e. One staff member holds the tube and one performs care. f. Move secretions into larger airways with turning every 2 hours.

30. a, e. To prevent dislodgement of the oral ET tube during care, two nurses work together; one holds the tube while it is unsecured and the other performs care. After completion of care, confirm the presence of bilateral breath sounds to ensure that the position of the tube was not changed and reconfirm cuff pressure. Suction pressure less than 120 mm Hg will prevent tracheal mucosal damage. Although the use of water swabs prevents mucosal drying, humidified inspired gas helps to thin secretions. Secretions are moved to larger airways with turning, postural drainage, and percussion; these actions will not prevent or detect tube dislodgement.

30. An unlicensed assistive personnel (UAP) is taking care of a patient with a chest tube. The nurse should intervene when she observes the UAP a. looping the drainage tubing on the bed. b. securing the drainage container in an upright position. c. stripping or milking the chest tube to promote drainage. d. reminding the patient to cough and deep breathe every 2 hours.

30. c. If chest tubes are to be milked or stripped, this procedure should be done only by the professional nurse. This procedure is no longer recommended, as it may dangerously increase pleural pressure, but there is no indication to milk the tubes when there is no bloody drainage, as in a pneumothorax. The UAP can loop the chest tubing on the bed to promote drainage and patients should be reminded to cough and deep breathe at least every 2 hours to aid in lung reexpansion. Securing the drainage container in an upright position is also a necessary activity that can be completed by UAP.

30. Prone positioning is considered for a patient with ARDS who has not responded to other measures to increase PaO2. The nurse knows that this strategy will a. increase the mobilization of pulmonary secretions. b. decrease the workload of the diaphragm and intercostal muscles. c. promote opening of atelectatic alveoli in the upper portion of the lung. d. promote perfusion of nonatelectatic alveoli in the anterior portion of the lung.

30. d. When a patient with ARDS is supine, alveoli in the posterior areas of the lung are dependent and fluid-filled and the heart and mediastinal contents place more pressure on the lungs, predisposing to atelectasis. If the patient is turned prone, air-filled nonatelectatic alveoli in the anterior portion of the lung receive more blood and perfusion may be better matched to ventilation, causing less V/Q mismatch. Lateral rotation therapy is used to stimulate postural drainage and help mobilize pulmonary secretions.

31. A What are appropriate nursing interventions for this patient (select all that apply)? a. Assess gag reflex. b. Ensure that the cuff is properly inflated. c. Suction the patient's mouth frequently. d. Keep the ventilator tubing cleared of condensed water. e. Raise the head of the bed 30 to 45 degrees unless the patient is unstable.

31. b, c, e. Because the patient with an ET tube cannot protect the airway from aspiration and cannot swallow, the cuff should always be inflated and the head of the bed (HOB) elevated while the patient is receiving tube feedings or mouth care is being performed. The HOB elevated 30 to 45 degrees reduces risk of aspiration. The mouth and oropharynx should be suctioned with Yankauer or tonsil suction to remove accumulated secretions that cannot be swallowed. Clearing the ventilatory tubing of condensed water is important to prevent respiratory infection.

31. Which chest surgery is used for the stripping of a fibrous membrane? a. Lobectomy b. Decortication c. Thoracotomy d. Wedge resection

31. b. Decortication is the stripping of a thick fibrous membrane. A lobectomy is the removal of one lung lobe. A thoracotomy is the incision into the thorax. A wedge resection is used to remove a small lesion.

32. Priority Decision: Although his oxygen saturation is above 92%, an orally intubated, mechanically ventilated patient is restless and very anxious. What intervention should be used first to decrease the risk of accidental extubation? a. Obtain an order and apply soft wrist restraints. b. Remind the patient that he needs the tube inserted to breathe. c. Administer sedatives and have a caregiver stay with the patient. d. Move the patient to an area close to the nurses' station for closer observation.

32. c. Sedation may be appropriate. As well, having someone the patient knows at the bedside talking to him and reassuring him may decrease his anxiety and calm him. Restraints have not been shown to be an absolute deterrent to self-extubation and the patient will need ongoing and frequent assessment of need. Reminding the patient of the need for the tube may help but it may not be enough to prevent him from pulling out the tube if he becomes extremely anxious. Moving the patient near the nurses' station will not be enough to prevent self-extubation since it can be done quickly.

33. Following a thoracotomy, the patient has a nursing diagnosis of ineffective airway clearance related to inability to cough as a result of pain and positioning. What is the best nursing intervention for this patient? a. Have the patient drink 16 oz of water before attempting to deep breathe. b. Auscultate the lungs before and after deep-breathing and coughing regimens. c. Place the patient in the Trendelenburg position for 30 minutes before the coughing exercises. d. Medicate the patient with analgesics 20 to 30 minutes before assisting to cough and deep breathe.

33. d. A thoracotomy incision is large and involves cutting into bone, muscle, and cartilage, resulting in significant postoperative pain. The patient has difficulty deep breathing and coughing because of the pain and analgesics should be provided before attempting these activities. Water intake is important to liquefy secretions but is not indicated in this case, nor should a patient with chest trauma or surgery be placed in Trendelenburg position, because it increases intrathoracic pressure.

34. What characteristics describe positive pressure ventilators (select all that apply)? a. Require an artificial airway b. Applied to outside of the body c. Most similar to physiologic ventilation d. Most frequently used with acutely ill patients e. Frequently used in the home for neuromuscular or nervous system disorders

34. a, d. Positive pressure ventilators require an artificial airway and are most frequently used with acutely ill patients. The other options describe negative pressure ventilators.

35. What is included in the description of positive pressure ventilation (select all that apply)? a. Peak inspiratory pressure predetermined b. Consistent volume delivered with each breath c. Increased risk for hyperventilation and hypoventilation d. Preset volume of gas delivered with variable pressure based on compliance e. Volume delivered varies based on selected pressure and patient lung compliance

35. a, c, e. Positive pressure ventilation has a predetermined peak inspiratory pressure, which increases the risk for hyperventilation and hypoventilation because the volume delivered varies based on the selected pressure and the patient's lung compliance. The other options describe volume ventilation.

35. Priority Decision: Two days after undergoing pelvic surgery, a patient develops marked dyspnea and anxiety. What is the first action that the nurse should take? a. Raise the head of the bed. c. Take the patient's pulse and blood pressure. b. Notify the health care provider. d. Determine the patient's SpO2 with an oximeter.

35. a. All of the activities are correct but the first thing to do is to raise the head of the bed to promote respiration in the patient who is dyspneic. The health care provider would not be called until the nurse has assessment data relating to vital signs, pulse oximetry, and any other patient complaints.

35. Which obstructive pulmonary disease would a 30-year-old white female patient with a parent with the disease be most likely to be diagnosed with? a. COPD b. Asthma c. Cystic fibrosis d. α1-Antitrypsin (AAT) deficiency

35. d. α1-Antitrypsin (AAT) deficiency is an autosomal recessive disorder that is a genetic risk factor for COPD. AAT occurs in 1 in 1700 to 3500 live births with an onset between ages 20 and 40 years. Although cystic fibrosis occurs in 1 in 3000 white births, legislation requires babies to be screened at birth, so it would have been previously diagnosed. Asthma is a multifactorial genetic disorder.

36. A pulmonary embolus is suspected in a patient with a deep vein thrombosis who develops hemoptysis, tachycardia, and chest pain. Diagnostic testing is scheduled. Which test should the nurse plan to teach the patient about? a. Chest x-rays c. Take the patient's pulse and blood pressure. b. Spiral (helical) CT scan d. Ventilation-perfusion lung scan

36. b. A spiral (helical) CT scan is the most frequently used test to diagnose pulmonary emboli because it allows illumination of all anatomic structures and produces a 3-D picture. If a patient cannot have contrast media, a ventilation-perfusion scan is done. Although pulmonary angiography is most sensitive, it is invasive, expensive, and carries more risk for complications. Chest x-rays do not detect pulmonary emboli until necrosis or abscesses occur.

36. What is the primary principle involved in the various airway clearance devices used for mobilizing secretions? a. Vibration b. Inhalation therapy c. Chest physiotherapy d. Positive expiratory pressure

36. d. Positive expiratory pressure (PEP) is the principle behind the airway clearance devices (ACDs) that mobilize secretions and benefit patients. Vibration, a form of chest physiotherapy, and inhalation therapy are therapies to assist patients with excessive secretions or to increase bronchodilation but they are not principles of ACDs' function.

36. Which mode of ventilation is used with critically ill patients and allows the patient to self-regulate the rate and depth of spontaneous respirations but may also deliver a preset volume and frequency of breaths? a. Assist-control ventilation (ACV) b. Pressure support ventilation (PSV) c. Pressure-controlled inverse ratio ventilation (PC-IRV) d. Synchronized intermittent mandatory ventilation (SIMV)

36. d. Synchronized intermittent mandatory ventilation (SIMV) is described. Assist-control ventilation (ACV) has a preset tidal volume delivered at a set frequency and more frequently when the patient attempts to inhale. Pressure support ventilation (PSV) applies positive pressure only during inspiration that supplies a rapid flow of gas with spontaneous respirations. Pressure-controlled inverse ratio ventilation (PC-IRV) delivers prolonged inspiration and shortened expiration to promote alveolar expansion and prevent collapse.

37. Which condition contributes to secondary pulmonary arterial hypertension by causing pulmonary capillary and alveolar damage? a. COPD c. Pulmonary fibrosis b. Sarcoidosis d. Pulmonary embolism

37. a. Chronic obstructive pulmonary disease (COPD) causes pulmonary capillary and alveolar damage. Sarcoidosis is a granulomatous disease. Pulmonary fibrosis stiffens the pulmonary vasculature and pulmonary embolism obstructs pulmonary blood flow.

37. A patient in acute respiratory failure is receiving ACV with a positive end-expiratory pressure (PEEP) of 10 cm H2O. What sign alerts the nurse to undesirable effects of increased airway and thoracic pressure? a. Decreased BP c. Increased crackles b. Decreased PaO2 d. Decreased spontaneous respirations

37. a. Positive pressure ventilation, especially with end- expiratory pressure, increases intrathoracic pressure with compression of thoracic vessels, resulting in decreased venous return to the heart, decreased left ventricular end- diastolic volume (preload), decreased CO, and lowered BP. None of the other factors is related to increased intrathoracic pressure.

38. While caring for a patient with idiopathic pulmonary arterial hypertension (IPAH), the nurse observes that the patient has exertional dyspnea and chest pain in addition to fatigue. What are these symptoms related to? a. Decreased left ventricular output c. Increased systemic arterial blood pressure b. Right ventricular hypertrophy and dilation d. Development of alveolar interstitial edema

38. b. High pressure in the pulmonary arteries increases the workload of the right ventricle and eventually causes right ventricular hypertrophy and dilation, known as cor pulmonale. Eventually, decreased left ventricular output may occur because of decreased return to the left atrium but it is not the primary effect of pulmonary hypertension. Alveolar interstitial edema is pulmonary edema associated with left ventricular failure. Pulmonary hypertension does not cause systemic hypertension.

38. What should the nurse recognize as a factor commonly responsible for sodium and fluid retention in the patient on mechanical ventilation? a. Increased release of ADH c. Increased insensible water loss via the airway b. Increased release of atrial natriuretic factor d. Decreased renal perfusion with release of renin

38. d. Decreased CO associated with positive pressure ventilation and positive end-expiratory pressure (PEEP) results in decreased renal perfusion, release of renin, and increased aldosterone secretion, which causes sodium and water retention. ADH may be released because of stress but ADH is responsible only for water retention. Increased intrathoracic pressure will decrease, not increase, the release of atrial natriuretic factor, causing sodium retention.

39. What is a primary treatment goal for cor pulmonale? a. Controlling dysrhythmias c. Strengthening the cardiac muscle b. Dilating the pulmonary arteries d. Treating the underlying pulmonary condition

39. d. If possible, the primary management of cor pulmonale is treatment of the underlying pulmonary problem that caused the heart problem. Low-flow oxygen therapy will help to prevent hypoxemia and hypercapnia, which cause pulmonary vasoconstriction.

4. Which of the following microorganisms are associated with both CAP and MCAP (select all that apply)? a. Klebsiella b. Staphylococcus aureus c. Haemophilus influenzae d. Mycoplasma pneumonia e. Pseudomonas aeruginosa f. Streptococcus pneumonia

4. a, b, f. Community-acquired pneumonia (CAP) and medical care-associated pneumonia (MCAP) are both associated with Klebsiella, Staphylococcus aureus, and Streptococcus pneumonia. Haemophilus influenzae and Mycoplasma pneumonia are only associated with CAP. Pseudomonas aeruginosa is only associated with MCAP.

4. What accurately describes the alveolar sacs? a. Line the lung pleura b. Warm and moisturize inhaled air c. Terminal structures of the respiratory tract d. Contain dead air that is not available for gas exchange

4. c. Alveolar sacs are terminal structures of the respiratory tract where gas exchange takes place. The visceral pleura lines the lungs and forms a closed, double-walled sac with the parietal pleura. Turbinates warm and moisturize inhaled air. The 150 mL of air is dead space in the trachea and bronchi.

4. When the V/Q lung scan result returns with a mismatch ratio that is greater than 1, which condition should be suspected? a. Pain c. Pulmonary embolus b. Atelectasis d. Ventricular septal defect

4. c. There will be more ventilation than perfusion (V/Q ratio greater than 1) with a pulmonary embolus. Pain and atelectasis will cause a V/Q ratio less than 1. A ventricular septal defect causes an anatomic shunt as the blood bypasses the lungs.

40. Six days after a heart-lung transplant, the patient develops a low-grade fever, dyspnea, and decreased SpO2. What should the nurse recognize that this may indicate? a. A normal response to extensive surgery b. A frequently fatal cytomegalovirus infection c. Acute rejection that may be treated with corticosteroids d. Obliterative bronchiolitis that plugs terminal bronchioles

40. c. Acute rejection may occur as early as 5 to 7 days after surgery and is manifested by low-grade fever, fatigue, and oxygen desaturation with exertion. Complete remission of symptoms can be accomplished with bolus corticosteroids. Cytomegalovirus and other infections can be fatal but usually occur weeks after surgery and manifest with symptoms of pneumonia. Obliterative bronchiolitis is a late complication of lung transplantation, reflecting chronic rejection.

40. A patient receiving mechanical ventilation is very anxious and agitated and neuromuscular blocking agents are used to promote ventilation. What should the nurse recognize about the care of this patient? a. The patient will be too sedated to be aware of the details of care. b. Caregivers should be encouraged to provide stimulation and diversion. c. The patient should always be addressed and explanations of care given. d. Communication will not be possible with the use of neuromuscular blocking agents.

40. c. Neuromuscular blocking agents produce a paralysis that facilitates ventilation but they do not sedate the patient. It is important for the nurse to remember that the patient can hear, see, think, and feel and should be addressed and given explanations accordingly. Communication with the patient is possible, especially from the nurse, but visitors for an anxious and agitated patient should provide a calming, restful effect on the patient.

41. While receiving prolonged mechanical ventilation, the patient developed anemia. The patient is also having difficulty being weaned from the ventilator related to a recurrent pneumonia and early fatigue with weaning. What is contributing to the patient's prolonged recovery? a. Hypoxemia c. Inadequate nutrition b. Enteral feeding d. Decreased activity level

41. c. The patient is experiencing effects of inadequate nutrition: anemia, delayed ventilator weaning with decreased respiratory strength, decreased resistance to infection, and prolonged recovery. Hypoxemia is related to anemia. Enteral feeding would provide needed nutrition. Decreased activity may be related to muscle weakness from lack of nutrition.

42. The nurse determines that alveolar hypoventilation is occurring in a patient on a ventilator when what happens? a. The patient develops cardiac dysrhythmias. b. Auscultation reveals an air leak around the ET tube cuff. c. ABG results show a PaCO2 of 32 mm Hg and a pH of 7.47. d. The patient tries to breathe faster than the ventilator setting.

42. b. A leaking cuff can lower tidal volume or respiratory rates. An SIMV rate that is too low, the presence of lung secretions, or obstruction can decrease tidal volume. A decreased PaCO2 and increased pH indicate a respiratory alkalosis from hyperventilation and cardiac dysrhythmias can occur with either hyperventilation or hypoventilation.

43. What plan should the nurse use when weaning a patient from a ventilator? a. Decrease the delivered FIO2 concentration b. Intermittent trials of spontaneous ventilation followed by ventilatory support to provide rest c. Substitute ventilator support with a manual resuscitation bag if the patient becomes hypoxemic d. Implement weaning procedures around the clock until the patient does not experience ventilatory fatigue

43. b. A variety of ventilator weaning methods is used but all should provide weaning trials with adequate rest between trials to prevent respiratory muscle fatigue. Weaning is usually carried out during the day, with the patient ventilated at night until there is sufficient spontaneous ventilation without excess fatigue. If the patient becomes hypoxemic, ventilator support is indicated.

44. A patient is to be discharged home with mechanical ventilation. Before discharge, what is most important for the nurse to do for the patient and caregiver? a. Teach the caregiver to care for the patient with a home ventilator. b. Help the caregiver to plan for placement of the patient in a long-term care facility. c. Stress the advantages for the patient in being cared for in the home environment. d. Have the caregiver arrange for around-the-clock home health nurses for the first several wee

44. a. Care of a ventilator-dependent patient in the home requires that the caregiver know how to manage the ventilator and take care of the patient on it. Before final decisions and arrangements are made, the nurse should ensure that caregivers understand the potential sacrifices they may have to make and the impact that home mechanical ventilation will have over time. Placement in long-term care facilities is not usually necessary unless the caregiver can no longer manage the care or the patient's condition deteriorates.

5. Which physiologic mechanism of hypoxemia occurs with pulmonary fibrosis? a. Anatomic shunt c. Intrapulmonary shunt b. Diffusion limitation d. V/Q mismatch ratio of less than 1

5. b. Diffusion limitation in pulmonary fibrosis is caused by thickened alveolar-capillary interface, which slows gas transport.

5. What is an indication of marked bronchoconstriction with air trapping and hyperinflation of the lungs in a patient with asthma? a. SaO2 of 85% b. PEF rate of <150 L/min c. FEV1 of 85% of predicted d. Chest x-ray showing a flattened diaphragm

5. b. Peak expiratory flow rates (PEFR) are normally up to 600 L/min and in status asthmaticus may be as low as 100 to 150 L/min. An SaO2 of 85% and a FEV1 of 85% of predicted are typical of mild to moderate asthma. A flattened diaphragm may be present in the patient with long-standing asthma but does not reflect current bronchoconstriction.

5. What covers the larynx during swallowing? a. Trachea b. Epiglottis c. Turbinates d. Parietal pleur

5. b. The epiglottis is a small flap closing over the larynx during swallowing. The trachea separates the larynx and the bronchi. The turbinates warm and moisturize inhaled air. The parietal pleura is a membrane that lines the chest cavity.

6. When obtaining a health history from a 76-year-old patient with suspected CAP, what does the nurse expect the patient or caregiver to report? a. Confusion b. A recent loss of consciousness c. An abrupt onset of fever and chills d. A gradual onset of headache and sore throat

6. a. Confusion possibly related to hypoxia may be the only finding in older adults. Although CAP is most commonly caused by Staphylococcus aureus pneumonia and is associated with an acute onset with fever, chills, productive cough with purulent or bloody sputum, and pleuritic chest pain, the older patient may not have classic symptoms. Other causes of pneumonia have a more gradual onset with dry, hacking cough; headache; and sore throat. A recent loss of consciousness or altered consciousness is common in those pneumonias associated with aspiration, such as anaerobic bacterial pneumonias.

6. Priority Decision: A 75-year-old patient who is breathing room air has the following arterial blood gas (ABG) results: pH 7.40, PaO2 74 mm Hg, SaO2 92%, PaCO2 40 mm Hg. What is the most appropriate action by the nurse? a. Document the results in the patient's record. b. Repeat the ABGs within an hour to validate the findings. c. Encourage deep breathing and coughing to open the alveoli. d. Initiate pulse oximetry for continuous monitoring of the patient's oxygen status.

6. a. Normal findings in arterial blood gases (ABGs) in the older adult include a small decrease in PaO2 and SaO2 but normal pH and PaCO2. No interventions are necessary for these findings. Usual PaO2 levels are expected in patients 60 years of age or younger.

6. Which patient with the following manifestations is most likely to develop hypercapnic respiratory failure? a. Rapid, deep respirations in response to pneumonia b. Slow, shallow respirations as a result of sedative overdose c. Large airway resistance as a result of severe bronchospasm d. Poorly ventilated areas of the lung caused by pulmonary edema

6. b. Hypercapnic respiratory failure is associated with alveolar hypoventilation with increases in alveolar and arterial carbon dioxide (CO2) and often is caused by problems outside the lungs. A patient with slow, shallow respirations is not exchanging enough gas volume to eliminate CO2. Deep, rapid respirations reflect hyperventilation and often accompany lung problems that cause hypoxemic respiratory failure. Pulmonary edema and large airway resistance cause obstruction of oxygenation and result in a V/Q mismatch or shunt typical of hypoxemic respiratory failure.

6. Priority Decision: Which medication should the nurse anticipate being used first in the emergency department for relief of severe respiratory distress related to asthma? a. Prednisone orally b. Ipratopium inhaler c. Fluticasone inhaler d. Albuterol nebulizer

6. d. The albuterol nebulizer will rapidly cause bronchodilation and be easier to use in an emergency situation than an inhaler. It will be used every 20 minutes to 4 hours as needed. The ipratropium inhaler could be used if the patient does not tolerate the short-acting β2-adrenergic agonists (SABA) but its onset is slower than albuterol. Inhaled or oral corticosteroids will be used to decrease the inflammation and provide better symptom control after the emergency situation is over.

7. Which arterial blood gas (ABG) results would most likely indicate acute respiratory failure in a patient with chronic lung disease? a. PaO2 52 mm Hg, PaCO2 56 mm Hg, pH 7.4 c. PaO2 48 mm Hg, PaCO2 54 mm Hg, pH 7.38 b. PaO2 46 mm Hg, PaCO2 52 mm Hg, pH 7.36 d. PaO2 50 mm Hg, PaCO2 54 mm Hg, pH 7.28

7. d. In a patient with normal lung function, respiratory failure is commonly defined as a PaO2 ≤60 mm Hg or a PaCO2 >45 mm Hg or both. However, because the patient with chronic pulmonary disease normally maintains low PaO2 and high PaCO2, acute respiratory failure in these patients can be defined as an acute decrease in PaO2 or an increase in PaCO2 from the patient's baseline parameters, accompanied by an acidic pH. The pH of 7.28 reflects an acidemia and a loss of compensation in the patient with chronic lung disease.

7. A patient's ABGs include a PaO2 of 88 mm Hg and a PaCO2 of 38 mm Hg, and mixed venous blood gases include a PvO2 of 40 mm Hg and PvCO2 of 46 mm Hg. What do these findings indicate? a. Impaired cardiac output b. Unstable hemodynamics c. Inadequate delivery of oxygen to the tissues d. Normal capillary oxygen-carbon dioxide exchange

7. d. Normal venous blood gas values reflect the normal uptake of oxygen from arterial blood and the release of carbon dioxide from cells into the blood, resulting in a much lower PaO2 and an increased PaCO2. The pH is also decreased in mixed venous blood gases because of the higher PvCO2. Normal mixed venous blood gases also have much lower PvO2 and SvO2 than ABGs. Mixed venous blood gases are used when patients are hemodynamically unstable to evaluate the amount of oxygen delivered to the tissue and the amount of oxygen consumed by the tissues.

7. What is the initial antibiotic treatment for pneumonia based on? a. The severity of symptoms b. The presence of characteristic leukocytes c. Gram stains and cultures of sputum specimens d. History and physical examination and characteristic chest x-ray findings

7. d. Prompt treatment of pneumonia with appropriate antibiotics is important in treating bacterial and mycoplasma pneumonia and antibiotics are often administered on the basis of the history, physical examination, and a chest x-ray indicating a typical pattern characteristic of a particular organism without further testing. Sputum and blood cultures take 24 to 72 hours for results and microorganisms often cannot be identified with either Gram stains or cultures. Whether the pneumonia is community acquired or medical-care associated is more significant than the severity of symptoms.

8. The patient is being admitted to the intensive care unit (ICU) with hypercapnic respiratory failure. Which manifestations should the nurse expect to assess in the patient (select all that apply)? a. Cyanosis b. Metabolic acidosis c. Morning headache d. Respiratory acidosis e. Use of tripod position f. Rapid, shallow respirations

8. c, d, e, f. Morning headache, respiratory acidosis, the use of tripod position, and rapid, shallow respirations would be expected. The other manifestations are characteristic of hypoxemic respiratory failure.

8. Priority Decision: After the health care provider sees a patient hospitalized with a stroke who developed a fever and adventitious lung sounds, the following orders are written. Which order should the nurse implement first? a. Anterior/posterior and lateral chest x-rays b. Start IV levofloxacin (Levaquin) 500 mg every 24 hr c. Sputum specimen for Gram stain and culture and sensitivity d. Complete blood count (CBC) with white blood cell (WBC) count and differential

8. c. A sputum specimen for Gram stain and culture should be done before initiating antibiotic therapy and while waiting for the antibiotic to be delivered from the pharmacy in a hospitalized patient with suspected pneumonia and then antibiotics should be started without delay. If the sputum specimen cannot be obtained rapidly, the chest x-ray will be done to assess the typical pattern characteristic of the infecting organism. Blood cell tests will not be altered significantly by delaying the tests until after the first dose of antibiotics.

8. Priority Decision: A pulse oximetry monitor indicates that the patient has a drop in SpO2 from 95% to 85% over several hours. What is the first action the nurse should take? a. Order stat ABGs to confirm the SpO2 with a SaO2. b. Start oxygen administration by nasal cannula at 2 L/min. c. Check the position of the probe on the finger or earlobe. d. Notify the health care provider of the change in baseline PaO2.

8. c. Pulse oximetry is inaccurate if the probe is loose, if there is low perfusion, or when skin color is dark. Before other measures are taken, the nurse should check the probe site. If the probe is intact at the site and perfusion is adequate, an ABG analysis will be ordered by the health care provider to verify accuracy and oxygen may be administered, depending on the patient's condition and the assessment of respiratory and cardiac status.

9. Which assessment finding should cause the nurse to suspect the early onset of hypoxemia? a. Restlessness c. Central cyanosis b. Hypotension d. Cardiac dysrhythmias

9. a. Because the brain is very sensitive to a decrease in oxygen delivery, restlessness, agitation, disorientation, and confusion are early signs of hypoxemia, for which the nurse should be alert. Mild hypertension is also an early sign, accompanied by tachycardia. Central cyanosis is an unreliable, late sign of hypoxemia. Cardiac dysrhythmias also occur later.

9. Pulse oximetry may not be a reliable indicator of oxygen saturation in which patient? a. Patient with a fever c. Patient in hypovolemic shock b. Patient who is anesthetized d. Patient receiving oxygen therapy

9. c. Poor peripheral perfusion that occurs with hypovolemia or other types of conditions that cause peripheral vasoconstriction will cause inaccurate pulse oximetry and ABGs may need to be used to monitor oxygenation status and ventilation status in these patients. Pulse oximetry would not be affected by fever or anesthesia and is a method of monitoring arterial oxygen saturation in patients who are receiving oxygen therapy.

7. Which of the following signs or symptoms would be most consistent with the following values? pH, 7.20; pO2, 106 mm Hg; pCO2, 35 mm Hg; HCO3-, 11 mEq/L. a. Diarrhea c. Central cyanosis b. Shortness of breath d. Peripheral cyanosis

ANS: A Diarrhea is one mechanism by which the body can lose large amounts of HCO3-. The other choices are indications of hypoxia, which is not indicated with a pO2 of 106 mm Hg.

1. A patient with asthma has a personal best peak expiratory flow rate (PEFR) of 400 L/minute. When explaining the asthma action plan, the nurse will teach the patient that a change in therapy is needed when the PEFR is less than ___ L/minute

ANS: 320 A PEFR less than 80% of the personal best indicates that the patient is in the yellow zone where changes in therapy are needed to prevent progression of the airway narrowing.

1. The nurse notes new onset confusion in an older patient who is normally alert and oriented. In which order should the nurse take the following actions? (Put a comma and a space between each answer choice [A, B, C, D].) a. Obtain the oxygen saturation. b. Check the patient's pulse rate. c. Document the change in status. d. Notify the health care provider.

ANS: A, B, D, C Assessment for physiologic causes of new onset confusion such as pneumonia, infection, or perfusion problems should be the first action by the nurse. Airway and oxygenation should be assessed first, then circulation. After assessing the patient, the nurse should notify the health care provider. Finally, documentation of the assessments and care should be done.

1. When assisting with oral intubation of a patient who is having respiratory distress, in which order will the nurse take these actions? (Put a comma and a space between each answer choice [A, B, C, D, E].) a. Obtain a portable chest-x-ray. b. Position the patient in the supine position. c. Inflate the cuff of the endotracheal tube after insertion. d. Attach an end-tidal CO2 detector to the endotracheal tube. e. Oxygenate the patient with a bag-valve-mask device for several minutes.

ANS: E, B, C, D, A The patient is pre-oxygenated with a bag-valve-mask system for 3 to 5 minutes before intubation and then placed in a supine position. Following the intubation, the cuff on the endotracheal tube is inflated to occlude and protect the airway. Tube placement is assessed first with an end-tidal CO2 sensor, then with a chest x-ray.

10. Which of the following levels would be classified as a low-flow system of oxygen administration? a. O2 via nasal cannula at 4 L/min b. O2 via nasal catheter at a FiO2 range of 60% to 75% c. O2 via transtracheal catheter at 10 L/min d. O2 via simple mask at 12 L/min.

ANS: A A low-flow oxygen delivery system provides supplemental oxygen directly into the patient's airway at a flow of 8 L/min or less. Because this flow is insufficient to meet the patient's inspiratory volume requirements, it results in a variable FiO2 as the supplemental oxygen is mixed with room air. Nasal catheter FiO2 range is 22% to 45%. Oxygen flow through a transtracheal catheter is 0.25 to 4 L/min. A simple mask is a reservoir delivery system.

29. Under normal situations, the normal ventilation to perfusion ratio is 4:5 (0.8). Atelectasis can cause a shunt-producing ventilation/perfusion mismatch. This occurs primarily as a result of a. perfusion exceeding ventilation. b. ventilation exceeding perfusion. c. an alveolus that is receiving ventilation but not perfusion. d. an alveolus that is receiving perfusion but not ventilation.

ANS: A A shunt-producing ventilation/perfusion mismatch is one in which perfusion exceeds ventilation. Whereas situations in which ventilation exceeds perfusion V/Q greater than 0.8 are considered to be dead space producing, situations in which perfusion exceeds ventilation V/Q less than 0.8 are considered to be shunt producing.

12. Which of the following conditions develops when air enters the pleural space from the lung on inhalation and cannot exit on exhalation? a. Tension pneumothorax c. Open pneumothorax b. Sucking chest wound d. Pulmonary interstitial empyema

ANS: A A tension pneumothorax develops when air enters the pleural space from either the lung or the chest wall on inhalation and cannot escape on exhalation. Open pneumothorax is a laceration in the parietal pleura that allows atmospheric air to enter the pleural space; it occurs as a result of penetrating chest trauma. Pulmonary interstitial emphysema is air in the pulmonary interstitial space.

25. A patient in the clinic with cystic fibrosis (CF) reports increased sweating and weakness during the summer months. Which action by the nurse would be most appropriate? a. Have the patient add dietary salt to meals. b. Teach the patient about the signs of hypoglycemia. c. Suggest decreasing intake of dietary fat and calories. d. Instruct the patient about pancreatic enzyme replacements.

ANS: A Added dietary salt is indicated whenever sweating is excessive, such as during hot weather, when fever is present, or from intense physical activity. The management of pancreatic insufficiency includes pancreatic enzyme replacement of lipase, protease, and amylase (e.g., Pancreaze, Creon, Ultresa, Zenpep) administered before each meal and snack. This patient is at risk for hyponatremia based on reported symptoms. Adequate intake of fat, calories, protein, and vitamins is important. Fat-soluble vitamins (vitamins A, D, E, and K) must be supplemented because they are malabsorbed. Use of caloric supplements improves nutritional status. Hyperglycemia due to pancreatic insufficiency is more likely to occur than hypoglycemia.

41. The nurse reviews the medication administration record (MAR) for a patient having an acute asthma attack. Which medication should the nurse administer first? a. Albuterol (Ventolin) 2.5 mg per nebulizer b. Methylprednisolone (Solu-Medrol) 60 mg IV c. Salmeterol (Serevent) 50 mcg per dry-powder inhaler (DPI) d. Triamcinolone (Azmacort) 2 puffs per metered-dose inhaler (MDI)

ANS: A Albuterol is a rapidly acting bronchodilator and is the first-line medication to reverse airway narrowing in acute asthma attacks. The other medications work more slowly.

21. Which route for ETT placement is used in an emergency? a. Orotracheal c. Nasopharyngeal b. Nasotracheal d. Trachea

ANS: A An endotracheal tube (ETT) may be placed through the orotracheal or the nasotracheal route. In most situations involving emergency placement, the orotracheal route is used because it is simpler and allows the use of a larger diameter ETT. Nasotracheal intubation provides greater patient comfort over time and is preferred in patients with a jaw fracture.

36. A patient who is experiencing an acute asthma attack is admitted to the emergency department. Which assessment should the nurse complete first? a. Listen to the patient's breath sounds. b. Ask about inhaled corticosteroid use. c. Determine when the dyspnea started. d. Obtain the forced expiratory volume (FEV) flow rate.

ANS: A Assessment of the patient's breath sounds will help determine how effectively the patient is ventilating and whether rapid intubation may be necessary. The length of time the attack has persisted is not as important as determining the patient's status at present. Most patients having an acute attack will be unable to cooperate with an FEV measurement. It is important to know about the medications the patient is using but not as important as assessing the breath sounds.

14. A patient with acute dyspnea is scheduled for a spiral computed tomography (CT) scan. Which information obtained by the nurse is a priority to communicate to the health care provider before the CT? a. Allergy to shellfish b. Apical pulse of 104 c. Respiratory rate of 30 d. Oxygen saturation of 90%

ANS: A Because iodine-based contrast media is used during a spiral CT, the patient may need to have the CT scan without contrast or be premedicated before injection of the contrast media. The increased pulse, low oxygen saturation, and tachypnea all indicate a need for further assessment or intervention but do not indicate a need to modify the CT procedure.

23. Which information about a patient who is receiving cisatracurium (Nimbex) to prevent asynchronous breathing with the positive pressure ventilator requires immediate action by the nurse? a. Only continuous IV opioids have been ordered. b. The patient does not respond to verbal stimulation. c. There is no cough or gag when the patient is suctioned. d. The patient's oxygen saturation fluctuates between 90% to 93%.

ANS: A Because neuromuscular blockade is extremely anxiety provoking, it is essential that patients who are receiving neuromuscular blockade receive concurrent sedation and analgesia. Absence of response to stimuli is expected in patients receiving neuromuscular blockade. The oxygen saturation is adequate.

17. The clinic nurse teaches a patient with a 42 pack-year history of cigarette smoking about lung disease. Which information will be most important for the nurse to include? a. Options for smoking cessation b. Reasons for annual sputum cytology testing c. Erlotinib (Tarceva) therapy to prevent tumor risk d. Computed tomography (CT) screening for lung cancer

ANS: A Because smoking is the major cause of lung cancer, the most important role for the nurse is teaching patients about the benefits of and means of smoking cessation. CT scanning is currently being investigated as a screening test for high-risk patients. However, if there is a positive finding, the person already has lung cancer. Erlotinib may be used in patients who have lung cancer, but it is not used to reduce the risk of developing cancer.

15. The nurse is caring for a patient with cor pulmonale. The nurse should monitor the patient for which expected finding? a. Peripheral edema b. Elevated temperature c. Clubbing of the fingers d. Complaints of chest pain

ANS: A Cor pulmonale causes clinical manifestations of right ventricular failure, such as peripheral edema. The other clinical manifestations may occur in the patient with other complications of chronic obstructive pulmonary disease (COPD) but are not indicators of cor pulmonale.

26. The nurse administers prescribed therapies for a patient with cor pulmonale and right-sided heart failure. Which assessment would best evaluate the effectiveness of the therapies? a. Observe for distended neck veins. b. Auscultate for crackles in the lungs. c. Palpate for heaves or thrills over the heart. d. Review hemoglobin and hematocrit values.

ANS: A Cor pulmonale is right ventricular failure caused by pulmonary hypertension, so clinical manifestations of right ventricular failure such as peripheral edema, jugular venous distention, and right upper-quadrant abdominal tenderness would be expected. Crackles in the lungs are likely to be heard with left-sided heart failure. Findings in cor pulmonale include evidence of right ventricular hypertrophy on electrocardiogram ECG and an increase in intensity of the second heart sound. Heaves or thrills are not common with cor pulmonale. Chronic hypoxemia leads to polycythemia and increased total blood volume and viscosity of the blood. The hemoglobin and hematocrit values are more likely to be elevated with cor pulmonale than decreased.

3. A patient with bacterial pneumonia has rhonchi and thick sputum. What is the nurse's most appropriate action to promote airway clearance? a. Assist the patient to splint the chest when coughing. b. Teach the patient about the need for fluid restrictions. c. Encourage the patient to wear the nasal oxygen cannula. d. Instruct the patient on the pursed lip breathing technique.

ANS: A Coughing is less painful and more likely to be effective when the patient splints the chest during coughing. Fluids should be encouraged to help liquefy secretions. Nasal oxygen will improve gas exchange, but will not improve airway clearance. Pursed lip breathing is used to improve gas exchange in patients with COPD, but will not improve airway clearance.

4. On auscultation of a patient's lungs, the nurse hears low-pitched, bubbling sounds during inhalation in the lower third of both lungs. How should the nurse document this finding? a. Inspiratory crackles at the bases b. Expiratory wheezes in both lungs c. Abnormal lung sounds in the apices of both lungs d. Pleural friction rub in the right and left lower lobes

ANS: A Crackles are low-pitched, bubbling sounds usually heard on inspiration. Wheezes are high-pitched sounds. They can be heard during the expiratory or inspiratory phase of the respiratory cycle. The lower third of both lungs are the bases, not apices. Pleural friction rubs are grating sounds that are usually heard during both inspiration and expiration.

11. Diminished to absent breath sounds on the right side, tracheal deviation to the left side, and asymmetrical chest movement are indicative of which of the following disorders? a. Tension pneumothorax c. Pulmonary fibrosis b. Pneumonia d. Atelectasis

ANS: A Diminished to absent breath sounds on the right side, tracheal deviation to the left side, and asymmetrical chest movement are indicative of tension pneumothorax.

13. A nurse is caring for a patient who is orally intubated and receiving mechanical ventilation. To decrease the risk for ventilator-associated pneumonia, which action will the nurse include in the plan of care? a. Elevate head of bed to 30 to 45 degrees. b. Suction the endotracheal tube every 2 to 4 hours. c. Limit the use of positive end-expiratory pressure. d. Give enteral feedings at no more than 10 mL/hr.

ANS: A Elevation of the head decreases the risk for aspiration. Positive end-expiratory pressure is frequently needed to improve oxygenation in patients receiving mechanical ventilation. Suctioning should be done only when the patient assessment indicates that it is necessary. Enteral feedings should provide adequate calories for the patient's high energy needs.

43. Which finding in a patient hospitalized with bronchiectasis is most important to report to the health care provider? a. Cough productive of bloody, purulent mucus b. Scattered rhonchi and wheezes heard bilaterally c. Respiratory rate 28 breaths/minute while ambulating in hallway d. Complaint of sharp chest pain with deep breathing

ANS: A Hemoptysis may indicate life-threatening hemorrhage and should be reported immediately to the health care provider. The other findings are frequently noted in patients with bronchiectasis and may need further assessment but are not indicators of life-threatening complications.

2. The nurse assesses the chest of a patient with pneumococcal pneumonia. Which finding would the nurse expect? a. Increased tactile fremitus b. Dry, nonproductive cough c. Hyperresonance to percussion d. A grating sound on auscultation

ANS: A Increased tactile fremitus over the area of pulmonary consolidation is expected with bacterial pneumonias. Dullness to percussion would be expected. Pneumococcal pneumonia typically presents with a loose, productive cough. Adventitious breath sounds such as crackles and wheezes are typical. A grating sound is more representative of a pleural friction rub rather than pneumonia.

2. While caring for a patient who has been admitted with a pulmonary embolism, the nurse notes a change in the patient's oxygen saturation (SpO2) from 94% to 88%. Which action should the nurse take next? a. Increase the oxygen flow rate. b. Suction the patient's oropharynx. c. Instruct the patient to cough and deep breathe. d. Help the patient to sit in a more upright position

ANS: A Increasing oxygen flow rate will usually improve oxygen saturation in patients with ventilation-perfusion mismatch, as occurs with pulmonary embolism. Because the problem is with perfusion, actions that improve ventilation, such as deep breathing and coughing, sitting upright, and suctioning, are not likely to improve oxygenation.

6. When admitting a patient with possible respiratory failure with a high PaCO2, which assessment information should be immediately reported to the health care provider? a. The patient is somnolent. b. The patient complains of weakness. c. The patient's blood pressure is 164/98. d. The patient's oxygen saturation is 90%.

ANS: A Increasing somnolence will decrease the patient's respiratory rate and further increase the PaCO2 and respiratory failure. Rapid action is needed to prevent respiratory arrest. An SpO2 of 90%, weakness, and elevated blood pressure all require ongoing monitoring but are not indicators of possible impending respiratory arrest.

21. The use of observation for assessment is known as a. inspection. c. percussion. b. palpation. d. auscultation.

ANS: A Inspection is the process of looking intently at the patient. Palpation is the process of touching the patient to judge the size, shape, texture, and temperature of the body surface or underlying structures. Percussion is the process of creating sound waves on the surface of the body to determine abnormal density of any underlying areas. Auscultation is the process of concentrated listening with a stethoscope to determine characteristics of body functions.

9. Which of the following ranges would be considered normal pulmonary artery systolic pressures? a. 15 to 30 mm Hg c. 25 to 35 mm Hg b. 4 to 12 mm Hg d. 1 to 11 mm Hg

ANS: A Pulmonary artery systolic pressure ranges from 15 to 30 mm Hg, pulmonary artery diastolic pressure ranges from 4 to 12 mm Hg, and pulmonary artery mean pressure ranges from 9 to 18 mm Hg. Pulmonary hypertensions is defined as pulmonary artery systolic pressure of greater than 35 mm Hg.

17. A patient was admitted to the critical care unit with acute respiratory failure. The patient has been on a ventilator for 3 days and is being considered for weaning. The ventilator high-pressure alarm keeps going off. When you enter the room, the ventilator inoperative alarm sounds. All of the following conditions would set off the high-pressure alarm except a. a leak in the patient's ET tube cuff b. a kink in the ET tubing c. coughing d. increased secretions in the patient's airway

ANS: A Low inspiratory pressure alarms will sound because of altered settings, unattached tubing or a leak around the endotracheal tube (ETT), the ETT displaced into the pharynx or esophagus, poor cuff inflation or leak, tracheoesophageal fistula, peak flows that are too low, low tidal volume (Vt), decreased airway resistance resulting from decreased secretions or relief of bronchospasm, increased lung compliance resulting from decreased atelectasis, reduction in pulmonary edema, resolution of ARDS, or a change in position. High-pressure alarms will sound because of improper alarm setting; airway obstruction resulting from patient fighting ventilator (holding breath as ventilator delivers Vt); patient circuit collapse; kinked tubing; the ETT in the right mainstem bronchus or against the carina; cuff herniation; increased airway resistance resulting from bronchospasm, airway secretions, plugs, and coughing; water from the humidifier in the ventilator tubing; and decreased lung compliance resulting from tension pneumothorax, change in patient position, acute respiratory distress syndrome, pulmonary edema, atelectasis, pneumonia, or abdominal distention.

31. Methemoglobin causes difficulty with oxygenation because a. methemoglobin does not carry oxygen as normal hemoglobin would. b. carbon monoxide has a greater affinity for the hemoglobin than oxygen. c. only 80% of the total oxygen is carried on the hemoglobin. d. the anemia causes decreased numbers of hemoglobin receptor sites.

ANS: A Methemoglobin occurs when the iron atoms within the hemoglobin molecule are oxidized from the ferrous state to the ferric state. Methemoglobin does not carry oxygen. The most common abnormality involving hemoglobin is a decrease in amount. This can be an acute or a chronic situation (anemia). Abnormal hemoglobin structure also can pose problems, such as hemoglobin S, which is responsible for sickle cell anemia. Hemoglobin carries approximately 97% of the total amount of oxygen held within the bloodstream.

36. A patient who is receiving positive pressure ventilation is scheduled for a spontaneous breathing trial (SBT). Which finding by the nurse is most important to discuss with the health care provider before starting the SBT? a. New ST segment elevation is noted on the cardiac monitor. b. Enteral feedings are being given through an orogastric tube. c. Scattered rhonchi are heard when auscultating breath sounds. d. HYDROmorphone (Dilaudid) is being used to treat postoperative pain.

ANS: A Myocardial ischemia is a contraindication for ventilator weaning. The ST segment elevation is an indication that weaning should be postponed until further investigation and/or treatment for myocardial ischemia can be done. The other information will also be shared with the health care provider, but ventilator weaning can proceed when opioids are used for pain management, abnormal lung sounds are present, or enteral feedings are being used.

11. An older patient is receiving standard multidrug therapy for tuberculosis (TB). The nurse should notify the health care provider if the patient exhibits which finding? a. Yellow-tinged skin b. Orange-colored sputum c. Thickening of the fingernails d. Difficulty hearing high-pitched voices

ANS: A Noninfectious hepatitis is a toxic effect of isoniazid (INH), rifampin, and pyrazinamide, and patients who develop hepatotoxicity will need to use other medications. Changes in hearing and nail thickening are not expected with the four medications used for initial TB drug therapy. Presbycusis is an expected finding in the older adult patient. Orange discoloration of body fluids is an expected side effect of rifampin and not an indication to call the health care provider.

17. A patient was admitted to the critical care unit after a left pneumonectomy. The patient is receiving 40% oxygen via a simple facemask. The morning chest radiography study reveals right lower lobe pneumonia. After eating breakfast, the patient suddenly vomits and aspirates. The patient becomes agitated, has decreased level of consciousness, and has an inability to maintain saturation. The nurse expects the next action will include a. placing the patient on a mechanical ventilator. b. change in antibiotics to control infection. c. suctioning and repositioning. d. administering a sedative to control anxiety.

ANS: A Nursing interventions include optimizing oxygenation and ventilation, preventing the spread of infection, providing comfort and emotional support, and maintaining surveillance for complications.

31. The nurse cares for a patient who has just had a thoracentesis. Which assessment information obtained by the nurse is a priority to communicate to the health care provider? a. Oxygen saturation is 88%. b. Blood pressure is 145/90 mm Hg. c. Respiratory rate is 22 breaths/minute when lying flat. d. Pain level is 5 (on 0 to 10 scale) with a deep breath.

ANS: A Oxygen saturation would be expected to improve after a thoracentesis. A saturation of 88% indicates that a complication such as pneumothorax may be occurring. The other assessment data also indicate a need for ongoing assessment or intervention, but the low oxygen saturation is the priority.

22. A patient experiences a chest wall contusion as a result of being struck in the chest with a baseball bat. The emergency department nurse would be most concerned if which finding is observed during the initial assessment? a. Paradoxic chest movement b. Complaint of chest wall pain c. Heart rate of 110 beats/minute d. Large bruised area on the chest

ANS: A Paradoxic chest movement indicates that the patient may have flail chest, which can severely compromise gas exchange and can rapidly lead to hypoxemia. Chest wall pain, a slightly elevated pulse rate, and chest bruising all require further assessment or intervention, but the priority concern is poor gas exchange.

4. Which of the following nursing interventions should be used to optimize oxygenation and ventilation in the patient with acute respiratory failure? a. Provide adequate rest and recovery time between procedures. b. Position the patient with the good lung up. c. Suction the patient every hour. d. Avoid hyperventilating the patient.

ANS: A Providing adequate rest and recovery time between various procedures prevents desaturation and optimizes oxygenation. In acute lung failure, the goal of positioning is to place the least affected area of the patient's lung in the most dependent position. Patients with unilateral lung disease should be positioned with the healthy lung in a dependent position. Hyperventilate the patient before suctioning; suction patients as needed.

22. Severe coughing and shortness of breath during a thoracentesis are indicative of which of the following complications? a. Re-expansion pulmonary edema c. Pneumothorax b. Pleural infection d. Hemothorax

ANS: A Re-expansion pulmonary edema can occur when a large amount of effusion fluid (~1000-1500 mL) is removed from the pleural space. Removal of the fluid increases the negative intrapleural pressure, which can lead to edema when the lung does not re-expand to fill the space. The patient experiences severe coughing and shortness of breath. The onset of these symptoms is an indication to discontinue the thoracentesis.

1. Why would the nurse perform an inspection of the oral cavity during a complete pulmonary assessment? a. To provide evidence of hypoxia b. To provide evidence of dyspnea c. To provide evidence of dehydration d. To provide evidence of nutritional status

ANS: A Severe hypoxia will be manifested by central cyanosis, which is evident in the oral and circumoral areas. Although dehydration and nutritional status can both be partially assessed by oral cavity inspection; this information is not as vital as determining hypoxia. Dyspnea means difficulty breathing.

12. A patient is admitted to the emergency department complaining of sudden onset shortness of breath and is diagnosed with a possible pulmonary embolus. How should the nurse prepare the patient for diagnostic testing to confirm the diagnosis? a. Start an IV so contrast media may be given. b. Ensure that the patient has been NPO for at least 6 hours. c. Inform radiology that radioactive glucose preparation is needed. d. Instruct the patient to undress to the waist and remove any metal objects.

ANS: A Spiral computed tomography (CT) scans are the most commonly used test to diagnose pulmonary emboli, and contrast media may be given IV. A chest x-ray may be ordered but will not be diagnostic for a pulmonary embolus. Preparation for a chest x-ray includes undressing and removing any metal. Bronchoscopy is used to detect changes in the bronchial tree, not to assess for vascular changes, and the patient should be NPO 6 to 12 hours before the procedure. Positron emission tomography (PET) scans are most useful in determining the presence of malignancy, and a radioactive glucose preparation is used.

23. A static lung compliance of 40 mL/cm H2O is indicative of which of the following disorders? a. Pneumonia c. Pulmonary emboli b. Bronchospasm d. Upper airway obstruction

ANS: A Static compliance is measured under no-flow conditions so that resistance forces are removed. Static compliance decreases with any decrease in lung compliance, such as occurs with pneumothorax, atelectasis, pneumonia, pulmonary edema, and chest wall restrictions. A normal value is 57 to 85 mL/cm of H2O.

27. A patient is admitted with acute respiratory failure attributable to pneumonia. Smoking history reveals that the patient smoked two packs of cigarettes a day for 25 years, stopping 10 years ago. ABG values on the current ventilator settings are pH, 7.37; PaCO2, 50 mm Hg; and HCO3-, 27 mEq/L. Chest radiography reveals a large right pleural effusion. Which of the following is the correct interpretation of the patient's ABG values? a. Compensated respiratory acidosis c. Uncompensated respiratory alkalosis b. Compensated metabolic alkalosis d. Uncompensated metabolic acidosis

ANS: A The ABG values reflect a compensated respiratory acidosis. Values include a pH of 7.35 to 7.39, PACO2 above 45 mm Hg, and HCO3- above 26 mEq/L. Uncompensated respiratory alkalosis values include a pH below 7.35, PACO2 above 45 mm Hg, and HCO3- of 22 to 26 mEq/L. Compensated metabolic alkalosis values include a pH of 7.41 to 7.45, PACO2 above 45 mm Hg, and HCO3- above 26 mEq/L. Uncompensated metabolic acidosis values include a pH above 7.35, PACO2 of 35 to 45 mm Hg, and HCO3- above 22 mEq/L.

22. A nurse is caring for a patient with acute respiratory distress syndrome (ARDS) who is receiving mechanical ventilation using synchronized intermittent mandatory ventilation (SIMV). The settings include fraction of inspired oxygen (FIO2) 80%, tidal volume 450, rate 16/minute, and positive end-expiratory pressure (PEEP) 5 cm. Which assessment finding is most important for the nurse to report to the health care provider? a. Oxygen saturation 99% b. Respiratory rate 22 breaths/minute c. Crackles audible at lung bases d. Heart rate 106 beats/minute

ANS: A The FIO2 of 80% increases the risk for oxygen toxicity. Because the patient's O2 saturation is 99%, a decrease in FIO2 is indicated to avoid toxicity. The other patient data would be typical for a patient with ARDS and would not need to be urgently reported to the health care provider.

21. The most important function of type I alveolar epithelial cells is a. that they comprise 90% of total alveolar surface in the lungs for gas exchange. b. the ability to produce, store, and secrete pulmonary surfactant. c. the ability to trap foreign particles for autodigestion. d. the maintenance, repair, and restoration of the mucociliary escalator.

ANS: A Type I alveolar epithelial cells comprise approximately 90% of the total alveolar surface within the lungs. The most important function of the type II cells is their ability to produce, store, and secrete pulmonary surfactant.

19. The nurse is caring for a 33-year-old patient who arrived in the emergency department with acute respiratory distress. Which assessment finding by the nurse requires the most rapid action? a. The patient's PaO2 is 45 mm Hg. b. The patient's PaCO2 is 33 mm Hg. c. The patient's respirations are shallow. d. The patient's respiratory rate is 32 breaths/minute.

ANS: A The PaO2 indicates severe hypoxemia and respiratory failure. Rapid action is needed to prevent further deterioration of the patient. Although the shallow breathing, rapid respiratory rate, and low PaCO2 also need to be addressed, the most urgent problem is the patient's poor oxygenation.

6. A patient has sustained a stroke. The speech therapist tells you that the patient cannot control his epiglottis. You know that this patient a. is at increased risk of aspiration. c. will need a tracheostomy to breathe. b. will need surgery to close his epiglottis. d. is at risk for a pneumothorax.

ANS: A The epiglottis is responsible for closing over the trachea and preventing entry of swallowed material into the lungs. An inability to control the epiglottis increases the risk of aspiration and may warrant placement of a feeding tube. The patient will still be able to breathe. Closure of the epiglottis over the trachea will occlude the airway.

6. The finding of normal breath sounds on the right side of the chest and absent breath sounds on the left side of the chest in a newly intubated patient is probably caused by a a. right mainstem intubation. c. right hemothorax. b. left pneumothorax. d. gastric intubation.

ANS: A The finding of normal breath sounds on the right side of the chest and absent breath sounds on the left side of the chest in a newly intubated patient is probably caused by a right mainstem intubation.

22. A patient with respiratory failure has arterial pressure-based cardiac output (APCO) monitoring and is receiving mechanical ventilation with peak end-expiratory pressure (PEEP) of 12 cm H2O. Which information indicates that a change in the ventilator settings may be required? a. The arterial pressure is 90/46. b. The heart rate is 58 beats/minute. c. The stroke volume is increased. d. The stroke volume variation is 12%.

ANS: A The hypotension suggests that the high intrathoracic pressure caused by the PEEP may be decreasing venous return and (potentially) cardiac output. The other assessment data would not be a direct result of PEEP and mechanical ventilation.

20. The nurse assesses a patient with chronic obstructive pulmonary disease (COPD) who has been admitted with increasing dyspnea over the last 3 days. Which finding is most important for the nurse to report to the health care provider? a. Respirations are 36 breaths/minute. b. Anterior-posterior chest ratio is 1:1. c. Lung expansion is decreased bilaterally. d. Hyperresonance to percussion is present.

ANS: A The increase in respiratory rate indicates respiratory distress and a need for rapid interventions such as administration of oxygen or medications. The other findings are common chronic changes occurring in patients with COPD.

33. A patient who is orally intubated and receiving mechanical ventilation is anxious and is "fighting" the ventilator. Which action should the nurse take next? a. Verbally coach the patient to breathe with the ventilator. b. Sedate the patient with the ordered PRN lorazepam (Ativan). c. Manually ventilate the patient with a bag-valve-mask device. d. Increase the rate for the ordered propofol (Diprivan) infusion.

ANS: A The initial response by the nurse should be to try to decrease the patient's anxiety by coaching the patient about how to coordinate respirations with the ventilator. The other actions may also be helpful if the verbal coaching is ineffective in reducing the patient's anxiety.

10. The oxygen saturation of a healthy individual rarely reaches 100% on room air. This can best be explained by which of the following concepts? a. Physiologic shunting c. Collateral air passages b. Alveolar capillary diffusion d. Anatomic dead space

ANS: A The mixing of venous blood from the bronchial circulation with the oxygenated blood in the left atrium decreases the saturation of left atrial blood to a range between 96% and 99%. For this reason, while a person is breathing room air, the oxygen saturation of arterial blood is less than 100%.

4. A patient's assessment data present as follows: pH, 7.10; PaCO2, 60 mm Hg; PaO2, 40 mm Hg; HCO3-, 24 mEq/L; RR, 34 breaths/min; HR, 128 beats/min; and BP, 180/92 mm Hg. This condition is best described as a. uncompensated respiratory acidosis. b. uncompensated metabolic acidosis. c. compensated metabolic acidosis. d. compensated respiratory acidosis.

ANS: A The pH is below normal range (7.35-7.45), so this is uncompensated acidosis. The PaCO2 is markedly elevated, and the HCO3- is normal. This indicates uncompensated respiratory acidosis. Uncompensated respiratory acidosis values include a pH below 7.35, PACO2 above 45 mm Hg, and HCO3- of 22 to 26 mEq/L. Uncompensated metabolic acidosis values include a pH below 7.35, PACO2 of 35 to 45 mm Hg, and HCO3- above 22 mEq/L. Compensated metabolic acidosis values include a pH of 7.35 to 7.39, PACO2 below 35 mm Hg, and HCO3- below 22 mEq/L. Compensated respiratory acidosis values include a pH of 7.35 to 7.35, PACO2 above 45 mm Hg, and HCO3- above 26 mEq/L.

38. The nurse in the emergency department receives arterial blood gas results for four recently admitted patients with obstructive pulmonary disease. Which patient will require the most rapid action by the nurse? a. 22-year-old with ABG results: pH 7.28, PaCO2 60 mm Hg, and PaO2 58 mm Hg b. 34-year-old with ABG results: pH 7.48, PaCO2 30 mm Hg, and PaO2 65 mm Hg c. 45-year-old with ABG results: pH 7.34, PaCO2 33 mm Hg, and PaO2 80 mm Hg d. 65-year-old with ABG results: pH 7.31, PaCO2 58 mm Hg, and PaO2 64 mm Hg

ANS: A The pH, PaCO2, and PaO2 indicate that the patient has severe uncompensated respiratory acidosis and hypoxemia. Rapid action will be required to prevent increasing hypoxemia and correct the acidosis. The other patients also should be assessed as quickly as possible but do not require interventions as quickly as the 22-year-old.

5. A nurse is caring for an obese patient with right lower lobe pneumonia. Which position will be best to improve gas exchange? a. On the left side b. On the right side c. In the tripod position d. In the high-Fowler's position

ANS: A The patient should be positioned with the "good" lung in the dependent position to improve the match between ventilation and perfusion. The obese patient's abdomen will limit respiratory excursion when sitting in the high-Fowler's or tripod positions.

19. The most common presenting signs and symptoms associated with PEs are a. tachycardia and tachypnea. b. hemoptysis and evidence of deep vein thromboses. c. apprehension and dyspnea. d. right ventricular failure and fever

ANS: A The patient with a pulmonary embolism may have any number of presenting signs and symptoms, with the most common being tachycardia and tachypnea. Additional signs and symptoms that may be present include dyspnea, apprehension, increased pulmonic component of the second heart sound (P1), fever, crackles, pleuritic chest pain, cough, evidence of deep vein thrombosis, and hemoptysis. Syncope and hemodynamic instability can occur as a result of right ventricular failure.

35. A patient who is experiencing an asthma attack develops bradycardia and a decrease in wheezing. Which action should the nurse take first? a. Notify the health care provider. b. Document changes in respiratory status. c. Encourage the patient to cough and deep breathe. d. Administer IV methylprednisolone (Solu-Medrol).

ANS: A The patient's assessment indicates impending respiratory failure, and the nurse should prepare to assist with intubation and mechanical ventilation after notifying the health care provider. IV corticosteroids require several hours before having any effect on respiratory status. The patient will not be able to cough or deep breathe effectively. Documentation is not a priority at this time.

42. A patient who was admitted the previous day with pneumonia complains of a sharp pain of 7 (based on 0 to 10 scale) "whenever I take a deep breath." Which action will the nurse take next? a. Auscultate breath sounds. b. Administer the PRN morphine. c. Have the patient cough forcefully. d. Notify the patient's health care provider.

ANS: A The patient's statement indicates that pleurisy or a pleural effusion may have developed and the nurse will need to listen for a pleural friction rub and/or decreased breath sounds. Assessment should occur before administration of pain medications. The patient is unlikely to be able to cough forcefully until pain medication has been administered. The nurse will want to obtain more assessment data before calling the health care provider.

38. Trauma to which vertebrae will cause ventilation dysfunction? a. C3-C5 c. T4-T6 b. C5-T3 d. T7-T10

ANS: A The phrenic nerve arises from the cervical plexus through the fourth cervical nerve, with secondary contributions by the third and fifth cervical nerves. For this reason and because the diaphragm does most of the work of inhalation, trauma involving levels C3 to C5 causes ventilation dysfunction.

18. The primary functions of the pulmonary system are a. gas exchange and the movement of air in and out of the lungs. b. gas exchange and the transfer of oxygen to the tissues. c. the movement of blood in and out of the lungs and the removal of body waste products. d. gas exchange and the prevention of infections.

ANS: A The primary functions of the pulmonary system are ventilation and respiration. Ventilation is the movement of air in and out of the lungs. Respiration is the process of gas exchange, that is, the movement of oxygen from the atmosphere into the bloodstream and the movement of carbon dioxide from the bloodstream into the atmosphere.

11. A patient with acute respiratory distress syndrome (ARDS) is placed in the prone position. When prone positioning is used, which information obtained by the nurse indicates that the positioning is effective? a. The patient's PaO2 is 89 mm Hg, and the SaO2 is 91%. b. Endotracheal suctioning results in clear mucous return. c. Sputum and blood cultures show no growth after 48 hours. d. The skin on the patient's back is intact and without redness.

ANS: A The purpose of prone positioning is to improve the patient's oxygenation as indicated by the PaO2 and SaO2. The other information will be collected but does not indicate whether prone positioning has been effective.

1. Following assessment of a patient with pneumonia, the nurse identifies a nursing diagnosis of ineffective airway clearance. Which assessment data best supports this diagnosis? a. Weak, nonproductive cough effort b. Large amounts of greenish sputum c. Respiratory rate of 28 breaths/minute d. Resting pulse oximetry (SpO2) of 85%

ANS: A The weak, nonproductive cough indicates that the patient is unable to clear the airway effectively. The other data would be used to support diagnoses such as impaired gas exchange and ineffective breathing pattern.

11. V/Q scans are ordered to evaluate the possibility of which of the following? a. Pulmonary emboli c. Emphysema b. Acute myocardial infarction d. Acute respiratory distress syndrome

ANS: A This test is ordered for the evaluation of pulmonary emboli. Electrocardiography or cardiac enzymes are ordered to evaluate for myocardial infarction; arterial blood gas analysis, chest radiography, and pulmonary function tests are ordered to evaluate for emphysema. Chest radiography and hemodynamic monitoring are ordered for evaluation of acute respiratory distress syndrome.

9. A patient presents moderately short of breath and dyspneic. A chest radiographic examination reveals a large right pleural effusion with significant atelectasis. The physician or nurse practitioner would be most likely to order which of the following procedures? a. Thoracentesis c. Ventilation/perfusion (V/Q) scan b. Bronchoscopy d. Repeat chest radiograph

ANS: A Thoracentesis is a procedure that can be performed at the bedside for the removal of fluid or air from the pleural space. It is used most often as a diagnostic measure; it may also be performed therapeutically for the drainage of a pleural effusion or empyema. No evidence is present that would necessitate a V/Q scan. A bronchoscopy cannot assist in fluid removal. A problem with this chest radiograph is not indicated.

39. Which nursing action for a patient with chronic obstructive pulmonary disease (COPD) could the nurse delegate to experienced unlicensed assistive personnel (UAP)? a. Obtain oxygen saturation using pulse oximetry. b. Monitor for increased oxygen need with exercise. c. Teach the patient about safe use of oxygen at home. d. Adjust oxygen to keep saturation in prescribed parameters.

ANS: A UAP can obtain oxygen saturation (after being trained and evaluated in the skill). The other actions require more education and a scope of practice that licensed practical/vocational nurses (LPN/LVNs) or registered nurses (RNs) would have.

48. The nurse is caring for a patient who has a right-sided chest tube after a right lower lobectomy. Which nursing action can the nurse delegate to the unlicensed assistive personnel (UAP)? a. Document the amount of drainage every eight hours. b. Obtain samples of drainage for culture from the system. c. Assess patient pain level associated with the chest tube. d. Check the water-seal chamber for the correct fluid level.

ANS: A UAP education includes documentation of intake and output. The other actions are within the scope of practice and education of licensed nursing personnel.

14. Normally, the central chemoreceptors responsible for triggering ventilation changes respond to which of the following? a. Increased PaCO2 c. Decreased PaO2 b. Increased HCO3- d. Increased PaO2

ANS: A Ventilation increases when the hydrogen ion concentration increases and decreases when the hydrogen ion concentration decreases. An increase in the partial pressure of carbon dioxide (PaCO2) causes the movement of carbon dioxide across the blood-brain barrier into the cerebrospinal fluid, stimulating the movement of hydrogen ions into the brain's extracellular fluid. Peripheral chemoreceptors respond to changes in PaO2 levels.

24. The movement of air into and out of the lungs is termed a. ventilation. c. diffusion. b. respiration. d. perfusion.

ANS: A Ventilation is the movement of air into and out of the lungs and is distinct from respiration, which refers to gas exchange, not movement by air. Respiration is the process of gas exchange by means of movement of oxygen from the atmosphere into the bloodstream and movement of carbon dioxide from the bloodstream into the atmosphere. Diffusion moves molecules from an area of high concentration to an area of low concentration. The distribution of perfusion through the lungs is related to gravity and intra-alveolar pressures.

13. Voice sounds such as bronchophony, egophony, and whispering pectoriloquy are increased in a. pneumonia with consolidation. c. asthma. b. pneumothorax. d. bronchiectasis.

ANS: A Voice sounds are increased in pneumonia with consolidation because there is increased vibration through material. Bronchophony and whispering pectoriloquy are heard as clear transmission of sounds on auscultation; egophony is heard as an "a" sound when the client is saying "e."

8. A young adult patient who denies any history of smoking is seen in the clinic with a new diagnosis of chronic obstructive pulmonary disease (COPD). It is most appropriate for the nurse to teach the patient about a. α1-antitrypsin testing. b. use of the nicotine patch. c. continuous pulse oximetry. d. effects of leukotriene modifiers.

ANS: A When COPD occurs in young patients, especially without a smoking history, a genetic deficiency in α1-antitrypsin should be suspected. Because the patient does not smoke, a nicotine patch would not be ordered. There is no indication that the patient requires continuous pulse oximetry. Leukotriene modifiers would be used in patients with asthma, not with COPD.

12. The assist-control mode of ventilation functions in which of the following manners? a. It delivers gas at preset volume, at a set rate, and in response to the patient's inspiratory efforts. b. It delivers gas at a preset volume, allowing the patient to breathe spontaneously at his or her own volume. c. It applies positive pressure during both ventilator breaths and spontaneous breaths. d. It delivers gas at preset rate and tidal volume regardless of the patient's inspiratory efforts.

ANS: A Whereas a breath that is initiated by the patient is known as a patient-triggered or patient-assisted breath, a breath that is initiated by the ventilator is known as a machine-triggered or machine-controlled breath. A time-triggered breath is a machine-controlled breath that is initiated by the ventilator after a preset length of time has elapsed. It is controlled by the rate setting on the ventilator (e.g., a rate of 10 breaths/min yields 1 breath every 6 seconds). Flow-triggered and pressure-triggered breaths are patient-assisted breaths that are initiated by decreased flow or pressure, respectively, within the breathing circuit.

12. A patient presents with absent lung sounds in the left lower lung fields, moderate shortness of breath, and dyspnea. The nurse suspects pneumothorax and notifies the physician or nurse practitioner. Orders for a STAT chest radiography and reading are obtained. Which of the following findings best supports the nurse's suspicions? a. Blackness in the left lower lung area b. Whiteness in the left lower lung area c. Blunted costophrenic angles d. Elevated left hemidiaphragm

ANS: A With a pneumothorax, the pleural edges become evident as one looks through and between the images of the ribs on the film. A thin line appears just parallel to the chest wall, indicating where the lung markings have pulled away from the chest wall. In addition, the collapsed lung will be manifested as an area of increased density separated by an area of radiolucency (blackness).

18. A client just involved in a motor vehicle accident has sustained blunt chest trauma as part of his injuries. The nurse assessment reveals absent breath sounds in the left lung field. A left-sided pneumothorax is suspected and is further validated when assessment of the trachea reveals a. a shift to the right. c. no deviation. b. a shift to the left. d. subcutaneous emphysema

ANS: A With a pneumothorax, the trachea shifts to the opposite side of the problem; with atelectasis, the trachea shifts to the same side as the problem. Subcutaneous emphysema is more commonly related to a pneumomediastinum and is not specifically related to the trachea but to air trapped in the mediastinum and general neck area.

2. Which of the following oxygen administration devices can deliver oxygen concentrations of 90%? a. Nonrebreathing mask c. Partial rebreathing mask b. Nasal cannula d. Simple mask

ANS: A With an FiO2 of 55% to 70%, a nonrebreathing mask with a tight seal over the face can deliver 90% to 100% oxygen. It is used in emergencies and short-term therapy requiring moderate to high FiO2.

2. Which of the following should be used when suctioning a mechanically ventilated patient? (Select all that apply.) a. Three hyperoxygenation breaths (breaths at 100% FiO2) b. Hyperinflation (breaths at 150% tidal volume) c. Limit the number of passes to three. d. Instill 5 to 10 mL of normal saline to facilitate secretion removal. e. Use intermittent suction to avoid damaging tracheal tissue.

ANS: A, B, C Hyperoxygenation, hyperinflation, and limiting the number of passes help avoid desaturation. There is no evidence to suggest that intermittent suction reduces damage, and saline instillation can actually increase the risk for infection.

4. Weaning methods that are used in combination with each other include (Select all that apply.) a. SIMV with CPAP. b. SIMV with PSV. c. CPAP with PSV. d. T-piece and PSV. e. PEEP with CPAP.

ANS: A, B, C, D A variety of weaning methods are available, but no one method has consistently proven to be superior to the others. These methods include T-tube (T-piece), continuous positive airway pressure (CPAP), pressure support ventilation (PSV), and synchronized intermittent mandatory ventilation (SIMV). One recent multicenter study lends evidence to support the use of PSV for weaning over T-tube or SIMV weaning. Often these weaning methods are used in combination with each other, such as SIMV with PSV, CPAP with PSV, or SIMV with CPAP.

3. Psychologic factors that may contribute to long-term mechanical ventilation dependence include (Select all that apply.) a. fear. b. delirium. c. lack of confidence in the ability to breathe. d. depression. e. trust in the staff so the patient displays a lack of effort.

ANS: A, B, C, D Psychologic factors contributing to long-term mechanical ventilation dependence include a loss of breathing pattern control (anxiety, fear, dyspnea, pain, ventilator asynchrony, lack of confidence in ability to breathe), lack of motivation and confidence (inadequate trust in staff, depersonalization, hopelessness, powerlessness, depression, inadequate communication), and delirium (sensory overload, sensory deprivation, sleep deprivation, pain medications).

1. Which of the following regarding the client history will assist the nurse in developing the plan of management? (Select all that apply.) a. Provides direction for the rest of the assessment b. Exposes key clinical manifestations c. Aids in developing the plan of care d. The degree of the client's distress determines the extent of the interview e. Determines length of stay in the hospital setting

ANS: A, B, C, D The initial presentation of the patient determines the rapidity and direction of the interview. For a patient in acute distress, the history should be curtailed to just a few questions about the patient's chief complaint and precipitating events.

1. Complications of ETT tubes include (Select all that apply.) a. tracheoesophageal fistula. b. cricoid abscess. c. tracheal stenosis. d. tube obstruction. e. tube displacements.

ANS: A, B, C, D, E Complications of endotracheal tubes include tube obstruction, tube displacement, sinusitis and nasal injury, tracheoesophageal fistula, mucosal lesions, laryngeal or tracheal stenosis, and cricoid abscess.

2. Nursing management of a patient undergoing a diagnostic procedure involves (Select all that apply.) a. positioning the patient for the procedure. b. monitoring the patient's responses to the procedure. c. monitoring vital signs. d. teaching the patient about the procedure. e. medicating the patient before and after procedure.

ANS: A, B, C, D, E Preparing the patient includes teaching the patient about the procedure, answering any questions, and positioning the patient for the procedure. Monitoring the patient's responses to the procedure includes observing the patient for signs of pain and anxiety and monitoring vital signs, breath sounds, and oxygen saturation. Assessing the patient after the procedure includes observing for complications of the procedure and medicating the patient for any postprocedural discomfort.

1. Medical management of a patient with status asthmaticus that supports oxygenation and ventilation include (Select all that apply.) a. oxygen therapy. b. bronchodilators. c. corticosteroids. d. antibiotics. e. intubation and mechanical ventilation.

ANS: A, B, C, E Medical management of a patient with status asthmaticus is directed toward supporting oxygenation and ventilation. Bronchodilators, corticosteroids, oxygen therapy, and intubation and mechanical ventilation are the mainstays of therapy.

1. Which factors will the nurse consider when calculating the CURB-65 score for a patient with pneumonia (select all that apply)? a. Age b. Blood pressure c. Respiratory rate d. Oxygen saturation e. Presence of confusion f. Blood urea nitrogen (BUN) level

ANS: A, B, C, E, F Data collected for the CURB-65 are mental status (confusion), BUN (elevated), blood pressure (decreased), respiratory rate (increased), and age (65 and older). The other information is also essential to assess, but are not used for CURB-65 scoring.

1. Risk factors that need to be considered with thoracentesis include (Select all that apply.) a. coagulation defects. b. intra-aortic balloon pump. c. pleural effusion. d. uncooperative patient. e. empyema.

ANS: A, B, D No absolute contraindications to thoracentesis exist, although some risks may contraindicate the procedure in all but emergency situations. These risk factors include unstable hemodynamics, coagulation defects, mechanical ventilation, the presence of an intra-aortic balloon pump, and patients who are uncooperative. It is used most often as a diagnostic measure; it may also be performed therapeutically for the drainage of a pleural effusion or empyema.

3. A patient with respiratory failure has a respiratory rate of 6 breaths/minute and an oxygen saturation (SpO2) of 88%. The patient is increasingly lethargic. Which intervention will the nurse anticipate? a. Administration of 100% oxygen by non-rebreather mask b. Endotracheal intubation and positive pressure ventilation c. Insertion of a mini-tracheostomy with frequent suctioning d. Initiation of continuous positive pressure ventilation (CPAP)

ANS: B The patient's lethargy, low respiratory rate, and SpO2 indicate the need for mechanical ventilation with ventilator-controlled respiratory rate. Administration of high flow oxygen will not be helpful because the patient's respiratory rate is so low. Insertion of a mini-tracheostomy will facilitate removal of secretions, but it will not improve the patient's respiratory rate or oxygenation. CPAP requires that the patient initiate an adequate respiratory rate to allow adequate gas exchange.

2. Nursing management of the patient with acute lung failure includes which of the following interventions? (Select all that apply.) a. Positioning the patient with the least affected side up b. Providing adequate rest between treatments c. Performing percussion and postural drainage every 4 hours d. Controlling fever e. Pharmaceutical medications to control anxiety

ANS: A, B, D, E The goal of positioning is to place the least affected area of the patient's lung in the most dependent position. Patients with unilateral lung disease should be positioned with the healthy lung in a dependent position. Patients with diffuse lung disease may benefit from being positioned with the right lung down because it is larger and more vascular than the left lung. For patients with alveolar hypoventilation, the goal of positioning is to facilitate ventilation. These patients benefit from nonrecumbent positions such as sitting or a semierect position. In addition, semirecumbency has been shown to decrease the risk of aspiration and inhibit the development of hospital-associated pneumonia. Frequent repositioning (at least every 2 hours) is beneficial in optimizing the patient's ventilatory pattern and ventilation/perfusion matching. These include performing procedures only as needed, hyperoxygenating the patient before suctioning, providing adequate rest and recovery time between various procedures, and minimizing oxygen consumption. Interventions to minimize oxygen consumption include limiting the patient's physical activity, administering sedation to control anxiety, and providing measures to control fever.

2. Deviation of the trachea occurs in which of the following conditions? (Select all that apply.) a. Pneumothorax b. Pulmonary fibrosis c. Chronic obstructive pulmonary disease d. Emphysema e. Pleural effusion

ANS: A, B, E Assessment of tracheal position assists in the diagnosis of pneumothorax, unilateral pneumonia, pulmonary fibrosis, and pleural effusion.

3. Which of the following statements are true regarding rotational therapy? (Select all that apply.) a. Continuous lateral rotation therapy (CLRT) can be effective for improving oxygenation if used for at least 18 hours/day. b. Kinetic therapy can decrease the incidence of ventilator-acquired pneumonia in neurologic and postoperative patients. c. Use of rotational therapy eliminates the need for other pressure ulcer prevention strategies. d. CLRT helps avoid hemodynamic instability secondary to the continuous, gentle turning of the patient. e. CLRT has minimal pulmonary benefits for critically ill patients.

ANS: A, B, E Studies have found that to achieve benefits with CLRT or kinetic therapy, rotation must be aggressive, and the patient must be at least 40 degrees per side, with a total arc of at least 80 degrees for at least 18 hours a day. Kinetic therapy has been shown to decrease the incidence of ventilator-acquired pneumonia, particularly in neurologic and postoperative patients. Complications of the procedure include dislodgment or obstruction of tubes, drains, and lines; hemodynamic instability; and pressure ulcers. Lateral rotation does not replace manual repositioning to prevent pressure ulcers. CLRT has been shown to be of minimal pulmonary benefit for the critically ill patients.

4. Identify the clinical manifestations associated with oxygen toxicity. (Select all that apply.) a. Substernal chest pain that increases with deep breathing b. Moist cough and tracheal irritation c. Pleuritic pain occurring on inhalation, followed by dyspnea d. Increasing CO2 e. Sore throat and eye and ear discomfort

ANS: A, C, E A number of clinical manifestations are associated with oxygen toxicity. The first symptom is substernal chest pain that is exacerbated by deep breathing. A dry cough and tracheal irritation follow. Eventually, definite pleuritic pain occurs on inhalation followed by dyspnea. Upper airway changes may include a sensation of nasal stuffiness, sore throat, and eye and ear discomforts.

1. Muscles of exhalation include which of the following? (Select all that apply.) a. Abdominal b. Diaphragm c. External intercostals d. Internal intercostals e. Scalene

ANS: A, D Exhalation occurs when the diaphragm relaxes and moves back up toward the lungs. The intrinsic elastic recoil of the lungs assists with exhalation. Because exhalation is a passive act, there are no true muscles of exhalation other than the internal intercostal muscles, which assist the inward movement of the ribs.

3. A diabetic patient's arterial blood gas (ABG) results are pH 7.28; PaCO2 34 mm Hg; PaO2 85 mm Hg; HCO3- 18 mEq/L. The nurse would expect which finding? a. Intercostal retractions b. Kussmaul respirations c. Low oxygen saturation (SpO2) d. Decreased venous O2 pressure

ANS: B Kussmaul (deep and rapid) respirations are a compensatory mechanism for metabolic acidosis. The low pH and low bicarbonate result indicate metabolic acidosis. Intercostal retractions, a low oxygen saturation rate, and a decrease in venous O2 pressure would not be caused by acidosis.

15. When caring for a patient who is hospitalized with active tuberculosis (TB), the nurse observes a student nurse who is assigned to take care of a patient. Which action, if performed by the student nurse, would require an intervention by the nurse? a. The patient is offered a tissue from the box at the bedside. b. A surgical face mask is applied before visiting the patient. c. A snack is brought to the patient from the unit refrigerator. d. Hand washing is performed before entering the patient's room.

ANS: B A high-efficiency particulate-absorbing (HEPA) mask, rather than a standard surgical mask, should be used when entering the patient's room because the HEPA mask can filter out 100% of small airborne particles. Hand washing before entering the patient's room is appropriate. Because anorexia and weight loss are frequent problems in patients with TB, bringing food to the patient is appropriate. The student nurse should perform hand washing after handling a tissue that the patient has used, but no precautions are necessary when giving the patient an unused tissue.

28. Place the steps for analyzing arterial blood gases in the proper order. 1. Assess HCO3- level for metabolic abnormalities. 2. Assess PaO2 for hypoxemia. 3. Examine PaCO2 for acidosis or alkalosis. 4. Re-examine pH to determine level of compensation. 5. Examine pH for acidemia or alkalemia. a. 5, 1, 2, 4, 3 c. 1, 2, 4, 3, 5 b. 2, 5, 3, 1, 4 d. 1, 3, 4, 5, 2

ANS: B A methodic approach when assessing arterial blood gases allows the nurse to detect subtle changes. A methodic approach includes look at the Pao2 level, look at the pH level, look at the Paco2 level, look at the HCO3-, and look again at the pH level.

42. The nurse receives a change-of-shift report on the following patients with chronic obstructive pulmonary disease (COPD). Which patient should the nurse assess first? a. A patient with loud expiratory wheezes b. A patient with a respiratory rate of 38/minute c. A patient who has a cough productive of thick, green mucus d. A patient with jugular venous distention and peripheral edema

ANS: B A respiratory rate of 38/minute indicates severe respiratory distress, and the patient needs immediate assessment and intervention to prevent possible respiratory arrest. The other patients also need assessment as soon as possible, but they do not need to be assessed as urgently as the tachypneic patient.

27. A patient with chronic obstructive pulmonary disease (COPD) has rhonchi throughout the lung fields and a chronic, nonproductive cough. Which nursing intervention will be most effective? a. Change the oxygen flow rate to the highest prescribed rate. b. Teach the patient to use the Flutter airway clearance device. c. Reinforce the ongoing use of pursed lip breathing techniques. d. Teach the patient about consistent use of inhaled corticosteroids.

ANS: B Airway clearance devices assist with moving mucus into larger airways where it can more easily be expectorated. The other actions may be appropriate for some patients with COPD, but they are not indicated for this patient's problem of thick mucus secretions.

13. A patient with chronic obstructive pulmonary disease requires intubation. After the physician intubates the patient, the nurse auscultates for breath sounds. Breath sounds are questionable in this patient. Which action would best assist in determining endotracheal tube placement in this patient? a. Stat chest radiographic examination b. End-tidal CO2 monitor c. V/Q scan d. Pulmonary artery catheter insertion

ANS: B Although a stat chest radiography examination would be helpful, it has a long turnaround time, and the patient's respiratory status can deteriorate quickly. An end-tidal CO2 monitor gives an immediate response, and the tube can then be reinserted without delay if incorrectly placed. The other tests are not for endotracheal tube placement.

27. A patient with idiopathic pulmonary arterial hypertension (IPAH) is receiving nifedipine (Procardia). Which assessment would best indicate to the nurse that the patient's condition is improving? a. Blood pressure (BP) is less than 140/90 mm Hg. b. Patient reports decreased exertional dyspnea. c. Heart rate is between 60 and 100 beats/minute. d. Patient's chest x-ray indicates clear lung fields.

ANS: B Because a major symptom of IPAH is exertional dyspnea, an improvement in this symptom would indicate that the medication was effective. Nifedipine will affect BP and heart rate, but these parameters would not be used to monitor the effectiveness of therapy for a patient with IPAH. The chest x-ray will show clear lung fields even if the therapy is not effective.

22. Which of the following statements describes the relationship between the pulmonary circulation and the pulmonary vascular bed? a. The pulmonary circulation is a high-pressure system with normal pressures averaging 100/60 to 120/70 mm Hg. b. Because of the low pulmonary arterial pressures, the right ventricular wall thickness needs to be only one-third that of the left ventricle. c. Pulmonary hypertension is defined as increased pulmonary artery systolic pressure above 20 mm Hg. d. The most common cause of pulmonary hypertension is right-sided heart failure.

ANS: B Because of low pulmonary artery pressures, right ventricular wall thickness needs to be only approximately one-third of left ventricular wall thickness. Pulmonary hypertension is defined as increased pressure (pulmonary artery systolic greater than 35 mm Hg and pulmonary artery mean less than 25 mm Hg at rest or less than 30 mm Hg with exertion) within the pulmonary arterial system. Pulmonary hypertension increases the afterload of the right ventricle and, when chronic, can result in right ventricular hypertrophy (cor pulmonale) and failure.

18. After the nurse has received change-of-shift report, which patient should the nurse assess first? a. A patient with pneumonia who has crackles in the right lung base b. A patient with possible lung cancer who has just returned after bronchoscopy c. A patient with hemoptysis and a 16-mm induration with tuberculin skin testing d. A patient with chronic obstructive pulmonary disease (COPD) and pulmonary function testing (PFT) that indicates low forced vital capacity

ANS: B Because the cough and gag are decreased after bronchoscopy, this patient should be assessed for airway patency. The other patients do not have clinical manifestations or procedures that require immediate assessment by the nurse.

7. A patient with acute respiratory distress syndrome (ARDS) and acute kidney injury has the following medications ordered. Which medication should the nurse discuss with the health care provider before giving? a. Pantoprazole (Protonix) 40 mg IV b. Gentamicin (Garamycin) 60 mg IV c. Sucralfate (Carafate) 1 g per nasogastric tube d. Methylprednisolone (Solu-Medrol) 60 mg IV

ANS: B Gentamicin, which is one of the aminoglycoside antibiotics, is potentially nephrotoxic, and the nurse should clarify the drug and dosage with the health care provider before administration. The other medications are appropriate for the patient with ARDS.

14. A patient admitted with acute respiratory failure has a nursing diagnosis of ineffective airway clearance related to thick, secretions. Which action is a priority for the nurse to include in the plan of care? a. Encourage use of the incentive spirometer. b. Offer the patient fluids at frequent intervals. c. Teach the patient the importance of ambulation. d. Titrate oxygen level to keep O2 saturation >93%.

ANS: B Because the reason for the poor airway clearance is the thick secretions, the best action will be to encourage the patient to improve oral fluid intake. Patients should be instructed to use the incentive spirometer on a regular basis (e.g., every hour) in order to facilitate the clearance of the secretions. The other actions may also be helpful in improving the patient's gas exchange, but they do not address the thick secretions that are causing the poor airway clearance.

2. A patient with acute respiratory failure may require a bronchodilator if which of the following occurs? a. Excessive secretions c. Thick secretions b. Bronchospasms d. Fighting the ventilator

ANS: B Bronchodilators aid in smooth muscle relaxation and are of particular benefit to patients with airflow limitations. Mucolytics and expectorants are no longer used because they have been found to be of no benefit in this patient population.

8. For which of the following conditions is a bronchoscopy indicated? a. Pulmonary edema c. Upper gastrointestinal bleed b. Ineffective clearance of secretions d. Instillation of surfactant

ANS: B Bronchoscopy visualizes the bronchial tree. If secretions are present, they can be removed by suctioning and sent for culture to help adjust antibiotic therapy.

10. Which statement by the nurse when explaining the purpose of positive end-expiratory pressure (PEEP) to the family members of a patient with ARDS is accurate? a. "PEEP will push more air into the lungs during inhalation." b. "PEEP prevents the lung air sacs from collapsing during exhalation." c. "PEEP will prevent lung damage while the patient is on the ventilator." d. "PEEP allows the breathing machine to deliver 100% oxygen to the lungs."

ANS: B By preventing alveolar collapse during expiration, PEEP improves gas exchange and oxygenation. PEEP will not prevent lung damage (e.g., fibrotic changes that occur with ARDS), push more air into the lungs, or change the fraction of inspired oxygen (FIO2) delivered to the patient.

12. The adventitious breath sounds that sound like popping in the small airways or alveoli are a. sonorous wheezes. c. sibilant wheezes. b. crackles. d. pleural friction rubs.

ANS: B Crackles or rales are short, discrete, popping or crackling sounds produced by fluid in the small airways or alveoli.

12. The central venous oxygen saturation (ScvO2) is decreasing in a patient who has severe pancreatitis. To determine the possible cause of the decreased ScvO2, the nurse assesses the patient's a. lipase. b. temperature. c. urinary output. d. body mass index.

ANS: B Elevated temperature increases metabolic demands and oxygen use by tissues, resulting in a drop in oxygen saturation of central venous blood. Information about the patient's body mass index, urinary output, and lipase will not help in determining the cause of the patient's drop in ScvO2.

16. A patient was admitted to the critical care unit with acute respiratory failure. The patient has been on a ventilator for 3 days and is being considered for weaning. The ventilator high-pressure alarm keeps going off. When you enter the room, the ventilator inoperative alarm sounds. What is the primary action the nurse would take? a. Troubleshoot the ventilator until the problem is found. b. Take the patient off the ventilator and manually ventilate her. c. Call the respiratory therapist for help. d. Silence the ventilator alarms until the problem is resolved.

ANS: B Ensure emergency equipment is at bedside at all times (e.g., manual resuscitation bag connected to oxygen, masks, suction equipment or supplies), including preparations for power failures. If the ventilator malfunctions, the patient should be removed from the ventilator and ventilated manually with a manual resuscitation bag.

30. Which of the following factors will result in a shift of the oxyhemoglobin dissociation curve to the left? a. Increased PaCO2 c. Increased temperature b. Increased pH d. Increased 2,3-DPG

ANS: B Factors shifting the curve to the left are increased pH, decreased PaCO2, decreased temperature, and decreased 2,3-DPG.

37. Which assessment finding in a patient who has received omalizumab (Xolair) is most important to report immediately to the health care provider? a. Pain at injection site b. Flushing and dizziness c. Peak flow reading 75% of normal d. Respiratory rate 22 breaths/minute

ANS: B Flushing and dizziness may indicate that the patient is experiencing an anaphylactic reaction, and immediate intervention is needed. The other information should also be reported, but do not indicate possibly life-threatening complications of omalizumab therapy.

14. Which finding by the nurse for a patient with a nursing diagnosis of impaired gas exchange will be most useful in evaluating the effectiveness of treatment? a. Even, unlabored respirations b. Pulse oximetry reading of 92% c. Respiratory rate of 18 breaths/minute d. Absence of wheezes, rhonchi, or crackles

ANS: B For the nursing diagnosis of impaired gas exchange, the best data for evaluation are arterial blood gases (ABGs) or pulse oximetry. The other data may indicate either improvement or impending respiratory failure caused by fatigue.

45. An experienced nurse instructs a new nurse about how to care for a patient with dyspnea caused by a pulmonary fungal infection. Which action by the new nurse indicates a need for further teaching? a. Listening to the patient's lung sounds several times during the shift b. Placing the patient on droplet precautions and in a private hospital room c. Increasing the oxygen flow rate to keep the oxygen saturation above 90% d. Monitoring patient serology results to identify the specific infecting organism

ANS: B Fungal infections are not transmitted from person to person. Therefore no isolation procedures are necessary. The other actions by the new nurse are appropriate.

23. A patient was taken to surgery for a left lung resection earlier today. The patient has been in the postoperative care unit for 30 minutes. When you are completing your assessment, you notice that the chest tube has drained 125 cc of red fluid in the past 30 minutes. The nurse contacts the physician and suspects that the patient has developed a. pulmonary edema. c. acute lung failure. b. hemorrhage. d. bronchopleural fistula.

ANS: B Hemorrhage is an early, life-threatening complication that can occur after a lung resection. It can result from bronchial or intercostal artery bleeding or disruption of a suture or clip around a pulmonary vessel. Excessive chest tube drainage can signal excessive bleeding. During the immediate postoperative period, chest tube drainage should be measured every 15 minutes; this frequency should be decreased as the patient stabilizes. If chest tube loss is greater than 100 mL/hr, fresh blood is noted, or a sudden increase in drainage occurs, hemorrhage should be suspected.

28. The nurse provides dietary teaching for a patient with chronic obstructive pulmonary disease (COPD) who has a low body mass index (BMI). Which patient statement indicates that the teaching has been effective? a. "I will drink lots of fluids with my meals." b. "I can have ice cream as a snack every day." c. "I will exercise for 15 minutes before meals." d. "I will decrease my intake of meat and poultry."

ANS: B High-calorie foods like ice cream are an appropriate snack for patients with COPD. Fluid intake of 3 L/day is recommended, but fluids should be taken between meals rather than with meals to improve oral intake of solid foods. The patient should avoid exercise for an hour before meals to prevent fatigue while eating. Meat and dairy products are high in protein and are good choices for the patient with COPD.

30. A patient has just been admitted with probable bacterial pneumonia and sepsis. Which order should the nurse implement first? a. Chest x-ray via stretcher b. Blood cultures from two sites c. Ciprofloxacin (Cipro) 400 mg IV d. Acetaminophen (Tylenol) rectal suppository

ANS: B Initiating antibiotic therapy rapidly is essential, but it is important that the cultures be obtained before antibiotic administration. The chest x-ray and acetaminophen administration can be done last.

22. Which action is appropriate for the nurse to delegate to unlicensed assistive personnel (UAP)? a. Listen to a patient's lung sounds for wheezes or rhonchi. b. Label specimens obtained during percutaneous lung biopsy. c. Instruct a patient about how to use home spirometry testing. d. Measure induration at the site of a patient's intradermal skin test.

ANS: B Labeling of specimens is within the scope of practice of UAP. The other actions require nursing judgment and should be done by licensed nursing personnel.

24. Which medication may be administered with a bronchodilator because it can cause bronchospasms? a. β2-Agonists c. Anticholinergic agents b. Mucloytics d. Xanthines

ANS: B Mucolytics may be administered with a bronchodilator because it can cause bronchospasms and inhibit ciliary function. Treatment is considered effective when bronchorrhea develops and coughing occurs. β2-Agonists are used to relax bronchial smooth muscle and dilate airways to prevent bronchospasms. Anticholinergic agents are used to block the constriction of bronchial smooth muscle and reduce mucus production. Xanthines are used to dilate bronchial smooth muscle and reverse diaphragmatic muscle fatigue.

15. Which of the following patients would be considered hypoxemic? a. A 70-year-old man with a PaO2 of 72 b. A 50-year-old woman with a PaO2 of 65 c. An 84-year-old man with a PaO2 of 96 d. A 68-year-old woman with a PaO2 of 80

ANS: B Normal PaO2 is 80 to 100 mm Hg in persons younger than 60 years. The formula for determining PaO2 for a person older than 60 years of age is 80 mm Hg minus 1 mm Hg for every year of age above 60 years of age, for example, 70 years old = 80 mm Hg - 10 mm Hg = 70 mm Hg; 84 years old = 80 mm Hg - 20 mm Hg = 60 mm Hg; and 68 years old = 80 mm Hg - 8 mm Hg = 72 mm Hg.

20. The use of touch to judge the character of the body surface and underlying organs is known as a. inspection. c. percussion. b. palpation. d. auscultation.

ANS: B Palpation is the process of touching the patient to judge the size, shape, texture, and temperature of the body surface or underlying structures. Inspection is the process of looking intently at the patient. Percussion is the process of creating sound waves on the surface of the body to determine abnormal density of any underlying areas. Auscultation is the process of concentrated listening with a stethoscope to determine characteristics of body functions.

15. Nursing management of the patient receiving a neuromuscular blocking agent should include a. withholding all sedation and narcotics. b. protecting the patient from the environment. c. keeping the patient supine. d. speaking to the patient only when necessary.

ANS: B Patient safety is a major concern for the patient receiving a neuromuscular blocking agent because these patients are unable to protect themselves from the environment. Special precautions should be taken to protect the patient at all times.

20. Patient safety precautions when working with oxygen involve a. observation for signs of oxygen-introduced hyperventilation. b. restriction of smoking. c. removal of all oxygen devices when eating to prevent aspiration. d. administration of oxygen at the nurse's discretion.

ANS: B Patient safety precautions when working with oxygen involve administration of oxygen and monitoring of its effectiveness. Activities include restricting smoking, administering supplemental oxygen as ordered, observing for signs of oxygen-induced hypoventilation, monitoring the patient's ability to tolerate removal of oxygen while eating, and changing the oxygen delivery device from a mask to nasal prongs during meals as tolerated.

1. Which actions should the nurse initiate to reduce the risk for ventilator-associated pneumonia (VAP) (select all that apply)? a. Obtain arterial blood gases daily. b. Provide a "sedation holiday" daily. c. Elevate the head of the bed to at least 30°. d. Give prescribed pantoprazole (Protonix). e. Provide oral care with chlorhexidine (0.12%) solution daily.

ANS: B, C, D, E All of these interventions are part of the ventilator bundle that is recommended to prevent VAP. Arterial blood gases may be done daily but are not always necessary and do not help prevent VAP.

37. The nurse receives change-of-shift report on the following four patients. Which patient should the nurse assess first? a. A 23-year-old patient with cystic fibrosis who has pulmonary function testing scheduled b. A 46-year-old patient on bed rest who is complaining of sudden onset of shortness of breath c. A 77-year-old patient with tuberculosis (TB) who has four antitubercular medications due in 15 minutes d. A 35-year-old patient who was admitted the previous day with pneumonia and has a temperature of 100.2° F (37.8° C)

ANS: B Patients on bed rest who are immobile are at high risk for deep vein thrombosis (DVT). Sudden onset of shortness of breath in a patient with a DVT suggests a pulmonary embolism and requires immediate assessment and action such as oxygen administration. The other patients should also be assessed as soon as possible, but there is no indication that they may need immediate action to prevent clinical deterioration.

4. The nurse provides discharge instructions to a patient who was hospitalized for pneumonia. Which statement, if made by the patient, indicates a good understanding of the instructions? a. "I will call the doctor if I still feel tired after a week." b. "I will continue to do the deep breathing and coughing exercises at home." c. "I will schedule two appointments for the pneumonia and influenza vaccines." d. "I'll cancel my chest x-ray appointment if I'm feeling better in a couple weeks."

ANS: B Patients should continue to cough and deep breathe after discharge. Fatigue is expected for several weeks. The Pneumovax and influenza vaccines can be given at the same time in different arms. Explain that a follow-up chest x-ray needs to be done in 6 to 8 weeks to evaluate resolution of pneumonia.

16. A patient was admitted to the critical care unit after a left pneumonectomy. The patient is receiving 40% oxygen via a simple facemask. The morning chest radiography study reveals right lower lobe pneumonia. After eating breakfast, the patient suddenly vomits and aspirates. The single most important measure to prevent the spread of infection between staff and patients is a. respiratory isolation. c. use of PPE. b. hand washing. d. antibiotics.

ANS: B Proper hand hygiene is the single most important measure available to prevent the spread of bacteria from person to person.

33. A patient is admitted to the critical care unit with an acute exacerbation of chronic emphysema. Respirations are 28 breaths/min. A pulmonary artery catheter is in place. Pulmonary artery systolic (PAS) is 38 mm Hg with a pulmonary artery mean (PAM) of 22 mm Hg. Blood gases reveal an uncompensated respiratory acidosis. The patient's work effort for breathing is increased related to the effect of emphysema, which is a. increased lung compliance. c. increased chest wall compliance. b. decreased lung recoil. d. widespread atelectasis.

ANS: B Pulmonary diseases that decrease lung compliance (e.g., atelectasis, pulmonary edema), decrease chest wall compliance (e.g., kyphoscoliosis), increase airway resistance (e.g., bronchitis, asthma), or decrease lung recoil (e.g., emphysema) can increase the work of breathing so much that one-third or more of the total body energy is used for ventilation.

3. While family members are visiting, a patient has a respiratory arrest and is being resuscitated. Which action by the nurse is best? a. Tell the family members that watching the resuscitation will be very stressful. b. Ask family members if they wish to remain in the room during the resuscitation. c. Take the family members quickly out of the patient room and remain with them. d. Assign a staff member to wait with family members just outside the patient room.

ANS: B Research indicates that family members want the option of remaining in the room during procedures such as cardiopulmonary resuscitation (CPR) and that this decreases anxiety and facilitates grieving. The other options may be appropriate if the family decides not to remain with the patient.

6. A patient with a chronic cough has a bronchoscopy. After the procedure, which intervention by the nurse is most appropriate? a. Elevate the head of the bed to 80 to 90 degrees. b. Keep the patient NPO until the gag reflex returns. c. Place on bed rest for at least 4 hours after bronchoscopy. d. Notify the health care provider about blood-tinged mucus.

ANS: B Risk for aspiration and maintaining an open airway is the priority. Because a local anesthetic is used to suppress the gag/cough reflexes during bronchoscopy, the nurse should monitor for the return of these reflexes before allowing the patient to take oral fluids or food. Blood-tinged mucus is not uncommon after bronchoscopy. The patient does not need to be on bed rest, and the head of the bed does not need to be in the high-Fowler's position.

25. Which of the following chest radiography findings is consistent with a left pneumothorax? a. Flattening of the diaphragm b. Shifting of the mediastinum to the right c. Presence of a gastric air bubble d. Increased radiolucency of the left lung field

ANS: B Shifting of the mediastinal structures away from the area of involvement is a sign of a pneumothorax.

8. While conducting a physical assessment, you note that the patient's breathing is rapid and shallow. This type of breathing pattern is known as a. hyperventilation. c. obstructive breathing. b. tachypnea. d. bradypnea.

ANS: B Tachypnea is manifested by an increase in the rate and decrease in the depth of ventilation. Hyperventilation is manifested by an increase in both the rate and depth of ventilation. Obstructive breathing is characterized by progressively more shallow breathing until the client actively and forcefully exhales. Bradypnea is a slow respiratory rate characterized as less than 12 breaths/min in an adult.

17. You would expect a patient in diabetic ketoacidosis to exhibit which of the following? a. Breathe faster to increase pH c. Breathe faster to decrease pH b. Breathe slower to increase pH d. Breathe slower to decrease pH

ANS: C Breathing faster increases the expiration of CO2, which results in less acid in the bloodstream and a decreased pH.

9. The nurse teaches a patient about the transmission of pulmonary tuberculosis (TB). Which statement, if made by the patient, indicates that teaching was effective? a. "I will avoid being outdoors whenever possible." b. "My husband will be sleeping in the guest bedroom." c. "I will take the bus instead of driving to visit my friends." d. "I will keep the windows closed at home to contain the germs."

ANS: B Teach the patient how to minimize exposure to close contacts and household members. Homes should be well ventilated, especially the areas where the infected person spends a lot of time. While still infectious, the patient should sleep alone, spend as much time as possible outdoors, and minimize time in congregate settings or on public transportation.

6. Which of the following diagnostic criteria is indicative of ARDS? a. Radiologic evidence of bibasilar atelectasis b. PaO2/FiO2 ratio less than or equal to 200 mm Hg c. Pulmonary artery wedge pressure greater than 18 mm Hg d. Increased static and dynamic compliance

ANS: B The Berlin Definition of ARDS is as follows: timing—within 1 week of known clinical insult or new or worsening respiratory symptoms; chest imaging—bilateral opacities not fully explained by effusions, lobar or lung collapse, or nodules; origin of edema—respiratory failure not fully explained by cardiac failure or fluid overload; need objective assessment to exclude hydrostatic edema if no risk factor present; oxygenation—mild (200 mg Hg less than PaO2/FiO2 less than or equal to 300 mm Hg with positive end-respiratory airway pressure (PEEP) or constant positive airway pressure greater than or equal to 5 cm H2O), moderate (100 mg Hg less than PaO2/FiO2 less than or equal to 200 mm Hg with PEEP greater than or equal to 5 cm H2O), or severe (PaO2/FiO2 less than or equal to 100 mm Hg with PEEP greater than or equal to 5 cm H2O). The mortality rate for ARDS is estimated to be 34% to 58%.

18. A patient is receiving 35% oxygen via a Venturi mask. To ensure the correct amount of oxygen delivery, which action by the nurse is most important? a. Teach the patient to keep mask on at all times. b. Keep the air entrainment ports clean and unobstructed. c. Give a high enough flow rate to keep the bag from collapsing. d. Drain moisture condensation from the oxygen tubing every hour.

ANS: B The air entrainment ports regulate the oxygen percentage delivered to the patient, so they must be unobstructed. A high oxygen flow rate is needed when giving oxygen by partial rebreather or non-rebreather masks. Draining oxygen tubing is necessary when caring for a patient receiving mechanical ventilation. The mask is uncomfortable and can be removed when the patient eats.

16. The nurse is admitting a patient diagnosed with an acute exacerbation of chronic obstructive pulmonary disease (COPD).What is the best way for the nurse to determine the appropriate oxygen flow rate? a. Minimize oxygen use to avoid oxygen dependency. b. Maintain the pulse oximetry level at 90% or greater. c. Administer oxygen according to the patient's level of dyspnea. d. Avoid administration of oxygen at a rate of more than 2 L/minute.

ANS: B The best way to determine the appropriate oxygen flow rate is by monitoring the patient's oxygenation either by arterial blood gases (ABGs) or pulse oximetry. An oxygen saturation of 90% indicates adequate blood oxygen level without the danger of suppressing the respiratory drive. For patients with an exacerbation of COPD, an oxygen flow rate of 2 L/min may not be adequate. Because oxygen use improves survival rate in patients with COPD, there is no concern about oxygen dependency. The patient's perceived dyspnea level may be affected by other factors (such as anxiety) besides blood oxygen level.

34. The nurse educator is evaluating the performance of a new registered nurse (RN) who is providing care to a patient who is receiving mechanical ventilation with 15 cm H2O of peak end-expiratory pressure (PEEP). Which action indicates that the new RN is safe? a. The RN plans to suction the patient every 1 to 2 hours. b. The RN uses a closed-suction technique to suction the patient. c. The RN tapes connection between the ventilator tubing and the ET. d. The RN changes the ventilator circuit tubing routinely every 48 hours.

ANS: B The closed-suction technique is used when patients require high levels of PEEP (>10 cm H2O) to prevent the loss of PEEP that occurs when disconnecting the patient from the ventilator. Suctioning should not be scheduled routinely, but it should be done only when patient assessment data indicate the need for suctioning. Taping connections between the ET and the ventilator tubing would restrict the ability of the tubing to swivel in response to patient repositioning. Ventilator tubing changes increase the risk for ventilator-associated pneumonia (VAP) and are not indicated routinely.

39. A patient is admitted to the emergency department with an open stab wound to the left chest. What is the first action that the nurse should take? a. Position the patient so that the left chest is dependent. b. Tape a nonporous dressing on three sides over the chest wound. c. Cover the sucking chest wound firmly with an occlusive dressing. d. Keep the head of the patient's bed at no more than 30 degrees elevation.

ANS: B The dressing taped on three sides will allow air to escape when intrapleural pressure increases during expiration, but it will prevent air from moving into the pleural space during inspiration. Placing the patient on the left side or covering the chest wound with an occlusive dressing will allow trapped air in the pleural space and cause tension pneumothorax. The head of the bed should be elevated to 30 to 45 degrees to facilitate breathing.

13. After 2 months of tuberculosis (TB) treatment with isoniazid (INH), rifampin (Rifadin), pyrazinamide (PZA), and ethambutol, a patient continues to have positive sputum smears for acid-fast bacilli (AFB). Which action should the nurse take next? a. Teach about treatment for drug-resistant TB treatment. b. Ask the patient whether medications have been taken as directed. c. Schedule the patient for directly observed therapy three times weekly. d. Discuss with the health care provider the need for the patient to use an injectable antibiotic.

ANS: B The first action should be to determine whether the patient has been compliant with drug therapy because negative sputum smears would be expected if the TB bacillus is susceptible to the medications and if the medications have been taken correctly. Assessment is the first step in the nursing process. Depending on whether the patient has been compliant or not, different medications or directly observed therapy may be indicated. The other options are interventions based on assumptions until an assessment has been completed.

3. Alveolar type II cells secrete which of the following lipoproteins? a. Trypsin c. Amylase b. Chyme d. Surfacta

ANS: D The most important function of the type II cells is their ability to produce, store, and secrete pulmonary surfactant. Trypsin and amylase are proteins used for digestion. Chyme is used to help with digestion.

14. A patient's pulse oximeter alarm goes off. The monitor reads 82%. What is the first action the nurse should perform? a. Prepare to intubate. b. Assess the patient's condition. c. Turn off the alarm and reapply the oximeter sensor. d. Increase O2 level to 4L/NC.

ANS: B The first nursing action would be to assess the patient to see if there is a change in his or her condition. If the patient is stable, then the nurse would turn off the alarm and reapply the oximeter sensor. The pulse oximeter cannot differentiate between normal and abnormal hemoglobin. Elevated levels of abnormal hemoglobin falsely elevate the Spo2. The ability of a pulse oximeter to detect hypoventilation is accurate only when the patient is breathing room air. Because most critically ill patients require some form of oxygen therapy, pulse oximetry is not a reliable method of detecting hypercapnia and should not be used for this purpose.

33. A patient with cystic fibrosis (CF) has blood glucose levels that are consistently between 180 to 250 mg/dL. Which nursing action will the nurse plan to implement? a. Discuss the role of diet in blood glucose control. b. Teach the patient about administration of insulin. c. Give oral hypoglycemic medications before meals. d. Evaluate the patient's home use of pancreatic enzymes.

ANS: B The glucose levels indicate that the patient has developed CF-related diabetes, and insulin therapy is required. Because the etiology of diabetes in CF is inadequate insulin production, oral hypoglycemic agents are not effective. Patients with CF need a high-calorie diet. Inappropriate use of pancreatic enzymes would not be a cause of hyperglycemia in a patient with CF.

5. The emergency department nurse is evaluating the effectiveness of therapy for a patient who has received treatment during an asthma attack. Which assessment finding is the best indicator that the therapy has been effective? a. No wheezes are audible. b. Oxygen saturation is >90%. c. Accessory muscle use has decreased. d. Respiratory rate is 16 breaths/minute.

ANS: B The goal for treatment of an asthma attack is to keep the oxygen saturation >90%. The other patient data may occur when the patient is too fatigued to continue with the increased work of breathing required in an asthma attack.

21. The nurse monitors a patient after chest tube placement for a hemopneumothorax. The nurse is most concerned if which assessment finding is observed? a. A large air leak in the water-seal chamber b. 400 mL of blood in the collection chamber c. Complaint of pain with each deep inspiration d. Subcutaneous emphysema at the insertion site

ANS: B The large amount of blood may indicate that the patient is in danger of developing hypovolemic shock. An air leak would be expected immediately after chest tube placement for a pneumothorax. Initially, brisk bubbling of air occurs in this chamber when a pneumothorax is evacuated. The pain should be treated but is not as urgent a concern as the possibility of continued hemorrhage. Subcutaneous emphysema should be monitored but is not unusual in a patient with pneumothorax. A small amount of subcutaneous air is harmless and will be reabsorbed.

10. The major hemodynamic consequence of a massive pulmonary embolus is a. increased systemic vascular resistance leading to left heart failure. b. pulmonary hypertension leading to right heart failure. c. portal vein blockage leading to ascites. d. embolism to the internal carotids leading to a stroke.

ANS: B The major hemodynamic consequence of a pulmonary embolus is the development of pulmonary hypertension, which is part of the effect of a mechanical obstruction when more than 50% of the vascular bed is occluded. In addition, the mediators released at the injury site and the development of hypoxia cause pulmonary vasoconstriction, which further exacerbates pulmonary hypertension.

8. Which of the following statements is correct concerning endotracheal tube cuff management? a. The cuff should be deflated every hour to minimize pressure on the trachea. b. A small leak should be heard on inspiration if the cuff has been inflated using the minimal leak technique. c. Cuff pressures should be kept between 40 to 50 mm Hg to ensure an adequate seal. d. Cuff pressure monitoring should be done once every 24 hours.

ANS: B The minimal leak technique consists of injecting air into the cuff until no leak is heard and then withdrawing the air until a small leak is heard on inspiration. Problems with this technique include difficulty maintaining positive end-expiratory pressure and aspiration around the cuff.

3. The most accurate and reliable control of FiO2 can be achieved through the use of a(n) a. simple mask. c. air-entrainment mask. b. nonrebreathing circuit (closed).

ANS: B The most reliable and accurate means of delivering a prescribed concentration of oxygen is through the use of a nonrebreathing circuit (closed).

24. The nurse is caring for a patient who is intubated and receiving positive pressure ventilation to treat acute respiratory distress syndrome (ARDS). Which finding is most important to report to the health care provider? a. Blood urea nitrogen (BUN) level 32 mg/dL b. Red-brown drainage from orogastric tube c. Scattered coarse crackles heard throughout lungs d. Arterial blood gases: pH 7.31, PaCO2 50, PaO2 68

ANS: B The nasogastric drainage indicates possible gastrointestinal bleeding and/or stress ulcer, and should be reported. The pH and PaCO2 are slightly abnormal, but current guidelines advocating for permissive hypercapnia indicate that these would not indicate an immediate need for a change in therapy. The BUN is slightly elevated but does not indicate an immediate need for action. Adventitious breath sounds are commonly heard in patients with ARDS.

26. A young adult female patient with cystic fibrosis (CF) tells the nurse that she is not sure about getting married and having children some day. Which initial response by the nurse is best? a. "Are you aware of the normal lifespan for patients with CF?" b. "Do you need any information to help you with that decision?" c. "Many women with CF do not have difficulty conceiving children." d. "You will need to have genetic counseling before making a decision."

ANS: B The nurse's initial response should be to assess the patient's knowledge level and need for information. Although the lifespan for patients with CF is likely to be shorter than normal, it would not be appropriate for the nurse to address this as the initial response to the patient's comments. The other responses have accurate information, but the nurse should first assess the patient's understanding about the issues surrounding pregnancy.

20. A patient with newly diagnosed lung cancer tells the nurse, "I don't think I'm going to live to see my next birthday." Which response by the nurse is best? a. "Would you like to talk to the hospital chaplain about your feelings?" b. "Can you tell me what it is that makes you think you will die so soon?" c. "Are you afraid that the treatment for your cancer will not be effective?" d. "Do you think that taking an antidepressant medication would be helpful?"

ANS: B The nurse's initial response should be to collect more assessment data about the patient's statement. The answer beginning "Can you tell me what it is" is the most open-ended question and will offer the best opportunity for obtaining more data. The answer beginning, "Are you afraid" implies that the patient thinks that the cancer will be immediately fatal, although the patient's statement may not be related to the cancer diagnosis. The remaining two answers offer interventions that may be helpful to the patient, but more assessment is needed to determine whether these interventions are appropriate.

6. A patient presents with the following values: pH, 7.20; pO2, 106 mm Hg; pCO2, 35 mm Hg; and HCO3-, 11 mEq/L. These values are most consistent with a. uncompensated respiratory acidosis. c. uncompensated metabolic alkalosis. b. uncompensated metabolic acidosis. d. uncompensated respiratory alkalosis.

ANS: B The pH indicates acidosis, and the HCO3- is markedly decreased, indicating a metabolic disorder. Uncompensated metabolic acidosis values include a pH below 7.35, PACO2 of 35 to 45 mm Hg, and HCO3- above 22 mEq/L. Uncompensated respiratory acidosis values include a pH below 7.35, PACO2 above 45 mm Hg, and HCO3- of 22 to 26 mEq/L. Uncompensated respiratory alkalosis values include a pH above 7.45, PACO2 below 35 mm Hg, and HCO3- of 22 to 26 mEq/L. Uncompensated metabolic alkalosis values include a pH above 7.45, PACO2 of 35 to 45 mm Hg, and HCO3- above 26 mEq/L.

13. The nurse teaches a patient about pursed lip breathing. Which action by the patient would indicate to the nurse that further teaching is needed? a. The patient inhales slowly through the nose. b. The patient puffs up the cheeks while exhaling. c. The patient practices by blowing through a straw. d. The patient's ratio of inhalation to exhalation is 1:3.

ANS: B The patient should relax the facial muscles without puffing the cheeks while doing pursed lip breathing. The other actions by the patient indicate a good understanding of pursed lip breathing.

4. The oxygen saturation (SpO2) for a patient with left lower lobe pneumonia is 90%. The patient has rhonchi, a weak cough effort, and complains of fatigue. Which action is a priority for the nurse to take? a. Position the patient on the left side. b. Assist the patient with staged coughing. c. Place a humidifier in the patient's room. d. Schedule a 2-hour rest period for the patient.

ANS: B The patient's assessment indicates that assisted coughing is needed to help remove secretions, which will improve oxygenation. A 2-hour rest period at this time may allow the oxygen saturation to drop further. Humidification will not be helpful unless the secretions can be mobilized. Positioning on the left side may cause a further decrease in oxygen saturation because perfusion will be directed more toward the more poorly ventilated lung.

40. The clinic nurse makes a follow-up telephone call to a patient with asthma. The patient reports having a baseline peak flow reading of 600 L/minute and the current peak flow is 420 L/minute. Which action should the nurse take first? a. Tell the patient to go to the hospital emergency department. b. Instruct the patient to use the prescribed albuterol (Proventil). c. Ask about recent exposure to any new allergens or asthma triggers. d. Question the patient about use of the prescribed inhaled corticosteroids.

ANS: B The patient's peak flow is 70% of normal, indicating a need for immediate use of short-acting β2-adrenergic SABA medications. Assessing for correct use of medications or exposure to allergens also is appropriate, but would not address the current decrease in peak flow. Because the patient is currently in the yellow zone, hospitalization is not needed.

15. A patient is admitted to the critical care unit with acute respiratory failure secondary to COPD. The patient has a 15-year history of emphysema and bronchitis. On inspection, the nurse observes that the patient is experiencing air trapping. While auscultating the chest, the nurse notes the presence of coarse, rumbling, low-pitched sounds in the right middle and lower lobes. On percussion of the lung fields, a patient with emphysema will predictably exhibit which tone? a. Resonance c. Tympany b. Hyperresonance d. Dullness

ANS: B The percussion tone of hyperresonance is heard with emphysema related to overinflation of the lung. Resonance can be found in normal lungs or with the diagnosis of bronchitis. Tympany occurs with the diagnosis of large pneumothorax and emphysematous blebs. Dullness occurs with the diagnosis of atelectasis, pleural effusion, pulmonary edema, pneumonia, and a lung mass.

11. The ventilator variable that causes inspiration is called the a. cycle. c. flow. b. trigger. d. pressure.

ANS: B The phase variable that initiates the change from exhalation to inspiration is called the trigger. Breaths may be pressure triggered or flow triggered based on the sensitivity setting of the ventilator and the patient's inspiratory effort or time triggered based on the rate setting of the ventilator.

37. Which nerve stimulates movement of the diaphragm? a. Musculocutaneous nerve c. Median nerve b. Phrenic nerve d. Axillary nerve

ANS: B The phrenic nerve arises from the cervical plexus through the fourth cervical nerve, with secondary contributions by the third and fifth cervical nerves. The other nerves control use and feeling of the arms.

18. A patient was admitted to the critical care unit with acute respiratory failure. The patient has been on a ventilator for 3 days and is being considered for weaning. The ventilator high-pressure alarm keeps going off. When you enter the room, the ventilator inoperative alarm sounds. Which of the following criteria would indicate that the patient is ready to be weaned? a. FiO2 greater than 50% b. Rapid shallow breathing index less than 105 c. Minute ventilation greater than 10 L/min d. Vital capacity/kg greater than or equal to 15 mL

ANS: B The rapid shallow breathing index (RSBI) has been shown to be predictive of weaning success. To calculate the RSBI, the patient's respiratory rate and minute ventilation are measured for 1 minute during spontaneous breathing. The measured respiratory rate is then divided by the tidal volume (expressed in liters). An RSBI less than 105 is considered predictive of weaning success. If the patient meets criteria for weaning readiness and has an RSBI less than 105, a spontaneous breathing trial can be performed.

5. The nurse develops a plan of care to prevent aspiration in a high-risk patient. Which nursing action will be most effective? a. Turn and reposition immobile patients at least every 2 hours. b. Place patients with altered consciousness in side-lying positions. c. Monitor for respiratory symptoms in patients who are immunosuppressed. d. Insert nasogastric tube for feedings for patients with swallowing problems.

ANS: B The risk for aspiration is decreased when patients with a decreased level of consciousness are placed in a side-lying or upright position. Frequent turning prevents pooling of secretions in immobilized patients but will not decrease the risk for aspiration in patients at risk. Monitoring of parameters such as breath sounds and oxygen saturation will help detect pneumonia in immunocompromised patients, but it will not decrease the risk for aspiration. Conditions that increase the risk of aspiration include decreased level of consciousness (e.g., seizure, anesthesia, head injury, stroke, alcohol intake), difficulty swallowing, and nasogastric intubation with or without tube feeding. With loss of consciousness, the gag and cough reflexes are depressed, and aspiration is more likely to occur. Other high-risk groups are those who are seriously ill, have poor dentition, or are receiving acid-reducing medications.

9. A nurse is caring for a patient with ARDS who is being treated with mechanical ventilation and high levels of positive end-expiratory pressure (PEEP). Which assessment finding by the nurse indicates that the PEEP may need to be reduced? a. The patient's PaO2 is 50 mm Hg and the SaO2 is 88%. b. The patient has subcutaneous emphysema on the upper thorax. c. The patient has bronchial breath sounds in both the lung fields. d. The patient has a first-degree atrioventricular heart block with a rate of 58.

ANS: B The subcutaneous emphysema indicates barotrauma caused by positive pressure ventilation and PEEP. Bradycardia, hypoxemia, and bronchial breath sounds are all concerns and will need to be addressed, but they are not specific indications that PEEP should be reduced.

36. Which pleura adheres to the lungs? a. Parietal c. Intrapleural b. Visceral d. Surfactant

ANS: B The visceral pleura adheres to the lungs, extending onto the hilar bronchi and into the major fissures. The parietal pleura lines the inner surface of the chest wall and mediastinum. The pleural space has a pressure within it called the intrapleural pressure. Surfactant is responsible for preventing the alveoli from completely collapsing on exhalation.

26. The therapeutic blood level for theophylline (Xanthines) is a. 5 to 10 mg/dL. c. 20 to 30 mg/dL. b. 10 to 20 mg/dL. d. 35 to 45 mg/dL.

ANS: B Therapeutic blood level for theophylline is 10 to 20 mg/dL.

29. Which instruction should the nurse include in an exercise teaching plan for a patient with chronic obstructive pulmonary disease (COPD)? a. "Stop exercising if you start to feel short of breath." b. "Use the bronchodilator before you start to exercise." c. "Breathe in and out through the mouth while you exercise." d. "Upper body exercise should be avoided to prevent dyspnea."

ANS: B Use of a bronchodilator before exercise improves airflow for some patients and is recommended. Shortness of breath is normal with exercise and not a reason to stop. Patients should be taught to breathe in through the nose and out through the mouth (using a pursed lip technique). Upper-body exercise can improve the mechanics of breathing in patients with COPD.

1. A patient with acute shortness of breath is admitted to the hospital. Which action should the nurse take during the initial assessment of the patient? a. Ask the patient to lie down to complete a full physical assessment. b. Briefly ask specific questions about this episode of respiratory distress. c. Complete the admission database to check for allergies before treatment. d. Delay the physical assessment to first complete pulmonary function tests

ANS: B When a patient has severe respiratory distress, only information pertinent to the current episode is obtained, and a more thorough assessment is deferred until later. Obtaining a comprehensive health history or full physical examination is unnecessary until the acute distress has resolved. Brief questioning and a focused physical assessment should be done rapidly to help determine the cause of the distress and suggest treatment. Checking for allergies is important, but it is not appropriate to complete the entire admission database at this time. The initial respiratory assessment must be completed before any diagnostic tests or interventions can be ordered.

28. A patient with a pleural effusion is scheduled for a thoracentesis. Which action should the nurse take to prepare the patient for the procedure? a. Start a peripheral IV line to administer the necessary sedative drugs. b. Position the patient sitting upright on the edge of the bed and leaning forward. c. Obtain a large collection device to hold 2 to 3 liters of pleural fluid at one time. d. Remove the water pitcher and remind the patient not to eat or drink anything for 6 hours.

ANS: B When the patient is sitting up, fluid accumulates in the pleural space at the lung bases and can more easily be located and removed. The patient does not usually require sedation for the procedure, and there are no restrictions on oral intake because the patient is not sedated or unconscious. Usually only 1000 to 1200 mL of pleural fluid is removed at one time. Rapid removal of a large volume can result in hypotension, hypoxemia, or pulmonary edema.

1. A patient is scheduled for a computed tomography (CT) of the chest with contrast media. Which assessment findings should the nurse immediately report to the health care provider (select all that apply)? a. Patient is claustrophobic. b. Patient is allergic to shellfish. c. Patient recently used a bronchodilator inhaler. d. Patient is not able to remove a wedding band. e. Blood urea nitrogen (BUN) and serum creatinine levels are elevated.

ANS: B,E Because the contrast media is iodine-based and may cause dehydration and decreased renal blood flow, asking about iodine allergies (such as allergy to shellfish) and monitoring renal function before the CT scan are necessary. The other actions are not contraindications for CT of the chest, although they may be for other diagnostic tests, such as magnetic resonance imaging (MRI) or pulmonary function testing (PFT).

3. A patient is scheduled for pulmonary function testing. Which action should the nurse take to prepare the patient for this procedure? a. Give the rescue medication immediately before testing. b. Administer oral corticosteroids 2 hours before the procedure. c. Withhold bronchodilators for 6 to 12 hours before the examination. d. Ensure that the patient has been NPO for several hours before the test.

ANS: C Bronchodilators are held before pulmonary function testing (PFT) so that a baseline assessment of airway function can be determined. Testing is repeated after bronchodilator use to determine whether the decrease in lung function is reversible. There is no need for the patient to be NPO. Oral corticosteroids should be held before PFTs. Rescue medications (which are bronchodilators) would not be given until after the baseline pulmonary function was assessed.

15. Which of the following V/Q ratios would most suggest intrapulmonary shunting? a. 0.8 c. 0.4 b. 2.2 d. V/Q ratios are not related to shunting

ANS: C A V/Q ratio of 4:5 or 0.8 is considered normal. A V/Q less than 0.8 is considered shunt producing, and a V/Q greater than 0.8 is considered dead space producing.

11. Which of the following findings confirms the diagnosis of a PE? a. Low-probability V/Q scan b. Negative pulmonary angiogram c. High-probability V/Q scan d. Absence of vascular markings on the chest radiograph

ANS: C A definitive diagnosis of a pulmonary embolus requires confirmation by a high-probability V/Q scan, an abnormal pulmonary angiogram or computed tomography scan, or strong clinical suspicion coupled with abnormal findings on lower extremity deep venous thrombosis studies.

12. The nurse interviews a patient with a new diagnosis of chronic obstructive pulmonary disease (COPD). Which information is most helpful in confirming a diagnosis of chronic bronchitis? a. The patient tells the nurse about a family history of bronchitis. b. The patient's history indicates a 30 pack-year cigarette history. c. The patient complains about a productive cough every winter for 3 months. d. The patient denies having any respiratory problems until the last 12 months.

ANS: C A diagnosis of chronic bronchitis is based on a history of having a productive cough for 3 months for at least 2 consecutive years. There is no family tendency for chronic bronchitis. Although smoking is the major risk factor for chronic bronchitis, a smoking history does not confirm the diagnosis.

40. The nurse notes that a patient has incisional pain, a poor cough effort, and scattered rhonchi after a thoracotomy. Which action should the nurse take first? a. Assist the patient to sit upright in a chair. b. Splint the patient's chest during coughing. c. Medicate the patient with prescribed morphine. d. Observe the patient use the incentive spirometer.

ANS: C A major reason for atelectasis and poor airway clearance in patients after chest surgery is incisional pain (which increases with deep breathing and coughing). The first action by the nurse should be to medicate the patient to minimize incisional pain. The other actions are all appropriate ways to improve airway clearance but should be done after the morphine is given.

17. Which of the following ABG values represents uncompensated metabolic acidosis? a. pH, 7.29; PaCO2, 57 mm Hg; HCO3-, 22 mEq/L b. pH, 7.36; PaCO2, 33 mm Hg; HCO3-, 18 mEq/L c. pH, 7.22; PaCO2, 42 mm Hg; HCO3-, 18 mEq/L d. pH, 7.52; PaCO2, 38 mm Hg; HCO3-, 29 mEq/L

ANS: C A pH of 7.22 is below normal, reflecting acidosis. The metabolic component (HCO3-) is low, indicating that the acidosis is metabolic in origin. Uncompensated metabolic acidosis values include a pH below 7.35, PaCO2 of 35 to 45 mm Hg, and HCO3- below 22 mEq/L

25. Indications to support a pneumonectomy are a. lesions confined to a single lobe. c. unilateral tuberculosis. b. bronchiectasis. d. lung abscesses or cyst.

ANS: C A pneumonectomy is the removal of entire lung with or without resection of the mediastinal lymph nodes. Indications include malignant lesions, unilateral tuberculosis, extensive unilateral bronchiectasis, multiple lung abscesses, massive hemoptysis, and bronchopleural fistula.

13. A pneumothorax greater than 15% requires a. systemic antibiotics to treat the inflammatory response. b. an occlusive dressing to equalize lung pressures. c. interventions to evacuate the air from the pleural space and facilitate re-expansion of the collapsed lung. d. mechanical ventilation to assist with re-expansion of the collapsed lung.

ANS: C A pneumothorax greater than 15% requires intervention to evacuate the air from the pleural space and facilitate re-expansion of the collapsed lung. Interventions include aspiration of the air with a needle and placement of a small-bore (12-20 Fr) or large-bore (24-40 Fr) chest tube.

26. A patient is admitted with acute respiratory failure attributable to pneumonia. Smoking history reveals that the patient smoked two packs of cigarettes a day for 25 years, stopping 10 years ago. ABG values on the current ventilator settings are pH, 7.37; PaCO2, 50 mm Hg; and HCO3-, 27 mEq/L. Chest radiograph reveals a large right pleural effusion. Intrapulmonary shunting value of 35% indicates a. normal gas exchange of venous blood. b. an abnormal finding indicative of a shunt-producing disorder. c. a serious and potentially life-threatening condition. d. metabolic alkalosis.

ANS: C A shunt greater than 10% is considered abnormal and indicative of a shunt-producing disorder. A shunt greater than 30% is a serious and potentially life-threatening condition that requires pulmonary intervention.

15. A patient was admitted to the critical care unit after a left pneumonectomy. The patient is receiving 40% oxygen via a simple facemask. The morning chest radiography study reveals right lower lobe pneumonia. After eating breakfast, the patient suddenly vomits and aspirates. Which test would the nurse expect the health care provider to order to identify the infectious pathogen? a. CBC with differential c. Sputum Gram stain and culture b. Wound culture of surgical site d. Urine specimen

ANS: C A sputum Gram stain and culture are done to facilitate the identification of the infectious pathogen. In 50% of cases, though, a causative agent is not identified. A diagnostic bronchoscopy may be needed, particularly if the diagnosis is unclear or current therapy is not working. In addition, a complete blood count with differential, chemistry panel, blood cultures, and arterial blood gas analysis is obtained.

25. During change-of-shift report on a medical unit, the nurse learns that a patient with aspiration pneumonia who was admitted with respiratory distress has become increasingly agitated. Which action should the nurse take first? a. Give the prescribed PRN sedative drug. b. Offer reassurance and reorient the patient. c. Use pulse oximetry to check the oxygen saturation. d. Notify the health care provider about the patient's status.

ANS: C Agitation may be an early indicator of hypoxemia. The other actions may also be appropriate, depending on the findings about oxygen saturation.

17. A patient is admitted to the critical care unit with acute respiratory failure secondary to COPD. The patient has a 15-year history of emphysema and bronchitis. On inspection, the nurse notes that the patient is experiencing air trapping. While auscultating his chest, you note the presence of coarse, rumbling, low-pitched sounds in the right middle and lower lobes. Which of the following best describes the patient's breathing pattern? a. Deep sighing breaths without pauses b. Rapid, shallow breaths c. Normal breathing pattern interspersed with forced expirations d. Irregular breathing pattern with both deep and shallow breaths

ANS: C Air trapping is described as a normal breathing pattern interspersed with forced expirations. As the patient breathes, air becomes trapped in the lungs, and ventilations become progressively shallower until the patient actively and forcefully exhales.

7. Long-term ventilator management over 21 days is best handled through use of a(n) a. oropharyngeal airway. c. tracheostomy tube. b. esophageal obturator airway. d. endotracheal intubation.

ANS: C Although no ideal time to perform the procedure has been identified, it is commonly accepted that if a patient has been intubated or is anticipated to be intubated for longer than 7 to 10 days, a tracheostomy should be performed.

17. A patient in metabolic alkalosis is admitted to the emergency department, and pulse oximetry (SpO2) indicates that the O2 saturation is 94%. Which action should the nurse take next? a. Administer bicarbonate. b. Complete a head-to-toe assessment. c. Place the patient on high-flow oxygen. d. Obtain repeat arterial blood gases (ABGs).

ANS: C Although the O2 saturation is adequate, the left shift in the oxyhemoglobin dissociation curve will decrease the amount of oxygen delivered to tissues, so high oxygen concentrations should be given. Bicarbonate would worsen the patient's condition. A head-to-toe assessment and repeat ABGs may be implemented. However, the priority intervention is to give high-flow oxygen.

1. To evaluate the effectiveness of ordered interventions for a patient with ventilatory failure, which diagnostic test will be most useful to the nurse? a. Chest x-ray b. Oxygen saturation c. Arterial blood gas analysis d. Central venous pressure monitoring

ANS: C Arterial blood gas (ABG) analysis is most useful in this setting because ventilatory failure causes problems with CO2 retention, and ABGs provide information about the PaCO2 and pH. The other tests may also be done to help in assessing oxygenation or determining the cause of the patient's ventilatory failure.

20. The nurse notes thick, white secretions in the endotracheal tube (ET) of a patient who is receiving mechanical ventilation. Which intervention will be most effective in addressing this problem? a. Increase suctioning to every hour. b. Reposition the patient every 1 to 2 hours. c. Add additional water to the patient's enteral feedings. d. Instill 5 mL of sterile saline into the ET before suctioning.

ANS: C Because the patient's secretions are thick, better hydration is indicated. Suctioning every hour without any specific evidence for the need will increase the incidence of mucosal trauma and would not address the etiology of the ineffective airway clearance. Instillation of saline does not liquefy secretions and may decrease the SpO2. Repositioning the patient is appropriate but will not decrease the thickness of secretions.

9. The nurse is caring for a patient with chronic obstructive pulmonary disease (COPD). Which information obtained from the patient would prompt the nurse to consult with the health care provider before administering the prescribed theophylline? a. The patient reports a recent 15-pound weight gain. b. The patient denies any shortness of breath at present. c. The patient takes cimetidine (Tagamet) 150 mg daily. d. The patient complains about coughing up green mucus.

ANS: C Cimetidine interferes with the metabolism of theophylline, and concomitant administration may lead rapidly to theophylline toxicity. The other patient information would not affect whether the theophylline should be administered or not.

24. A patient who has a right-sided chest tube following a thoracotomy has continuous bubbling in the suction-control chamber of the collection device. Which action by the nurse is most appropriate? a. Document the presence of a large air leak. b. Notify the surgeon of a possible pneumothorax. c. Take no further action with the collection device. d. Adjust the dial on the wall regulator to decrease suction.

ANS: C Continuous bubbling is expected in the suction-control chamber and indicates that the suction-control chamber is connected to suction. An air leak would be detected in the water-seal chamber. There is no evidence of pneumothorax. Increasing or decreasing the vacuum source will not adjust the suction pressure. The amount of suction applied is regulated by the amount of water in this chamber and not by the amount of suction applied to the system.

43. A patient has acute bronchitis with a nonproductive cough and wheezes. Which topic should the nurse plan to include in the teaching plan? a. Purpose of antibiotic therapy b. Ways to limit oral fluid intake c. Appropriate use of cough suppressants d. Safety concerns with home oxygen therapy

ANS: C Cough suppressants are frequently prescribed for acute bronchitis. Because most acute bronchitis is viral in origin, antibiotics are not prescribed unless there are systemic symptoms. Fluid intake is encouraged. Home oxygen is not prescribed for acute bronchitis, although it may be used for chronic bronchitis.

4. Use of oxygen therapy in the patient who is hypercapnic may result in a. oxygen toxicity. c. carbon dioxide retention. b. absorption atelectasis. d. pneumothorax.

ANS: C Deoxygenated hemoglobin carries more CO2 compared with oxygenated hemoglobin. Administration of oxygen increases the proportion of oxygenated hemoglobin, which causes increased release of CO2 at the lung level. Because of the risk of CO2 accumulation, all patients who are chronically hypercapnic require careful low-flow oxygen administration.

14. A patient is admitted to the critical care unit with acute respiratory failure secondary to chronic obstructive pulmonary disease. The patient has a 15-year history of emphysema and bronchitis. On inspection, the nurse observes that the patient is experiencing air trapping. While auscultating the chest, the nurse notes the presence of coarse, rumbling, low-pitched sounds in the right middle and lower lobes. On further inspection of the patient, the nurse observes that his fingers appear discolored. This is a result of a. clubbing. c. peripheral cyanosis. b. central cyanosis. d. chronic tuberculosis.

ANS: C Discoloration of the fingers is an indication of peripheral cyanosis. Central cyanosis occurs when the unsaturated hemoglobin of arterial blood exceeds 5 g/dL and is considered a life-threatening situation. Clubbing refers to an abnormality of the fingers caused by chronically low blood levels of oxygen often related to a heart or lung disease.

33. A patient is diagnosed with both human immunodeficiency virus (HIV) and active tuberculosis (TB) disease. Which information obtained by the nurse is most important to communicate to the health care provider? a. The Mantoux test had an induration of 7 mm. b. The chest-x-ray showed infiltrates in the lower lobes. c. The patient is being treated with antiretrovirals for HIV infection. d. The patient has a cough that is productive of blood-tinged mucus.

ANS: C Drug interactions can occur between the antiretrovirals used to treat HIV infection and the medications used to treat TB. The other data are expected in a patient with HIV and TB.

11. A patient with chronic obstructive pulmonary disease (COPD) has a nursing diagnosis of imbalanced nutrition: less than body requirements. Which intervention would be most appropriate for the nurse to include in the plan of care? a. Encourage increased intake of whole grains. b. Increase the patient's intake of fruits and fruit juices. c. Offer high-calorie snacks between meals and at bedtime. d. Assist the patient in choosing foods with high vegetable and mineral content.

ANS: C Eating small amounts more frequently (as occurs with snacking) will increase caloric intake by decreasing the fatigue and feelings of fullness associated with large meals. Patients with COPD should rest before meals. Foods that have a lot of texture like whole grains may take more energy to eat and get absorbed and lead to decreased intake. Although fruits, juices, and vegetables are not contraindicated, foods high in protein are a better choice.

7. The nurse completes a shift assessment on a patient admitted in the early phase of heart failure. When auscultating the patient's lungs, which finding would the nurse most likely hear? a. Continuous rumbling, snoring, or rattling sounds mainly on expiration b. Continuous high-pitched musical sounds on inspiration and expiration c. Discontinuous, high-pitched sounds of short duration heard on inspiration d. A series of long-duration, discontinuous, low-pitched sounds during inspiration

ANS: C Fine crackles are likely to be heard in the early phase of heart failure. Fine crackles are discontinuous, high-pitched sounds of short duration heard on inspiration. Rhonchi are continuous rumbling, snoring, or rattling sounds mainly on expiration. Course crackles are a series of long-duration, discontinuous, low-pitched sounds during inspiration. Wheezes are continuous high-pitched musical sounds on inspiration and expiration.

9. The nurse teaches a patient about pulmonary function testing (PFT). Which statement, if made by the patient, indicates teaching was effective? a. "I will use my inhaler right before the test." b. "I won't eat or drink anything 8 hours before the test." c. "I should inhale deeply and blow out as hard as I can during the test." d. "My blood pressure and pulse will be checked every 15 minutes after the test."

ANS: C For PFT, the patient should inhale deeply and exhale as long, hard, and fast as possible. The other actions are not needed with PFT. The administration of inhaled bronchodilators should be avoided 6 hours before the procedure.

5. On assessment of a client, you note fremitus over the trachea but not in the lung periphery. You know that this most likely represents a. bilateral pleural effusion. c. a normal finding. b. bronchial obstruction. d. apical pneumothorax.

ANS: C Fremitus is described as normal, decreased, or increased. With normal fremitus, vibrations can be felt over the trachea but are barely palpable over the periphery. With decreased fremitus, there is interference with the transmission of vibrations. Examples of disorders that decrease fremitus include pleural effusion, pneumothorax, bronchial obstruction, pleural thickening, and emphysema.

25. The nurse provides preoperative instruction for a patient scheduled for a left pneumonectomy for cancer of the lung. Which information should the nurse include about the patient's postoperative care? a. Positioning on the right side b. Bed rest for the first 24 hours c. Frequent use of an incentive spirometer d. Chest tube placement with continuous drainage

ANS: C Frequent deep breathing and coughing are needed after chest surgery to prevent atelectasis. To promote gas exchange, patients after pneumonectomy are positioned on the surgical side. Early mobilization decreases the risk for postoperative complications such as pneumonia and deep vein thrombosis. In a pneumonectomy, chest tubes may or may not be placed in the space from which the lung was removed. If a chest tube is used, it is clamped and only released by the surgeon to adjust the volume of serosanguineous fluid that will fill the space vacated by the lung. If the cavity overfills, it could compress the remaining lung and compromise the cardiovascular and pulmonary function. Daily chest x-rays can be used to assess the volume and space.

9. Nursing interventions to limit the complications of suctioning include a. inserting the suction catheter no more than 5 inches. b. premedicating the patient with atropine. c. hyperoxygenating the patient with 100% oxygen. d. increasing the suction to 150 mm Hg.

ANS: C Hypoxemia can be minimized by giving the patient three hyperoxygenation breaths (breaths at 100% FiO2) with the ventilator before the procedure and again after each pass of the suction catheter.

1. Which of the following causes of hypoxemia is the result of blood passing through unventilated portions of the lungs? a. Alveolar hypoventilation c. Intrapulmonary shunting b. Dead space ventilation d. Drug overdose

ANS: C Hypoxemia is the result of impaired gas exchange and is the hallmark of acute respiratory failure. Hypercapnia may be present, depending on the underlying cause of the problem. The main causes of hypoxemia are alveolar hypoventilation, ventilation/perfusion (V/Q) mismatching, and intrapulmonary shunting. Intrapulmonary shunting occurs when blood passes through a portion of a lung that is not ventilated. Drug overdose is an extrapulmonary cause that affects the brain.

21. According to the National Association of Medical Direction of Respiratory Care Consensus Panel prolonged mechanical ventilation has been defined as a. "the need for >=30 consecutive days of mechanical ventilation for >=8 hours per day" b. "the need for >=15 consecutive days of mechanical ventilation for >=12 hours per day" c. "the need for >=21 consecutive days of mechanical ventilation for >=6 hours per day" d. "the need for >=40 consecutive days of mechanical ventilation for >=4 hours per day"

ANS: C In 2005, the National Association for Medical Direction of Respiratory Care Consensus Panel recommended that LTMVD (which they referred to as prolonged mechanical ventilation) be defined as "the need for >=21 consecutive days of mechanical ventilation for >=6 hours per day."

2. Which of the following is a passive event in a spontaneously breathing patient? a. Coughing c. Exhalation b. Inhalation d. Yawning

ANS: C Inhalation involves the contraction of the diaphragm, an active event, as do yawning and coughing. Exhalation in the healthy lung is a passive event requiring very little energy.

32. A patient newly diagnosed with asthma is being discharged. The nurse anticipates including which topic in the discharge teaching? a. b. Side effects of sustained-release theophylline Use of long-acting β-adrenergic medications c. Self-administration of inhaled corticosteroids d. Complications associated with oxygen therapy

ANS: C Inhaled corticosteroids are more effective in improving asthma than any other drug and are indicated for all patients with persistent asthma. The other therapies would not typically be first-line treatments for newly diagnosed asthma.

10. The nurse observes a student who is listening to a patient's lungs who is having no problems with breathing. Which action by the student indicates a need to review respiratory assessment skills? a. The student starts at the apices of the lungs and moves to the bases. b. The student compares breath sounds from side to side avoiding bony areas. c. The student places the stethoscope over the posterior chest and listens during inspiration. d. The student instructs the patient to breathe slowly and a little more deeply than normal through the mouth.

ANS: C Listening only during inspiration indicates the student needs a review of respiratory assessment skills. At each placement of the stethoscope, listen to at least one cycle of inspiration and expiration. During chest auscultation, instruct the patient to breathe slowly and a little deeper than normal through the mouth. Auscultation should proceed from the lung apices to the bases, comparing opposite areas of the chest, unless the patient is in respiratory distress or will tire easily. If so, start at the bases (see Fig. 26-7). Place the stethoscope over lung tissue, not over bony prominences.

19. Diaphragmatic excursion is a measurement of the difference in the level of the diaphragm on inspiration and expiration determined by percussion. It is increased in a. atelectasis and emphysema. c. atelectasis and paralysis. b. hepatomegaly and ascites. d. pneumonia and pneumothorax.

ANS: C Normal diaphragmatic excursion is 3 to 5 cm and is part of the percussion component of the physical examination. An assessment finding other than normal would indicate the need for further evaluation such as chest radiographic examination.

6. Normal anteroposterior (AP) diameter ranges from 1:2 to 5:7. An increase in AP diameter of the chest that is characterized by displacement of the sternum forward and the ribs outward is indicative of a. a funnel chest. c. a barrel chest. b. a pigeon breast. d. Harrison's groove.

ANS: C Normal ratio of anteroposterior diameter to lateral diameter ranges from 1:2 to 5:7. A barrel chest is characterized by displacement of the sternum forward and the ribs outward and is suggestive of chronic obstructive pulmonary disease. Funnel chest, pectus excavatum, creates a pit-shaped depression. Pigeon chest, pectus carinatum, causes an increase in anteroposterior diameter. Both are related to restrictive pulmonary disease. Harrison's groove, a rib deformity, is a result of rickets.

10. A patient who is taking rifampin (Rifadin) for tuberculosis calls the clinic and reports having orange discolored urine and tears. Which is the best response by the nurse? a. Ask if the patient is experiencing shortness of breath, hives, or itching. b. Ask the patient about any visual abnormalities such as red-green color discrimination. c. Explain that orange discolored urine and tears are normal while taking this medication. d. Advise the patient to stop the drug and report the symptoms to the health care provider.

ANS: C Orange-colored body secretions are a side effect of rifampin. The patient does not have to stop taking the medication. The findings are not indicative of an allergic reaction. Alterations in red-green color discrimination commonly occurs when taking ethambutol (Myambutol), which is a different TB medication.

5. When caring for a patient with pulmonary hypertension, which parameter is most appropriate for the nurse to monitor to evaluate the effectiveness of the treatment? a. Central venous pressure (CVP) b. Systemic vascular resistance (SVR) c. Pulmonary vascular resistance (PVR) d. Pulmonary artery wedge pressure (PAWP)

ANS: C PVR is a major contributor to pulmonary hypertension, and a decrease would indicate that pulmonary hypertension was improving. The other parameters also may be monitored but do not directly assess for pulmonary hypertension.

18. The two most common causes of hospital-acquired pneumonia in the United States are a. Staphylococcus aureus and Pseudomonas aeruginosa b. Escherichia coli and Haemophilus influenzae c. methicillin-resistant Staphylococcus aureus and Pseudomonas aeruginosa d. Klebsiella spp. and Enterobacter spp.

ANS: C Pathogens that can cause health care-associated pneumonia are similar to those causing both community- and hospital-acquired pneumonia (HAP) with Pseudomonas aeruginosa and methicillin-resistant Staphylococcus aureus (MRSA) being the most common in the United States. Pathogens that can cause HAP include Escherichia coli, Haemophilus influenzae, methicillin-sensitive Staphylococcus aureus, Streptococcus pneumoniae, P. aeruginosa, Acinetobacter baumannii, MRSA, Klebsiella spp., and Enterobacter spp.

22. A patient with chronic obstructive pulmonary disease (COPD) has poor gas exchange. Which action by the nurse would be most appropriate? a. Have the patient rest in bed with the head elevated to 15 to 20 degrees. b. Ask the patient to rest in bed in a high-Fowler's position with the knees flexed. c. Encourage the patient to sit up at the bedside in a chair and lean slightly forward. d. Place the patient in the Trendelenburg position with several pillows behind the head.

ANS: C Patients with COPD improve the mechanics of breathing by sitting up in the "tripod" position. Resting in bed with the head elevated in a semi-Fowler's position would be an alternative position if the patient was confined to bed, but sitting in a chair allows better ventilation. The Trendelenburg position or sitting upright in bed with the knees flexed would decrease the patient's ability to ventilate well.

22. The creation of sound waves across the body surface to determine abnormal densities is known as a. inspection. c. percussion. b. palpation. d. auscultation.

ANS: C Percussion is the process of creating sound waves on the surface of the body to determine abnormal density of any underlying areas. Palpation is the process of touching the patient to judge the size, shape, texture, and temperature of the body surface or underlying structures. Inspection is the process of looking intently at the patient. Auscultation is the process of concentrated listening with a stethoscope to determine characteristics of body functions.

21. Preprocedural medications for a diagnostic bronchoscopy may include a. aspirin for anticoagulation. b. vecuronium to inhibit breathing. c. codeine to decrease the cough reflex. d. cimetidine to decrease hydrochloric acid secretion.

ANS: C Preprocedural medications for a diagnostic bronchoscopy may include atropine and intramuscular codeine. Whereas atropine lessens the vasovagal response and reduces the secretions, codeine decreases the cough reflex. When a bronchoscopy is performed therapeutically to remove secretions, decreased cough and gag reflexes are present, which may impair secretion clearance.

16. An occupational health nurse works at a manufacturing plant where there is potential exposure to inhaled dust. Which action, if recommended by the nurse, will be most helpful in reducing the incidence of lung disease? a. Treat workers with pulmonary fibrosis. b. Teach about symptoms of lung disease. c. Require the use of protective equipment. d. Monitor workers for coughing and wheezing.

ANS: C Prevention of lung disease requires the use of appropriate protective equipment such as masks. The other actions will help in recognition or early treatment of lung disease but will not be effective in prevention of lung damage. Repeated exposure eventually results in diffuse pulmonary fibrosis. Fibrosis is the result of tissue repair after inflammation.

8. A patient develops increasing dyspnea and hypoxemia 2 days after heart surgery. To determine whether the patient has acute respiratory distress syndrome (ARDS) or pulmonary edema caused by heart failure, the nurse will plan to assist with a. obtaining a ventilation-perfusion scan. b. drawing blood for arterial blood gases. c. insertion of a pulmonary artery catheter. d. positioning the patient for a chest x-ray.

ANS: C Pulmonary artery wedge pressures are normal in the patient with ARDS because the fluid in the alveoli is caused by increased permeability of the alveolar-capillary membrane rather than by the backup of fluid from the lungs (as occurs in cardiogenic pulmonary edema). The other tests will not help in differentiating cardiogenic from noncardiogenic pulmonary edema.

10. A 75-kg, 5 foot, 8 inch patient is on a ventilator. The physician or nurse practitioner states the patient may be ready for extubation. Orders for a negative inspiratory pressure (NIP) and force ventilatory capacity (FVC) tests are written. Which of the following results best suggests that the patient is ready for extubation? a. NIP, -10; FVC, 4600 c. NIP, -22; FVC, 4400 b. NIP, -18; FVC, 4700 d. NIP, -24; FVC, 4800

ANS: D These values indicate that the patient is taking a strong enough and deep enough inspiration to have the best chance of tolerating extubation.

32. A patient is admitted to the critical care unit with an acute exacerbation of chronic emphysema. Respirations are 28 breaths/min. A pulmonary artery catheter is in place. Pulmonary artery systolic (PAS) is 38 mm Hg with a pulmonary artery mean (PAM) of 22 mm Hg. Blood gases reveal an uncompensated respiratory acidosis. Based on the readings obtained from the pulmonary artery catheter, the patient shows evidence of which of the following? a. Physiologic shunting c. Pulmonary hypertension b. Widespread atelectasis d. Ventilation/perfusion mismatch

ANS: C Pulmonary hypertension is defined as increased pressure (PAS greater than 35 mm Hg and PAM greater than 25 mm Hg at rest or greater than 30 mm Hg with exertion) within the pulmonary arterial system. It occurs when the cross-sectional area of the pulmonary bed decreases as a result of vasoconstriction or structural changes in the vascular bed.

12. Which of the following is considered physiologic dead space? a. Respiratory bronchiole and unperfused alveoli b. Trachea and perfused alveoli c. Trachea and unperfused alveoli d. Trachea and mainstem bronchi

ANS: C Respiratory bronchioles participate in gas exchange. The areas in the lungs that are ventilated but in which no gas exchange occurs are known as dead space regions (trachea and mainstem bronchi). These unperfused alveoli are known as alveolar dead space. Anatomic dead space plus alveolar dead space is called physiologic dead space.

28. Which of the following factors will increase diffusion of gases across the alveolar capillary membrane? a. A decrease in surface area of the membrane b. An increase in thickness of the membrane c. An increase in the driving pressure of the gas d. A decrease in the solubility coefficient of the gas

ANS: C Several factors affect the rate of diffusion, including increasing the driving pressure of the gas.

7. The health care provider writes an order for bacteriologic testing for a patient who has a positive tuberculosis skin test. Which action should the nurse take? a. Teach about the reason for the blood tests. b. Schedule an appointment for a chest x-ray. c. Teach about the need to get sputum specimens for 2 to 3 consecutive days. d. Instruct the patient to expectorate three specimens as soon as possible.

ANS: C Sputum specimens are obtained on 2 to 3 consecutive days for bacteriologic testing for M. tuberculosis. The patient should not provide all the specimens at once. Blood cultures are not used for tuberculosis testing. A chest x-ray is not bacteriologic testing. Although the findings on chest x-ray examination are important, it is not possible to make a diagnosis of TB solely based on chest x-ray findings because other diseases can mimic the appearance of TB.

4. Lack of surfactant can cause which of the following conditions? a. Pulmonary embolus c. Pulmonary atelectasis b. Pulmonary hypertension d. Pulmonary edema

ANS: C Surfactant is responsible for preventing the alveoli from completely collapsing on exhalation. Lack of this lipoprotein allows the alveoli to collapse, producing atelectasis. Lack of surfactant is not responsible for the other conditions.

23. A nurse is weaning a 68-kg male patient who has chronic obstructive pulmonary disease (COPD) from mechanical ventilation. Which patient assessment finding indicates that the weaning protocol should be stopped? a. The patient's heart rate is 97 beats/min. b. The patient's oxygen saturation is 93%. c. The patient respiratory rate is 32 breaths/min. d. The patient's spontaneous tidal volume is 450 mL.

ANS: C Tachypnea is a sign that the patient's work of breathing is too high to allow weaning to proceed. The patient's heart rate is within normal limits, although the nurse should continue to monitor it. An oxygen saturation of 93% is acceptable for a patient with COPD. A spontaneous tidal volume of 450 mL is within the acceptable range.

22. The Passy-Muir valve is contraindicated in patients a. who are trying to relearn normal breathing patterns. b. who experience low secretions. c. with laryngeal or pharyngeal dysfunction. d. who want to speak while on the ventilator.

ANS: C The Passy-Muir valve is contraindicated in patients with laryngeal or pharyngeal dysfunction, excessive secretions, or poor lung compliance.

4. A patient presents with chest trauma from an MVA. Upon assessment, the nurse documents that the patient is complaining of dyspnea, shortness of breath, tachypnea, and tracheal deviation to the right. In addition, the client's tongue is blue-gray. Based on the following data, what would the nurse would expect to find? a. PaO2 of 88 and PCO2 of 55 b. Absent breath sounds in all right lung fields c. Absent breath sounds in all left lung fields d. Diminished breath sounds in all fields

ANS: C The clinical picture described is most consistent with left pneumothorax. This would cause the trachea to deviate to the right, away from the increasing pressure of the left. A pneumothorax this severe would completely collapse the right lung, thus causing absent breath sounds in that lung.

8. While caring for a patient with respiratory disease, the nurse observes that the patient's SpO2 drops from 93% to 88% while the patient is ambulating in the hallway. What is the priority action of the nurse? a. Notify the health care provider. b. Document the response to exercise. c. Administer the PRN supplemental O2. d. Encourage the patient to pace activity.

ANS: C The drop in SpO2 to 85% indicates that the patient is hypoxemic and needs supplemental oxygen when exercising. The other actions are also important, but the first action should be to correct the hypoxemia.

41. The nurse is caring for a patient with idiopathic pulmonary arterial hypertension (IPAH) who is receiving epoprostenol (Flolan). Which assessment information requires the most immediate action by the nurse? a. The oxygen saturation is 94%. b. The blood pressure is 98/56 mm Hg. c. The patient's central IV line is disconnected. d. The international normalized ratio (INR) is prolonged.

ANS: C The half-life of this drug is 6 minutes, so the nurse will need to restart the infusion as soon as possible to prevent rapid clinical deterioration. The other data also indicate a need for ongoing monitoring or intervention, but the priority action is to reconnect the infusion.

19. Which assessment finding obtained by the nurse when caring for a patient receiving mechanical ventilation indicates the need for suctioning? a. The patient's oxygen saturation is 93%. b. The patient was last suctioned 6 hours ago. c. The patient's respiratory rate is 32 breaths/minute. d. The patient has occasional audible expiratory wheezes.

ANS: C The increase in respiratory rate indicates that the patient may have decreased airway clearance and requires suctioning. Suctioning is done when patient assessment data indicate that it is needed, not on a scheduled basis. Occasional expiratory wheezes do not indicate poor airway clearance, and suctioning the patient may induce bronchospasm and increase wheezing. An oxygen saturation of 93% is acceptable and does not suggest that immediate suctioning is needed.

44. Which action by the nurse will be most effective in decreasing the spread of pertussis in a community setting? a. Providing supportive care to patients diagnosed with pertussis b. Teaching family members about the need for careful hand washing c. Teaching patients about the need for adult pertussis immunizations d. Encouraging patients to complete the prescribed course of antibiotics

ANS: C The increased rate of pertussis in adults is thought to be due to decreasing immunity after childhood immunization. Immunization is the most effective method of protecting communities from infectious diseases. Hand washing should be taught, but pertussis is spread by droplets and contact with secretions. Supportive care does not shorten the course of the disease or the risk for transmission. Taking antibiotics as prescribed does assist with decreased transmission, but patients are likely to have already transmitted the disease by the time the diagnosis is made.

7. A patient is admitted to the unit in respiratory distress secondary to pneumonia. The nurse knows that obtaining a history is very important. What is the appropriate intervention at this time for obtaining this data? a. Collect an overview of past medical history, present history, and current health status. b. Do not obtain any history at this time. c. Curtail the history to just a few questions about the client's chief complaint and precipitating events. d. Complete the history and then provide measures to assist the client to breathe easier.

ANS: C The initial presentation of the client determines the rapidity and direction for the interview. For a client in acute distress, the history should be curtailed to just a few questions about the client's chief complaint and the precipitating events.

35. The lobes are divided into 18 segments. How many are on the right lung? a. 3 c. 10 b. 8 d. 15

ANS: C The lobes are divided into 18 segments, each of which has its own bronchus branching immediately off a lobar bronchus. Ten segments are located in the right lung and eight in the left lung.

19. Most of the work of breathing is done by the a. pleura. c. diaphragm. b. intercostal muscles. d. sternocleidomastoid.

ANS: C The main muscle of inhalation is the diaphragm. It is connected to the sternum, ribs, and vertebrae. During normal, quiet breathing, the diaphragm does approximately 80% of the work of breathing. The most important of these are the external intercostal muscles, which elevate the ribs and expand the chest cage outward. The scalene, anterior serratus, and sternocleidomastoid muscles also participate to elevate the first two ribs and sternum.

6. A patient with right lower-lobe pneumonia has been treated with IV antibiotics for 3 days. Which assessment data obtained by the nurse indicates that the treatment has been effective? a. Bronchial breath sounds are heard at the right base. b. The patient coughs up small amounts of green mucus. c. The patient's white blood cell (WBC) count is 9000/μL. d. Increased tactile fremitus is palpable over the right chest.

ANS: C The normal WBC count indicates that the antibiotics have been effective. All the other data suggest that a change in treatment is needed.

14. Employee health test results reveal a tuberculosis (TB) skin test of 16-mm induration and a negative chest x-ray for a staff nurse working on the pulmonary unit. The nurse has no symptoms of TB. Which information should the occupational health nurse plan to teach the staff nurse? a. Standard four-drug therapy for TB b. Need for annual repeat TB skin testing c. Use and side effects of isoniazid (INH) d. Bacille Calmette-Guérin (BCG) vaccine

ANS: C The nurse is considered to have a latent TB infection and should be treated with INH daily for 6 to 9 months. The four-drug therapy would be appropriate if the nurse had active TB. TB skin testing is not done for individuals who have already had a positive skin test. BCG vaccine is not used in the United States for TB and would not be helpful for this individual, who already has a TB infection.

31. The nurse notes that a patient's endotracheal tube (ET), which was at the 22-cm mark, is now at the 25-cm mark and the patient is anxious and restless. Which action should the nurse take next? a. Offer reassurance to the patient. b. Bag the patient at an FIO2 of 100%. c. Listen to the patient's breath sounds. d. Notify the patient's health care provider.

ANS: C The nurse should first determine whether the ET tube has been displaced into the right mainstem bronchus by listening for unilateral breath sounds. If so, assistance will be needed to reposition the tube immediately. The other actions are also appropriate, but detection and correction of tube malposition are the most critical actions.

On admission, a patient presents as follows: pH, 7.38; respiratory rate, 24 breaths/min, regular, pursed-lip breathing; PaO2, 66 mm Hg; heart rate, 112 beats/min, sinus tachycardia; PaCO2, 52 mm Hg; blood pressure, 110/68 mm Hg; HCO3-, 24 mEq/L; and SpO2, 90% on O2 2 L/min nasal cannula. These gases show a. uncompensated metabolic alkalosis. b. uncompensated respiratory acidosis. c. compensated metabolic acidosis. d. compensated respiratory alkalosis.

ANS: C The pH is closer to the acidic level, so the primary disorder is acidosis. Compensated metabolic acidosis values include a pH of 7.35 to 7.39, PACO2 below 35 mm Hg, and HCO3- below 22 mEq/L. Uncompensated respiratory acidosis values include a pH below 7.35, PACO2 above 45 mm Hg, and HCO3- of 22 to 26 mEq/L. Compensated respiratory alkalosis values include a pH of 7.41 to 7.45, PACO2 below 35 mm Hg, and HCO3-below 22 mEq/L. Uncompensated metabolic alkalosis values include a pH above 7.45, PACO2 of 35 to 45 mm Hg, and HCO3- above 26 mEq/L.

1. The nurse teaches a patient with chronic bronchitis about a new prescription for Advair Diskus (combined fluticasone and salmeterol). Which action by the patient would indicate to the nurse that teaching about medication administration has been successful? a. The patient shakes the device before use. b. The patient attaches a spacer to the Diskus. c. The patient rapidly inhales the medication. d. The patient performs huff coughing after inhalation.

ANS: C The patient should inhale the medication rapidly. Otherwise the dry particles will stick to the tongue and oral mucosa and not get inhaled into the lungs. Advair Diskus is a dry powder inhaler; shaking is not recommended. Spacers are not used with dry powder inhalers. Huff coughing is a technique to move mucus into larger airways to expectorate. The patient should not huff cough or exhale forcefully after taking Advair in order to keep the medication in the lungs.

20. Depending on the patient's risk for the recurrence of PE, a patient may be placed on warfarin for a. 1 to 3 months. c. 3 to 12 months. b. 3 to 6 months. d. 12 to 36 months.

ANS: C The patient should remain on warfarin for 3 to 12 months depending on his or her risk for thromboembolic disease.

12. The nurse documents the vital signs for a patient admitted 2 days ago with gram-negative sepsis: temperature 101.2° F, blood pressure 90/56 mm Hg, pulse 92, respirations 34. Which action should the nurse take next? a. Give the scheduled IV antibiotic. b. Give the PRN acetaminophen (Tylenol). c. Obtain oxygen saturation using pulse oximetry. d. Notify the health care provider of the patient's vital signs.

ANS: C The patient's increased respiratory rate in combination with the admission diagnosis of gram-negative sepsis indicates that acute respiratory distress syndrome (ARDS) may be developing. The nurse should check for hypoxemia, a hallmark of ARDS. The health care provider should be notified after further assessment of the patient. Giving the scheduled antibiotic and the PRN acetaminophen will also be done, but they are not the highest priority for a patient who may be developing ARDS.

6. A patient seen in the asthma clinic has recorded daily peak flows that are 75% of the baseline. Which action will the nurse plan to take next? a. Increase the dose of the leukotriene inhibitor. b. Teach the patient about the use of oral corticosteroids. c. Administer a bronchodilator and recheck the peak flow. d. Instruct the patient to keep the next scheduled follow-up appointment.

ANS: C The patient's peak flow reading indicates that the condition is worsening (yellow zone). The patient should take the bronchodilator and recheck the peak flow. Depending on whether the patient returns to the green zone, indicating well-controlled symptoms, the patient may be prescribed oral corticosteroids or a change in dosing of other medications. Keeping the next appointment is appropriate, but the patient also needs to be taught how to control symptoms now and use the bronchodilator.

7. Which of the following therapeutic measures would be the most effective in treating hypoxemia in the presence of intrapulmonary shunting associated with ARDS? a. Sedating the patient to blunt noxious stimuli b. Increasing the FiO2 on the ventilator c. Administering positive-end expiratory pressure (PEEP) d. Restricting fluids to 500 mL per shift

ANS: C The purpose of using positive-end expiratory pressure (PEEP) in a patient with acute respiratory distress syndrome is to improve oxygenation while reducing FiO2 to less toxic levels. PEEP has several positive effects on the lungs, including opening collapsed alveoli, stabilizing flooded alveoli, and increasing functional residual capacity. Thus, PEEP decreases intrapulmonary shunting and increases compliance.

7. Patients who have aspiration pneumonia present with right lower lobe involvement more often than with involvement of the left lower lobe. You know this is because a. the left mainstem bronchus angles down more than the right. b. more people are right-side dominant. c. the right mainstem bronchus angles down more than the left. d. the right mainstem bronchus is narrower than the left.

ANS: C The right bronchus is wider than the left and angles at 20 to 30 degrees from the midline. Because of this angulation and the forces of gravity, the most common site of aspiration of foreign objects is through the right mainstem bronchus into the lower lobe of the right lung.

20. Which of the following statements describes the left bronchus? a. The left bronchus has a slight angle of 20 to 30 degrees from the midline. b. The two mainstem bronchi are structurally and functionally similar. c. The left bronchus is slightly narrower. d. The bronchi are the end units of the bronchial tree.

ANS: C The two mainstem bronchi are structurally different. The right bronchus is wider and angles at 20 to 30 degrees from the midline. The right mainstem bronchus is the most common site of aspiration of foreign objects. The left bronchus is slightly narrower than the right, and because of its position above the heart, the left bronchus angles directly toward the left lung at approximately 45 to 55 degrees from the midline.

20. A patient is intubated, and sputum for culture and sensitivity is ordered. Which of the following is important for obtaining the best specimen? a. After the specimen is in the container, dilute thick secretions with sterile water. b. Apply suction when the catheter is advanced to obtain secretions from within the endotracheal tube. c. Do not apply suction while the catheter is being withdrawn because this can contaminate the sample with sputum left in the endotracheal tube. d. Do not clear the endotracheal tube of all local secretions before obtaining the specimen.

ANS: C To prevent contamination of secretions in the upper portion of the endotracheal tube, do not apply suction while the catheter is being withdrawn. Clear the endotracheal or tracheostomy tube for all local secretions, avoiding deep airway penetration. This will prevent contamination with upper airway flora. Do not dilute thick secretions with sterile water. This will compromise the specimen.

34. The nurse assesses a patient with a history of asthma. Which assessment finding indicates that the nurse should take immediate action? a. Pulse oximetry reading of 91% b. Respiratory rate of 26 breaths/minute c. Use of accessory muscles in breathing d. Peak expiratory flow rate of 240 L/minute

ANS: C Use of accessory muscle indicates that the patient is experiencing respiratory distress and rapid intervention is needed. The other data indicate the need for ongoing monitoring and assessment but do not suggest that immediate treatment is required.

24. The nurse is caring for a patient receiving a continuous norepinephrine (Levophed) IV infusion. Which patient assessment finding indicates that the infusion rate may need to be adjusted? a. Heart rate is 58 beats/minute. b. Mean arterial pressure (MAP) is 56 mm Hg. c. Systemic vascular resistance (SVR) is elevated. d. Pulmonary artery wedge pressure (PAWP) is low.

ANS: C Vasoconstrictors such as norepinephrine (Levophed) will increase SVR, and this will increase the work of the heart and decrease peripheral perfusion. The infusion rate may need to be decreased. Bradycardia, hypotension (MAP of 56 mm Hg), and low PAWP are not associated with norepinephrine infusion.

23. Oxygen saturation of left atrial blood is normally between 96% and 99%. What is the explanation for less than 100% saturation? a. As blood passes to the alveolar-capillary membrane, a predicted percentage of hemoglobin will not bind with oxygen. b. During normal respiration, the majority of alveoli are not expanded. c. Venous blood from the bronchial circulation is returned to the left atrium. d. A small amount of blood leaks from the right atrium to the left atrium with each ventricular contraction.

ANS: C Venous blood from the bronchial circulation returns directly into the left atrium. The mixing of venous blood decreases the saturation of left atrial blood to a range between 96% and 99%.

19. A patient was admitted to the critical care unit with acute respiratory failure. The patient has been on a ventilator for 3 days and is being considered for weaning. The ventilator high-pressure alarm keeps going off. When you enter the room, the ventilator inoperative alarm sounds. Which of the following criteria would indicate that the patient is not tolerating weaning? a. A decrease in heart rate from 92 to 80 beats/min b. An SpO2 of 92% c. An increase in respiratory rate from 22 to 38 breaths/min d. Spontaneous tidal volumes of 300 to 350 mL

ANS: C Weaning intolerance indicators include (1) a decrease in level of consciousness; (2) a systolic blood pressure increased or decreased by 20 mm Hg; (3) a diastolic blood pressure greater than 100 mm Hg; (4) a heart rate increased by 20 beats/min; (5) premature ventricular contractions greater than 6/min, couplets, or runs of ventricular tachycardia; (6) changes in ST segment (usually elevation); (7) a respiratory rate greater than 30 breaths/min or less than 10 breaths/min; (8) a respiratory rate increased by 10 breaths/min; (9) a spontaneous tidal volume less than 250 mL; (10) a PaCO2 increased by 5 to 8 mm Hg or pH less than 7.30; (11) an SpO2 less than 90%; (12) use of accessory muscles of ventilation; (13) complaints of dyspnea, fatigue, or pain; (14) paradoxical chest wall motion or chest abdominal asynchrony; (15) diaphoresis; and (16) severe agitation or anxiety unrelieved with reassurance.

34. Based on the oxyhemoglobin dissociation curve, respiratory acidosis will have which effect? a. A shifting of the curve to the left b. Increased oxygen saturation c. Enhanced oxygen delivery at the tissue level d. Hypothermia

ANS: C When the curve is shifted to the right, as occurs in acidosis, although the saturation is lower than expected, a right shift enhances oxygen delivery at the tissue level because hemoglobin unloads more readily.

31. The nurse takes an admission history on a patient with possible asthma who has new-onset wheezing and shortness of breath. Which information may indicate a need for a change in therapy? a. The patient has chronic inflammatory bowel disease. b. The patient has a history of pneumonia 6 months ago. c. The patient takes propranolol (Inderal) for hypertension. d. The patient uses acetaminophen (Tylenol) for headaches.

ANS: C β-Blockers such as propranolol can cause bronchospasm in some patients with asthma. The other information will be documented in the health history but does not indicate a need for a change in therapy.

2. The lymphatic system plays which of the following important roles? (Select all that apply.) a. Ridding lung tissue of excess CO2 b. Connecting the thebesian veins to the coronary circulation c. Removing fluid from the lungs d. Producing immune responses e. Removing cell debris from the lungs

ANS: C, D, E The lymphatic system in the lungs serves two purposes. As part of the immune system, it is responsible for removing foreign particles and cell debris from the lungs and for producing both antibody and cell-mediated immune responses. It also is responsible for removing fluid from the lungs and for keeping the alveoli clear.

24. A patient is admitted with signs and symptoms of a pulmonary embolus (PE). The diagnostic test most conclusive to determine this diagnosis is a(n) a. ABG. c. pulmonary function test. b. bronchoscopy. d. V/Q scan.

ANS: D A V/Q scan is the most conclusive test for a pulmonary embolus. Arterial blood gas analysis tests oxygen levels in the blood, bronchoscopy is to used view the bronchi, and pulmonary function tests are used to measure lung volume.

17. A patient with chronic obstructive pulmonary disease (COPD) arrives in the emergency department complaining of shortness of breath and dyspnea on minimal exertion. Which assessment finding by the nurse is most important to report to the health care provider? a. The patient has bibasilar lung crackles. b. The patient is sitting in the tripod position. c. The patient's respirations have decreased from 30 to 10 breaths/minute. d. The patient's pulse oximetry indicates an O2 saturation of 91%.

ANS: D A decrease in respiratory rate in a patient with respiratory distress suggests the onset of fatigue and a high risk for respiratory arrest. Therefore immediate action such as positive pressure ventilation is needed. Patients who are experiencing respiratory distress frequently sit in the tripod position because it decreases the work of breathing. Crackles in the lung bases may be the baseline for a patient with COPD. An oxygen saturation of 91% is common in patients with COPD and will provide adequate gas exchange and tissue oxygenation.

2. The nurse teaches a patient how to administer formoterol (Perforomist) through a nebulizer. Which action by the patient indicates good understanding of the teaching? a. The patient attaches a spacer before using the inhaler. b. The patient coughs vigorously after using the inhaler. c. The patient activates the inhaler at the onset of expiration. d. The patient removes the facial mask when misting has ceased.

ANS: D A nebulizer is used to administer aerosolized medication. A mist is seen when the medication is aerosolized, and when all of the medication has been used, the misting stops. The other options refer to inhaler use. Coughing vigorously after inhaling and activating the inhaler at the onset of expiration are both incorrect techniques when using an inhaler.

16. A patient is admitted to the critical care unit with acute respiratory failure. Upon auscultation, the health care provider hears creaking, leathery, coarse breath sounds in the lower anterolateral chest area during inspiration and expiration. The nurse suspects that the patient has a(n) a. emphysema. c. pulmonary fibrosis. b. atelectasis. d. pleural friction rub.

ANS: D A pleural friction rub is the result of irritated pleural surfaces rubbing together and is characterized by a leathery, dry, loud, coarse sound. A pleural friction rub is seen with pleural effusions or pleurisy and is not indicative of emphysema.

11. COPD produces increased work of breathing by which of the following mechanisms? a. Decreased lung compliance c. Increased atelectasis b. Decreased chest wall compliance d.

ANS: D All of the causes may be present in a COPD patient with other diagnoses. However, COPD increases airway resistance because of obstructions

15. The nurse analyzes the results of a patient's arterial blood gases (ABGs). Which finding would require immediate action? a. The bicarbonate level (HCO3-) is 31 mEq/L. b. The arterial oxygen saturation (SaO2) is 92%. c. The partial pressure of CO2 in arterial blood (PaCO2) is 31 mm Hg. d. The partial pressure of oxygen in arterial blood (PaO2) is 59 mm Hg.

ANS: D All the values are abnormal, but the low PaO2 indicates that the patient is at the point on the oxyhemoglobin dissociation curve where a small change in the PaO2 will cause a large drop in the O2 saturation and a decrease in tissue oxygenation. The nurse should intervene immediately to improve the patient's oxygenation.

34. A patient with pneumonia has a fever of 101.4° F (38.6° C), a nonproductive cough, and an oxygen saturation of 88%. The patient complains of weakness, fatigue, and needs assistance to get out of bed. Which nursing diagnosis should the nurse assign as the highest priority? a. Hyperthermia related to infectious illness b. Impaired transfer ability related to weakness c. Ineffective airway clearance related to thick secretions d. Impaired gas exchange related to respiratory congestion

ANS: D All these nursing diagnoses are appropriate for the patient, but the patient's oxygen saturation indicates that all body tissues are at risk for hypoxia unless the gas exchange is improved.

5. The correct procedure for selecting an oropharyngeal airway is to: a. measure from the tip of the nose to the ear lobe. b. measure from the mouth to the ear lobe. c. measure from the tip of the nose to the middle of the trachea. d. measure the airway from the corner of the patient's mouth to the angle of the jaw.

ANS: D An oropharyngeal airway's proper size is selected by holding the airway against the side of the patient's face and ensuring that it extends from the corner of the mouth to the angle of the jaw. If the airway is improperly sized, it will occlude the airway. Nasopharyngeal airways are measured by holding the tube against the side of the patient's face and ensuring that it extends from the tip of the nose to the ear lobe.

5. A patient has been admitted to the critical care unit with the diagnosis of acute respiratory distress syndrome (ARDS). Arterial blood gasses (ABGs) revealed an elevated pH and decreased PaCO2. The patient is becoming fatigued, and the health care provider orders a repeat ABG. The nurse anticipates the following results a. elevated pH and decreased PaCO2 c. decreased pH and decreased PaCO2 b. elevated pH and elevated PaCO2 d. decreased pH and elevated PaCO2

ANS: D Arterial blood gas analysis reveals a low PaO2 despite increases in supplemental oxygen administration (refractory hypoxemia). Initially, the PaCO2 is low as a result of hyperventilation, but eventually the PaCO2 increases as the patient fatigues. The pH is high initially but decreases as respiratory acidosis develops.

10. Auscultation of the anterior chest should be performed using which of the following sequences? a. Right side, top to bottom, then left side, top to bottom b. Left side, top to bottom, then right side, top to bottom c. Side to side, bottom to top d. Side to side, top to bottom

ANS: D Auscultation should be done in a systematic sequence: side to side, top to bottom, posteriorly, laterally, and anteriorly.

18. When assessing a patient with chronic obstructive pulmonary disease (COPD), the nurse finds a new onset of agitation and confusion. Which action should the nurse take first? a. Notify the health care provider. b. Check pupils for reaction to light. c. Attempt to calm and reorient the patient. d. Assess oxygenation using pulse oximetry.

ANS: D Because agitation and confusion are frequently the initial indicators of hypoxemia, the nurse's initial action should be to assess oxygen saturation. The other actions are also appropriate, but assessment of oxygenation takes priority over other assessments and notification of the health care provider.

15. A patient with acute respiratory distress syndrome (ARDS) who is intubated and receiving mechanical ventilation develops a right pneumothorax. Which action will the nurse anticipate taking next? a. Increase the tidal volume and respiratory rate. b. Increase the fraction of inspired oxygen (FIO2). c. Perform endotracheal suctioning more frequently. d. Lower the positive end-expiratory pressure (PEEP).

ANS: D Because barotrauma is associated with high airway pressures, the level of PEEP should be decreased. The other actions will not decrease the risk for pneumothorax.

19. Postural drainage with percussion and vibration is ordered twice daily for a patient with chronic bronchitis. Which intervention should the nurse include in the plan of care? a. Schedule the procedure 1 hour after the patient eats. b. Maintain the patient in the lateral position for 20 minutes. c. Perform percussion before assisting the patient to the drainage position. d. Give the ordered albuterol (Proventil) before the patient receives the therapy.

ANS: D Bronchodilators are administered before chest physiotherapy. Postural drainage, percussion, and vibration should be done 1 hour before or 3 hours after meals. Patients remain in each postural drainage position for 5 minutes. Percussion is done while the patient is in the postural drainage position.

5. In a patient who is hemodynamically stable, which procedure can be used to estimate the PaCO2 levels? a. PaO2/FIO2 ratio c. Residual volume (RV) b. A-a gradient d. End-tidal CO2

ANS: D Capnography is the measurement of exhaled carbon dioxide (CO2) gas; it is also known as end-tidal CO2 monitoring. Normally, alveolar and arterial CO2 concentrations are equal in the presence of normal V/Q relationships. In a patient who is hemodynamically stable, the end-tidal CO2 (Petco2) can be used to estimate the PaCO2. Normally, the Pao2/Fio2 ratio is greater than 286; the lower the value, the worse the lung function. The a?2-a gradient (P[a ?2- a]o2) is normally less than 20 mm Hg on room air for patients younger than 61 years. This estimate of intrapulmonary shunting is the least reliable clinically, but it is used often in clinical decision making. Residual volume is the amount of air left in the lung after maximal exhalation. A normal value is 1200 to 1300 mL.

16. When assessing the respiratory system of an older patient, which finding indicates that the nurse should take immediate action? a. Weak cough effort b. Barrel-shaped chest c. Dry mucous membranes d. Bilateral crackles at lung bases

ANS: D Crackles in the lower half of the lungs indicate that the patient may have an acute problem such as heart failure. The nurse should immediately accomplish further assessments, such as oxygen saturation, and notify the health care provider. A barrel-shaped chest, hyperresonance to percussion, and a weak cough effort are associated with aging. Further evaluation may be needed, but immediate action is not indicated. An older patient has a less forceful cough and fewer and less functional cilia. Mucous membranes tend to be drier.

12. An alcoholic and homeless patient is diagnosed with active tuberculosis (TB). Which intervention by the nurse will be most effective in ensuring adherence with the treatment regimen? a. Arrange for a friend to administer the medication on schedule. b. Give the patient written instructions about how to take the medications. c. Teach the patient about the high risk for infecting others unless treatment is followed. d. Arrange for a daily noon meal at a community center where the drug will be administered.

ANS: D Directly observed therapy is the most effective means for ensuring compliance with the treatment regimen, and arranging a daily meal will help ensure that the patient is available to receive the medication. The other nursing interventions may be appropriate for some patients but are not likely to be as helpful for this patient.

25. How much of the basal oxygen consumption is required by the pulmonary system during normal quiet breathing? a. 10% to 20% c. 3% to 5% b. 5% to 10% d. 1% to 2%

ANS: D During normal quiet ventilation, only 1% to 2% of basal oxygen consumption is required by the pulmonary system.

9. Which action will the nurse need to do when preparing to assist with the insertion of a pulmonary artery catheter? a. Determine if the cardiac troponin level is elevated. b. Auscultate heart and breath sounds during insertion. c. Place the patient on NPO status before the procedure. d. Attach cardiac monitoring leads before the procedure.

ANS: D Dysrhythmias can occur as the catheter is floated through the right atrium and ventricle, and it is important for the nurse to monitor for these during insertion. Pulmonary artery catheter insertion does not require anesthesia, and the patient will not need to be NPO. Changes in cardiac troponin or heart and breath sounds are not expected during pulmonary artery catheter insertion.

32. A patient who has just been admitted with community-acquired pneumococcal pneumonia has a temperature of 101.6° F with a frequent cough and is complaining of severe pleuritic chest pain. Which prescribed medication should the nurse give first? a. Codeine b. Guaifenesin (Robitussin) c. Acetaminophen (Tylenol) d. Piperacillin/tazobactam (Zosyn)

ANS: D Early initiation of antibiotic therapy has been demonstrated to reduce mortality. The other medications are also appropriate and should be given as soon as possible, but the priority is to start antibiotic therapy.

20. The nurse develops a teaching plan to help increase activity tolerance at home for an older adult with severe chronic obstructive pulmonary disease (COPD). Which instructions would be most appropriate for the nurse to include in the plan of care? a. Stop exercising when short of breath. b. Walk until pulse rate exceeds 130 beats/minute. c. Limit exercise to activities of daily living (ADLs). d. Walk 15 to 20 minutes daily at least 3 times/week.

ANS: D Encourage the patient to walk 15 to 20 minutes a day at least three times a week with gradual increases. Shortness of breath is normal with exercise and not an indication that the patient should stop. Limiting exercise to ADLs will not improve the patient's exercise tolerance. A 70-year-old patient should have a pulse rate of 120 or less with exercise (80% of the maximal heart rate of 150).

16. To verify the correct placement of an oral endotracheal tube (ET) after insertion, the best initial action by the nurse is to a. auscultate for the presence of bilateral breath sounds. b. obtain a portable chest x-ray to check tube placement. c. observe the chest for symmetric chest movement with ventilation. d. use an end-tidal CO2 monitor to check for placement in the trachea.

ANS: D End-tidal CO2 monitors are currently recommended for rapid verification of ET placement. Auscultation for bilateral breath sounds and checking chest expansion are also used, but they are not as accurate as end-tidal CO2 monitoring. A chest x-ray confirms the placement but is done after the tube is secured.

9. Which of the following is an example of a disorder with increased tactile fremitus? a. Emphysema c. Pneumothorax b. Pleural effusion d. Pneumonia

ANS: D Examples of disorders that increase tactile fremitus include pneumonia, lung cancer, and pulmonary fibrosis. Emphysema, pleural effusion, and pneumothorax are disorders that decrease fremitus.

11. A patient who has a history of chronic obstructive pulmonary disease (COPD) was hospitalized for increasing shortness of breath and chronic hypoxemia (SaO2 levels of 89% to 90%). In planning for discharge, which action by the nurse will be most effective in improving compliance with discharge teaching? a. Start giving the patient discharge teaching on the day of admission. b. Have the patient repeat the instructions immediately after teaching. c. Accomplish the patient teaching just before the scheduled discharge. d. Arrange for the patient's caregiver to be present during the teaching.

ANS: D Hypoxemia interferes with the patient's ability to learn and retain information, so having the patient's caregiver present will increase the likelihood that discharge instructions will be followed. Having the patient repeat the instructions will indicate that the information is understood at the time, but it does not guarantee retention of the information. Because the patient is likely to be distracted just before discharge, giving discharge instructions just before discharge is not ideal. The patient is likely to be anxious and even more hypoxemic than usual on the day of admission, so teaching about discharge should be postponed.

35. The nurse is caring for a patient with a subarachnoid hemorrhage who is intubated and placed on a mechanical ventilator with 10 cm H2O of peak end-expiratory pressure (PEEP). When monitoring the patient, the nurse will need to notify the health care provider immediately if the patient develops a. oxygen saturation of 93%. b. respirations of 20 breaths/minute. c. green nasogastric tube drainage. d. increased jugular venous distention.

ANS: D Increases in jugular venous distention in a patient with a subarachnoid hemorrhage may indicate an increase in intracranial pressure (ICP) and that the PEEP setting is too high for this patient. A respiratory rate of 20, O2 saturation of 93%, and green nasogastric tube drainage are within normal limits.

3. On admission, a patient presents as follows: pH, 7.38; respiratory rate, 24 breaths/min, regular, pursed-lip breathing; PaO2, 66 mm Hg; heart rate, 112 beats/min, sinus tachycardia; PaCO2, 52 mm Hg; blood pressure, 110/68 mm Hg; HCO3-, 24 mEq/L; and SpO2, 90% on O2 2 L/min nasal cannula. What treatment would the physician or nurse practitioner likely order for this patient? a. Increase O2 to 6 L/min. b. Prepare for emergency intubation. c. Administer 1 ampule of sodium bicarbonate. d. Repeat ABG testing in 4 hours.

ANS: D Increasing the FiO2 on this patient could decrease the respiratory rate and increase the severity of the patient's CO2 retention. The patient's arterial blood gas (ABG) values do not warrant intubation at this time. Additional sodium bicarbonate is not indicated because this patient has a fully compensated pH. A repeat ABG may be ordered to assess the patient's ongoing respiratory status. Other factors must be considered when reviewing a patient's ABGs, including oxygen saturation, oxygen content, base excess and deficit, and anion gap analysis.

21. Which nursing interventions included in the care of a mechanically ventilated patient with acute respiratory failure can the registered nurse (RN) delegate to an experienced licensed practical/vocational nurse (LPN/LVN) working in the intensive care unit? a. Assess breath sounds every hour. b. Monitor central venous pressures. c. Place patient in the prone position. d. Insert an indwelling urinary catheter.

ANS: D Insertion of indwelling urinary catheters is included in LPN/LVN education and scope of practice and can be safely delegated to an LPN/LVN who is experienced in caring for critically ill patients. Placing a patient who is on a ventilator in the prone position requires multiple staff, and should be supervised by an RN. Assessment of breath sounds and obtaining central venous pressures require advanced assessment skills and should be done by the RN caring for a critically ill patient

30. The nurse completes an admission assessment on a patient with asthma. Which information given by patient is most indicative of a need for a change in therapy? a. The patient uses albuterol (Proventil) before any aerobic exercise. b. The patient says that the asthma symptoms are worse every spring. c. The patient's heart rate increases after using the albuterol (Proventil) inhaler. d. The patient's only medications are albuterol (Proventil) and salmeterol (Serevent).

ANS: D Long-acting β2-agonists should be used only in patients who also are using an inhaled corticosteroid for long-term control. Salmeterol should not be used as the first-line therapy for long-term control. Using a bronchodilator before exercise is appropriate. The other information given by the patient requires further assessment by the nurse, but is not unusual for a patient with asthma.

46. Which intervention will the nurse include in the plan of care for a patient who is diagnosed with a lung abscess? a. Teach the patient to avoid the use of over-the-counter expectorants. b. Assist the patient with chest physiotherapy and postural drainage. c. Notify the health care provider immediately about any bloody or foul-smelling sputum. d. Teach about the need for prolonged antibiotic therapy after discharge from the hospital.

ANS: D Long-term antibiotic therapy is needed for effective eradication of the infecting organisms in lung abscess. Chest physiotherapy and postural drainage are not recommended for lung abscess because they may lead to spread of the infection. Foul smelling and bloody sputum are common clinical manifestations in lung abscess. Expectorants may be used because the patient is encouraged to cough.

29. The nurse completes discharge teaching for a patient who has had a lung transplant. The nurse evaluates that the teaching has been effective if the patient makes which statement? a. "I will make an appointment to see the doctor every year." b. "I will stop taking the prednisone if I experience a dry cough." c. "I will not worry if I feel a little short of breath with exercise." d. "I will call the health care provider right away if I develop a fever."

ANS: D Low-grade fever may indicate infection or acute rejection so the patient should notify the health care provider immediately if the temperature is elevated. Patients require frequent follow-up visits with the transplant team. Annual health care provider visits would not be sufficient. Home oxygen use is not an expectation after lung transplant. Shortness of breath should be reported. Low-grade fever, fatigue, dyspnea, dry cough, and oxygen desaturation are signs of rejection. Immunosuppressive therapy, including prednisone, needs to be continued to prevent rejection.

18. A lobectomy is scheduled for a patient with stage I non-small cell lung cancer. The patient tells the nurse, "I would rather have chemotherapy than surgery." Which response by the nurse is most appropriate? a. "Are you afraid that the surgery will be very painful?" b. "Did you have bad experiences with previous surgeries?" c. "Surgery is the treatment of choice for stage I lung cancer." d. "Tell me what you know about the various treatments available."

ANS: D More assessment of the patient's concerns about surgery is indicated. An open-ended response will elicit the most information from the patient. The answer beginning, "Surgery is the treatment of choice" is accurate, but it discourages the patient from sharing concerns about surgery. The remaining two answers indicate that the nurse has jumped to conclusions about the patient's reasons for not wanting surgery. Chemotherapy is the primary treatment for small cell lung cancer. In non-small cell lung cancer, chemotherapy may be used in the treatment of nonresectable tumors or as adjuvant therapy to surgery.

8. A patient is admitted with active tuberculosis (TB). The nurse should question a health care provider's order to discontinue airborne precautions unless which assessment finding is documented? a. Chest x-ray shows no upper lobe infiltrates. b. TB medications have been taken for 6 months. c. Mantoux testing shows an induration of 10 mm. d. Three sputum smears for acid-fast bacilli are negative.

ANS: D Negative sputum smears indicate that Mycobacterium tuberculosis is not present in the sputum, and the patient cannot transmit the bacteria by the airborne route. Chest x-rays are not used to determine whether treatment has been successful. Taking medications for 6 months is necessary, but the multidrug-resistant forms of the disease might not be eradicated after 6 months of therapy. Repeat Mantoux testing would not be done because the result will not change even with effective treatment.

38. The nurse is performing tuberculosis (TB) skin tests in a clinic that has many patients who have immigrated to the United States. Which question is most important for the nurse to ask before the skin test? a. "Is there any family history of TB?" b. "How long have you lived in the United States?" c. "Do you take any over-the-counter (OTC) medications?" d. "Have you received the bacille Calmette-Guérin (BCG) vaccine for TB?"

ANS: D Patients who have received the BCG vaccine will have a positive Mantoux test. Another method for screening (such as a chest x-ray) will need to be used in determining whether the patient has a TB infection. The other information also may be valuable but is not as pertinent to the decision about doing TB skin testing.

8. Patients with left-sided pneumonia may benefit from placing them in which of the following positions? a. Reverse Trendelenburg c. On the left side b. Supine d. On the right side

ANS: D Patients with unilateral lung disease should be positioned with the healthy lung in a dependent position. Because gravity normally facilitates preferential ventilation and perfusion to the dependent areas of the lungs, the best gas exchange would take place in the dependent areas of the lungs. Thus, the goal of positioning is to place the least affected area of the patient's lung in the most dependent position. Patients with unilateral lung disease should be positioned with the healthy lung in a dependent position.

35. The nurse supervises unlicensed assistive personnel (UAP) who are providing care for a patient with right lower lobe pneumonia. The nurse should intervene if which action by UAP is observed? a. UAP splint the patient's chest during coughing. b. UAP assist the patient to ambulate to the bathroom. c. UAP help the patient to a bedside chair for meals. d. UAP lower the head of the patient's bed to 15 degrees.

ANS: D Positioning the patient with the head of the bed lowered will decrease ventilation. The other actions are appropriate for a patient with pneumonia.

14. Which of the following statements best describes the effects of positive-pressure ventilation on cardiac output? a. Positive-pressure ventilation increases intrathoracic pressure, which increases venous return and cardiac output. b. Positive-pressure ventilation decreases venous return, which increases preload and cardiac output. c. Positive-pressure ventilation increases venous return, which decreases preload and cardiac output. d. Positive-pressure ventilation increases intrathoracic pressure, which decreases venous return and cardiac output.

ANS: D Positive-pressure ventilation increases intrathoracic pressure, which decreases venous return to the right side of the heart. Impaired venous return decreases preload, which results in a decrease in cardiac output.

13. Preset positive pressure used to augment the patient's inspiratory effort is known as a. positive end-expiratory pressure (PEEP). b. continuous positive airway pressure (CPAP). c. pressure control ventilation (PCV). d. pressure support ventilation (PSV).

ANS: D Preset positive pressure used to augment the patient's inspiratory efforts is known as pressure support ventilation. With continuous positive airway pressure, positive pressure is applied during spontaneous breaths; the patient controls rate, inspiratory flow, and tidal volume. Positive end-expiratory pressure is positive pressure applied at the end of expiration of ventilator breaths.

47. The nurse provides discharge teaching for a patient who has two fractured ribs from an automobile accident. Which statement, if made by the patient, would indicate that teaching has been effective? a. "I am going to buy a rib binder to wear during the day." b. "I can take shallow breaths to prevent my chest from hurting." c. "I should plan on taking the pain pills only at bedtime so I can sleep." d. "I will use the incentive spirometer every hour or two during the day."

ANS: D Prevention of the complications of atelectasis and pneumonia is a priority after rib fracture. This can be ensured by deep breathing and coughing. Use of a rib binder, shallow breathing, and taking pain medications only at night are likely to result in atelectasis.

19. Determination of oxygenation status by oxygen saturation alone is inadequate. What other value must be known? a. pH c. HCO3- b. PaCO2 d. Hemoglobin (Hgb)

ANS: D Proper evaluation of the oxygen saturation level is vital. For example, an Sao2 of 97% means that 97% of the available hemoglobin is bound with oxygen. The word available is essential to evaluating the Sao2 level because the hemoglobin level is not always within normal limits and oxygen can bind only with what is available.

23. A 55-year-old patient with increasing dyspnea is being evaluated for a possible diagnosis of chronic obstructive pulmonary disease (COPD). When teaching a patient about pulmonary function testing (PFT) for this condition, what is the most important question the nurse should ask? a. "Are you claustrophobic?" b. "Are you allergic to shellfish?" c. "Do you have any metal implants or prostheses?" d. "Have you taken any bronchodilators in the past 6 hours?"

ANS: D Pulmonary function testing will help establish the COPD diagnosis. Bronchodilators should be avoided at least 6 hours before the test. PFTs do not involve being placed in an enclosed area such as for magnetic resonance imaging (MRI). Contrast dye is not used for PFTs. The patient may still have PFTs done if metal implants or prostheses are present, as these are contraindications for an MRI.

10. A patient with chronic obstructive pulmonary disease (COPD) has a nursing diagnosis of impaired breathing pattern related to anxiety. Which nursing action is most appropriate to include in the plan of care? a. Titrate oxygen to keep saturation at least 90%. b. Discuss a high-protein, high-calorie diet with the patient. c. Suggest the use of over-the-counter sedative medications. d. Teach the patient how to effectively use pursed lip breathing.

ANS: D Pursed lip breathing techniques assist in prolonging the expiratory phase of respiration and decrease air trapping. There is no indication that the patient requires oxygen therapy or an improved diet. Sedative medications should be avoided because they decrease respiratory drive.

7. The nurse teaches a patient who has asthma about peak flow meter use. Which action by the patient indicates that teaching was successful? a. The patient inhales rapidly through the peak flow meter mouthpiece. b. The patient takes montelukast (Singulair) for peak flows in the red zone. c. The patient calls the health care provider when the peak flow is in the green zone. d. The patient uses albuterol (Proventil) metered dose inhaler (MDI) for peak flows in the yellow zone.

ANS: D Readings in the yellow zone indicate a decrease in peak flow. The patient should use short-acting β2-adrenergic (SABA) medications. Readings in the green zone indicate good asthma control. The patient should exhale quickly and forcefully through the peak flow meter mouthpiece to obtain the readings. Readings in the red zone do not indicate good peak flow, and the patient should take a fast-acting bronchodilator and call the health care provider for further instructions. Singulair is not indicated for acute attacks but rather is used for maintenance therapy.

24. A young adult patient with cystic fibrosis (CF) is admitted to the hospital with increased dyspnea. Which intervention should the nurse include in the plan of care? a. Schedule a sweat chloride test. b. Arrange for a hospice nurse visit. c. Place the patient on a low-sodium diet. d. Perform chest physiotherapy every 4 hours.

ANS: D Routine scheduling of airway clearance techniques is an essential intervention for patients with CF. A sweat chloride test is used to diagnose CF, but it does not provide any information about the effectiveness of therapy. There is no indication that the patient is terminally ill. Patients with CF lose excessive sodium in their sweat and require high amounts of dietary sodium.

9. Aspiration can best be prevented by a. observing the amount given in the tube feeding. b. assessing the patient's level of consciousness. c. encouraging the patient to cough and to breathe deeply. d. positioning a patient in a semirecumbent position.

ANS: D Semirecumbency has been shown to decrease the risk of aspiration and inhibit the development of hospital-associated pneumonia.

3. Supplemental oxygen administration is usually effective in treating hypoxemia related to a. physiologic shunting. b. dead space ventilation. c. hypercapnia with a PaCO2 of 35 mm Hg. d. ventilation/perfusion mismatchin

ANS: D Supplemental oxygen administration is effective in treating hypoxemia related to alveolar hypoventilation and ventilation/perfusion mismatching. When intrapulmonary shunting exists, supplemental oxygen alone is ineffective. In this situation, positive pressure is necessary to open collapsed alveoli and facilitate their participation in gas exchange. Positive pressure is delivered via invasive and noninvasive mechanical ventilation. If the patient is also experiencing hypercapnia, the PaCO2 will be greater than 45 mm Hg. In patients with chronically elevated PaCO2 levels, these criteria must be broadened to include a pH less than 7.35.

3. Which of the following describes the major difference between tachypnea and hyperventilation? a. Tachypnea has increased rate; hyperventilation has decreased rate. b. Tachypnea has decreased rate; hyperventilation has increased rate. c. Tachypnea has increased depth; hyperventilation has decreased depth. d. Tachypnea has decreased depth; hyperventilation has increased depth.

ANS: D Tachypnea is manifested by an increase in the rate and decrease in the depth of ventilation. Hyperventilation is manifested by an increase in both the rate and depth of ventilation.

1. Which of the following arterial blood gas values would indicate a need for oxygen therapy? a. PaO2 of 80 mm Hg c. HCO3- of 24 mEq b. PaCO2 of 35 mm Hg d. SaO2 of 87%

ANS: D The amount of oxygen administered depends on the pathophysiologic mechanisms affecting the patient's oxygenation status. In most cases, the amount required should provide an arterial partial pressure of oxygen (PaO2) of greater than 60 mm Hg or an arterial hemoglobin saturation (SaO2) of greater than 90% during both rest and exercise.

13. If a patient sustained an injury to the apneustic center in the lower pons area, in which of the following areas would you most expect the patient to exhibit problems? a. Respiratory rate c. Respiratory rhythm b. Triggering exhalation d. Depth of respiration

ANS: D The apneustic center in the lower pons is thought to work with the pneumotaxic center to regulate the depth of inspiration. The pneumotaxic center in the pons is responsible for limiting inhalation and triggering exhalation. This response also facilitates control of the rate and pattern of respiration. The ventral respiratory group, located in the medulla, is responsible for inspiration and expiration during periods of increased ventilation.

16. Which blood gas parameter is the acid-base component that reflects kidney function? a. pH c. PaCO2 b. PaO2 d. HCO3-

ANS: D The bicarbonate (HCO3-) is the acid-base component that reflects kidney function. The bicarbonate is reduced or increased in the plasma by renal mechanisms. The normal range is 22 to 26 mEq/L. pH measures the hydrogen ion concentration of plasma. PaO2 measures partial pressure of oxygen dissolved in arterial blood plasma. PaCO2 measures the partial pressure of carbon dioxide dissolved in arterial blood plasma.

27. The central chemoreceptors are stimulated by a. decreased PaO2. c. decreased SaO2. b. increased PaO2. d. increased PaCO2.

ANS: D The central chemoreceptors respond to changes in the hydrogen ion concentration of that fluid. Ventilation is increased when the hydrogen ion concentration increases, as evidenced by a rise in the plasma arterial PaCO2.

5. The conducting airways a. participate in gas exchange. c. remove moisture from incoming air. b. cool the incoming airway. d. prevent the entry of foreign material.

ANS: D The conducting airways consist of the upper airways, the trachea, and the bronchial tree. Their major functions are to warm and humidify the inhaled air, prevent the entrance of foreign matter into the gas exchange areas, and serve as a passageway for air entering and leaving the gas exchange regions of the lungs.

37. After change-of-shift report on a ventilator weaning unit, which patient should the nurse assess first? a. Patient who failed a spontaneous breathing trial and has been placed in a rest mode on the ventilator b. Patient who is intubated and has continuous partial pressure end-tidal CO2 (PETCO2) monitoring c. Patient with a central venous oxygen saturation (ScvO2) of 69% while on bilevel positive airway pressure (BiPAP) d. Patient who was successfully weaned and extubated 4 hours ago and now has no urine output for the last 6 hours

ANS: D The decreased urine output may indicate acute kidney injury or that the patient's cardiac output and perfusion of vital organs have decreased. Any of these causes would require rapid action. The data about the other patients indicate that their conditions are stable and do not require immediate assessment or changes in their care. Continuous PETCO2 monitoring is frequently used when patients are intubated. The rest mode should be used to allow patient recovery after a failed SBT, and an ScvO2 of 69% is within normal limits.

2. On admission, a patient presents as follows: pH, 7.38; respiratory rate, 24 breaths/min, regular, pursed-lip breathing; PaO2, 66 mm Hg; heart rate, 112 beats/min, sinus tachycardia; PaCO2, 52 mm Hg; blood pressure, 110/68 mm Hg; HCO3-, 24 mEq/L; and SpO2, 90% on O2 2 L/min nasal cannula. Which of the following diagnoses would be most consistent with the above arterial blood gas values? a. Acute pulmonary embolism b. Acute myocardial infarction c. Congestive heart failure d. Chronic obstructive pulmonary disease

ANS: D The fact that the HCO3- level has increased enough to compensate for the increased pCO2 level indicates that this is not an acute condition because the kidneys can take several days to adjust. The other choices would present with a lower HCO3- level. The values indicate respiratory acidosis, and one of the potential causes is chronic obstructive pulmonary disease. Potential causes for respiratory alkalosis are pulmonary embolism, acute myocardial infarction, and congestive heart failure.

32. The nurse educator is evaluating the care that a new registered nurse (RN) provides to a patient receiving mechanical ventilation. Which action by the new RN indicates the need for more education? a. The RN increases the FIO2 to 100% before suctioning. b. The RN secures a bite block in place using adhesive tape. c. The RN asks for assistance to reposition the endotracheal tube. d. The RN positions the patient with the head of bed at 10 degrees.

ANS: D The head of the patient's bed should be positioned at 30 to 45 degrees to prevent ventilator-associated pneumonia. The other actions by the new RN are appropriate.

13. The nurse admits a patient who has a diagnosis of an acute asthma attack. Which statement indicates that the patient may need teaching regarding medication use? a. "I have not had any acute asthma attacks during the last year." b. "I became short of breath an hour before coming to the hospital." c. "I've been taking Tylenol 650 mg every 6 hours for chest-wall pain." d. "I've been using my albuterol inhaler more frequently over the last 4 days."

ANS: D The increased need for a rapid-acting bronchodilator should alert the patient that an acute attack may be imminent and that a change in therapy may be needed. The patient should be taught to contact a health care provider if this occurs. The other data do not indicate any need for additional teaching.

17. To maintain proper cuff pressure of an endotracheal tube (ET) when the patient is on mechanical ventilation, the nurse should a. inflate the cuff with a minimum of 10 mL of air. b. inflate the cuff until the pilot balloon is firm on palpation. c. inject air into the cuff until a manometer shows 15 mm Hg pressure. d. inject air into the cuff until a slight leak is heard only at peak inflation.

ANS: D The minimal occluding volume technique involves injecting air into the cuff until an air leak is present only at peak inflation. The volume to inflate the cuff varies with the ET and the patient's size. Cuff pressure should be maintained at 20 to 25 mm Hg. An accurate assessment of cuff pressure cannot be obtained by palpating the pilot balloon.

23. Which of the following conditions will commonly reveal breath sounds with inspiration greater than expiration on assessment? 1. Normal lung 2. Bronchiectasis 3. Emphysema 4. Acute bronchitis 5. Diffuse pulmonary fibrosis 6. Consolidating pneumonia a. 1, 2, 5 c. 2, 3, 4 b. 3, 5, 6 d. 1, 2, 4

ANS: D The normal lung, bronchiectasis, and acute bronchitis will commonly present with an inspiration greater than expiration ratio. Acute bronchitis can also have inspiration that equals expiration ratio as also seen with emphysema, diffuse pulmonary fibrosis, and consolidating pneumonia. Noting that many conditions present with the same findings affirms the need for further assessment and evaluation.

30. The nurse responds to a ventilator alarm and finds the patient lying in bed holding the endotracheal tube (ET). Which action should the nurse take next? a. Activate the rapid response team. b. Provide reassurance to the patient. c. Call the health care provider to reinsert the tube. d. Manually ventilate the patient with 100% oxygen.

ANS: D The nurse should ensure maximal patient oxygenation by manually ventilating with a bag-valve-mask system. Offering reassurance to the patient, notifying the health care provider about the need to reinsert the tube, and activating the rapid response team are also appropriate after the nurse has stabilized the patient's oxygenation.

18. Which of the following ABG values reflects compensation? a. pH, 7.26; PaCO2, 55 mm Hg; HCO3-, 24 mEq/L b. pH, 7.30; PaCO2, 32 mm Hg; HCO3-, 18 mEq/L c. pH, 7.48; PaCO2, 30 mm Hg; HCO3-, 22 mEq/L d. pH, 7.38; PaCO2, 58 mm Hg; HCO3-, 30 mEq/L

ANS: D The pH is within normal limits, and both the PaCO2 and the HCO3- values are abnormal. Compensated respiratory acidosis values include a pH of 7.35 to 7.39, PaCO2 above 45 mm Hg, and HCO3- above 26 mEq/L

36. A patient with a possible pulmonary embolism complains of chest pain and difficulty breathing. The nurse finds a heart rate of 142 beats/minute, blood pressure of 100/60 mmHg, and respirations of 42 breaths/minute. Which action should the nurse take first? a. Administer anticoagulant drug therapy. b. Notify the patient's health care provider. c. Prepare patient for a spiral computed tomography (CT). d. Elevate the head of the bed to a semi-Fowler's position.

ANS: D The patient has symptoms consistent with a pulmonary embolism (PE). Elevating the head of the bed will improve ventilation and gas exchange. The other actions can be accomplished after the head is elevated (and oxygen is started). A spiral CT may be ordered by the health care provider to identify PE. Anticoagulants may be ordered after confirmation of the diagnosis of PE.

19. An hour after a thoracotomy, a patient complains of incisional pain at a level 7 (based on 0 to 10 scale) and has decreased left-sided breath sounds. The pleural drainage system has 100 mL of bloody drainage and a large air leak. Which action is best for the nurse to take next? a. Milk the chest tube gently to remove any clots. b. Clamp the chest tube momentarily to check for the origin of the air leak. c. Assist the patient to deep breathe, cough, and use the incentive spirometer. d. Set up the patient controlled analgesia (PCA) and administer the loading dose of morphine.

ANS: D The patient is unlikely to take deep breaths or cough until the pain level is lower. A chest tube output of 100 mL is not unusual in the first hour after thoracotomy and would not require milking of the chest tube. An air leak is expected in the initial postoperative period after thoracotomy.

21. Four hours after mechanical ventilation is initiated for a patient with chronic obstructive pulmonary disease (COPD), the patient's arterial blood gas (ABG) results include a pH of 7.51, PaO2 of 82 mm Hg, PaCO2 of 26 mm Hg, and HCO3- of 23 mEq/L (23 mmol/L). The nurse will anticipate the need to a. increase the FIO2. b. increase the tidal volume. c. increase the respiratory rate. d. decrease the respiratory rate.

ANS: D The patient's PaCO2 and pH indicate respiratory alkalosis caused by too high a respiratory rate. The PaO2 is appropriate for a patient with COPD and increasing the respiratory rate and tidal volume would further lower the PaCO2.

23. When assessing a patient who has just arrived after an automobile accident, the emergency department nurse notes tachycardia and absent breath sounds over the right lung. For which intervention will the nurse prepare the patient? a. Emergency pericardiocentesis b. Stabilization of the chest wall with tape c. Administration of an inhaled bronchodilator d. Insertion of a chest tube with a chest drainage system

ANS: D The patient's history and absent breath sounds suggest a right-sided pneumothorax or hemothorax, which will require treatment with a chest tube and drainage. The other therapies would be appropriate for an acute asthma attack, flail chest, or cardiac tamponade, but the patient's clinical manifestations are not consistent with these problems.

49. After change-of-shift report, which patient should the nurse assess first? a. 72-year-old with cor pulmonale who has 4+ bilateral edema in his legs and feet b. 28-year-old with a history of a lung transplant and a temperature of 101° F (38.3° C) c. 40-year-old with a pleural effusion who is complaining of severe stabbing chest pain d. 64-year-old with lung cancer and tracheal deviation after subclavian catheter insertion

ANS: D The patient's history and symptoms suggest possible tension pneumothorax, a medical emergency. The other patients also require assessment as soon as possible, but tension pneumothorax will require immediate treatment to avoid death from inadequate cardiac output or hypoxemia.

20. The nurse is caring for a 78-year-old patient who was hospitalized 2 days earlier with community-acquired pneumonia. Which assessment information is most important to communicate to the health care provider? a. Scattered crackles bilaterally in the posterior lung bases. b. Persistent cough that is productive of blood-tinged sputum. c. Temperature of 101.5° F (38.6° C) after 2 days of IV antibiotic therapy. d. Decreased oxygen saturation to 90% with 100% O2 by non-rebreather mask.

ANS: D The patient's low SpO2 despite receiving a high fraction of inspired oxygen (FIO2) indicates the possibility of acute respiratory distress syndrome (ARDS). The patient's blood-tinged sputum and scattered crackles are not unusual in a patient with pneumonia, although they do require continued monitoring. The continued temperature elevation indicates a possible need to change antibiotics, but this is not as urgent a concern as the progression toward hypoxemia despite an increase in O2 flow rate.

21. A patient with severe chronic obstructive pulmonary disease (COPD) tells the nurse, "I wish I were dead! I'm just a burden on everybody." Based on this information, which nursing diagnosis is most appropriate? a. Complicated grieving related to expectation of death b. Ineffective coping related to unknown outcome of illness c. Deficient knowledge related to lack of education about COPD d. Chronic low self-esteem related to increased physical dependence

ANS: D The patient's statement about not being able to do anything for himself or herself supports this diagnosis. Although deficient knowledge, complicated grieving, and ineffective coping may also be appropriate diagnoses for patients with COPD, the data for this patient do not support these diagnoses.

26. The portion of total ventilation that participates in gas exchange is known as a. alveolar dead space. c. physiologic dead space. b. anatomic dead space. d. alveolar ventilation.

ANS: D The portion of total ventilation that participates in gas exchange is known as alveolar ventilation. The areas in the lungs that are ventilated but in which no gas exchange occurs are known as dead space regions. The conducting airways are referred to as anatomic dead space because they are ventilated but not perfused and therefore not able to participate in gas exchange. These unperfused alveoli are known as alveolar dead space. Anatomic dead space plus alveolar dead space is called physiologic dead space.

8. Which of the following arteries has the lowest oxygen concentration? a. Aorta c. Carotid b. Subclavian d. Pulmonary

ANS: D The pulmonary artery delivers blood from the right ventricle to the lungs, where they receive oxygen from the alveoli. The aorta, subclavian artery, and carotid artery are all supplied from the left ventricle, where the oxygen concentration is highest.

10. When assisting with the placement of a pulmonary artery (PA) catheter, the nurse notes that the catheter is correctly placed when the monitor shows a a. typical PA pressure waveform. b. tracing of the systemic arterial pressure. c. tracing of the systemic vascular resistance. d. typical PA wedge pressure (PAWP) tracing.

ANS: D The purpose of a PA line is to measure PAWP, so the catheter is floated through the pulmonary artery until the dilated balloon wedges in a distal branch of the pulmonary artery, and the PAWP readings are available. After insertion, the balloon is deflated and the PA waveform will be observed. Systemic arterial pressures are obtained using an arterial line and the systemic vascular resistance is a calculated value, not a waveform.

21. Using the illustrated technique, the nurse is assessing for which finding in a patient with chronic obstructive pulmonary disease (COPD)? (IMAGE IS A NURSE ASSESSING DIAPHRAGMATIC EXCURSION) a. Hyperresonance b. Tripod positioning c. Accessory muscle use d. Reduced chest expansion

ANS: D The technique for palpation for chest expansion is shown in the illustrated technique. Reduced chest movement would be noted on palpation of a patient's chest with COPD. Hyperresonance would be assessed through percussion. Accessory muscle use and tripod positioning would be assessed by inspection.

1. The trachea divides into the right and left mainstem bronchi at what point? a. Posterior larynx c. Epiglottis b. Cricoid cartilage d. Major carina

ANS: D The trachea is a hollow tube approximately 11 cm in length and 2.5 cm in diameter. It begins at the cricoid cartilage and ends at the bifurcation (the major carina) from which the two mainstem bronchi arise.

2. The nurse prepares a patient with a left-sided pleural effusion for a thoracentesis. How should the nurse position the patient? a. Supine with the head of the bed elevated 30 degrees b. In a high-Fowler's position with the left arm extended c. On the right side with the left arm extended above the head d. Sitting upright with the arms supported on an over bed table

ANS: D The upright position with the arms supported increases lung expansion, allows fluid to collect at the lung bases, and expands the intercostal space so that access to the pleural space is easier. The other positions would increase the work of breathing for the patient and make it more difficult for the health care provider performing the thoracentesis.

17. A patient hospitalized with chronic obstructive pulmonary disease (COPD) is being discharged home on oxygen therapy. Which instruction should the nurse include in the discharge teaching? a. Storage of oxygen tanks will require adequate space in the home. b. Travel opportunities will be limited because of the use of oxygen. c. Oxygen flow should be increased if the patient has more dyspnea. d. Oxygen use can improve the patient's prognosis and quality of life.

ANS: D The use of home oxygen improves quality of life and prognosis. Because increased dyspnea may be a symptom of an acute process such as pneumonia, the patient should notify the physician rather than increasing the oxygen flow rate if dyspnea becomes worse. Oxygen can be supplied using liquid, storage tanks, or concentrators, depending on individual patient circumstances. Travel is possible using portable oxygen concentrators.

19. The laboratory has just called with the arterial blood gas (ABG) results on four patients. Which result is most important for the nurse to report immediately to the health care provider? a. pH 7.34, PaO2 82 mm Hg, PaCO2 40 mm Hg, and O2 sat 97% b. pH 7.35, PaO2 85 mm Hg, PaCO2 45 mm Hg, and O2 sat 95% c. pH 7.46, PaO2 90 mm Hg, PaCO2 32 mm Hg, and O2 sat 98% d. pH 7.31, PaO2 91 mm Hg, PaCO2 50 mm Hg, and O2 sat 96%

ANS: D These ABGs indicate uncompensated respiratory acidosis and should be reported to the health care provider. The other values are normal or close to normal.

16. After receiving change-of-shift report on a medical unit, which patient should the nurse assess first? a. A patient with cystic fibrosis who has thick, green-colored sputum b. A patient with pneumonia who has crackles bilaterally in the lung bases c. A patient with emphysema who has an oxygen saturation of 90% to 92% d. A patient with septicemia who has intercostal and suprasternal retractions

ANS: D This patient's history of septicemia and labored breathing suggest the onset of ARDS, which will require rapid interventions such as administration of oxygen and use of positive pressure ventilation. The other patients should also be assessed as quickly as possible, but their assessment data are typical of their disease processes and do not suggest deterioration in their status.

5. The nurse palpates the posterior chest while the patient says "99" and notes absent fremitus. Which action should the nurse take next? a. Palpate the anterior chest and observe for barrel chest. b. Encourage the patient to turn, cough, and deep breathe. c. Review the chest x-ray report for evidence of pneumonia. d. Auscultate anterior and posterior breath sounds bilaterally.

ANS: D To assess for tactile fremitus, the nurse should use the palms of the hands to assess for vibration when the patient repeats a word or phrase such as "99." After noting absent fremitus, the nurse should then auscultate the lungs to assess for the presence or absence of breath sounds. Absent fremitus may be noted with pneumothorax or atelectasis. The vibration is increased in conditions such as pneumonia, lung tumors, thick bronchial secretions, and pleural effusion. Turning, coughing, and deep breathing is an appropriate intervention for atelectasis, but the nurse needs to first assess breath sounds. Fremitus is decreased if the hand is farther from the lung or the lung is hyperinflated (barrel chest).The anterior of the chest is more difficult to palpate for fremitus because of the presence of large muscles and breast tissue.

4. Which information will the nurse include in the asthma teaching plan for a patient being discharged? a. Use the inhaled corticosteroid when shortness of breath occurs. b. Inhale slowly and deeply when using the dry powder inhaler (DPI). c. Hold your breath for 5 seconds after using the bronchodilator inhaler. d. Tremors are an expected side effect of rapidly acting bronchodilators.

ANS: D Tremors are a common side effect of short-acting β2-adrenergic (SABA) medications and not a reason to avoid using the SABA inhaler. Inhaled corticosteroids do not act rapidly to reduce dyspnea. Rapid inhalation is needed when using a DPI. The patient should hold the breath for 10 seconds after using inhalers.

2. Which of the following lung sounds would be most likely heard in a client experiencing an asthma attack? a. Coarse rales c. Fine crackles b. Pleural friction rub d. Expiratory wheezes

ANS: D Wheezes are high-pitched, squeaking, whistling sounds produced by airflow through narrowed small airways. They are heard mainly on expiration but may also be heard throughout the ventilatory cycle. Depending on their severity, wheezes can be further classified as mild, moderate, or severe. Rales are crackling sounds produced by fluid in the small airways or alveoli or by the snapping open of collapsed airways during inspiration. A pleural friction rub is a dry, coarse sound produced by irritated pleural surfaces rubbing together and is caused by inflammation of the pleura.

14. A patient was admitted to the critical care unit after a left pneumonectomy. The patient is receiving 40% oxygen via a simple facemask. The morning chest radiography study reveals right lower lobe pneumonia. After eating breakfast, the patient suddenly vomits and aspirates. The first action that should be taken after the patient's aspiration event is a. lavaging his airway with normal saline. b. placing him on his back in a semi-Fowler position. c. administering manual ventilations with a resuscitation bag. d. suctioning his airway.

ANS: D When aspiration is witnessed, emergency treatment should be instituted to secure the airway and minimize pulmonary damage. The patient's head should be turned to the side, and the oral cavity and upper airway should be suctioned immediately to remove the gastric contents.

25. When caring for the patient with a pulmonary artery (PA) pressure catheter, the nurse observes that the PA waveform indicates that the catheter is in the wedged position. Which action should the nurse take next? a. Zero balance the transducer. b. Activate the fast flush system. c. Notify the health care provider. d. Deflate and reinflate the PA balloon.

ANS: D When the catheter is in the wedge position, blood flow past the catheter is obstructed, placing the patient at risk for pulmonary infarction. A health care provider or advanced practice nurse should be called to reposition the catheter. The other actions will not correct the wedging of the PA catheter.

16. A shift to the left of the oxyhemoglobin dissociation curve would cause which of the following? a. Better tissue perfusion c. Decreased hemoglobin affinity for O2 b. Lower SpO2 d. Impaired tissue oxygen delivery

ANS: D When the curve is shifted to the left, there is a higher arterial saturation for any given PaO2 because hemoglobin has an increased affinity for oxygen. Although the saturation is higher, oxygen delivery to the tissues is impaired because hemoglobin does not unload as easily


Ensembles d'études connexes

International Business Law Chapter 1

View Set

Phlebotomy Laboratory Mathematics Review

View Set

TDDE31 - MapReduce, Spark, Big Data for ML

View Set

Participation and Academic Honesty Verification

View Set

Chapter 24 Asepsis Practice Questions

View Set

ATI Comprehensive physical assessment of an adult post test

View Set